首页 高中语文必修4导学案

高中语文必修4导学案

举报
开通vip

高中语文必修4导学案高中语文必修4导学案 必修4 语文学科课程纲要 第四模块(主题) 本册教科书“阅读鉴赏”部分所选的课文,有中外戏剧,宋元词曲,古代人物传记及社会科学论文。“表达交流”中的写作部分主要训练议论文的写作,口语交际专题为辨论。“梳理探究”设计的专题为:逻辑和语文学习,走近文学大师,影视文化。“名著导读”介绍了莎士比亚戏剧和朱光潜的《谈美》。 一、课标要求 《普通高中语文课程标准》对必修4课程的学习主要有以下要求: 1(在阅读与鉴赏活动中,不断充实精神生活,完善自我人格,提升人生境界,逐步加深对个人与国家、个...

高中语文必修4导学案
高中语文必修4导学案 必修4 语文学科课程纲要 第四模块(主题) 本册教科书“阅读鉴赏”部分所选的课文,有中外戏剧,宋元词曲,古代人物传记及社会科学论文。“表达交流”中的写作部分主要训练议论文的写作,口语交际专题为辨论。“梳理探究” 设计 领导形象设计圆作业设计ao工艺污水处理厂设计附属工程施工组织设计清扫机器人结构设计 的专题为:逻辑和语文学习,走近文学大师,影视文化。“名著导读”介绍了莎士比亚戏剧和朱光潜的《谈美》。 一、课标要求 《普通高中语文课程 标准 excel标准偏差excel标准偏差函数exl标准差函数国标检验抽样标准表免费下载红头文件格式标准下载 》对必修4课程的学习主要有以下要求: 1(在阅读与鉴赏活动中,不断充实精神生活,完善自我人格,提升人生境界,逐步加深对个人与国家、个人与社会、个人与自然关系的思考和认识。 2(能阅读论述类、实用类、文学类等多种文本,根据不同的阅读目的、针对不同的阅读材料,灵活运用精读、略读、浏览、速读等阅读方法,提高阅读效率。 3(学习鉴赏中外文学作品,具有积极的鉴赏态度,注重审美体验,陶冶性情,涵养心灵。能感受形象,品味语言,领悟作品的丰富内涵,体会其艺术表现力,有自己的情感体验和思考。努力探索作品中蕴涵的民族心理和时代精神,了解人类丰富的社会生活和情感世界。 4(在阅读鉴赏中,了解诗歌、散文、小说、戏剧等文学体裁的基本特征及主要表现手法。了解作品所涉及的有关背景材料,用于分析和理解作品。 5(学习中国古代优秀作品,体会其中蕴涵的中华民族精神,为形成一定的传统文化底蕴奠定基础。学习从历史发展的角度理解古代作品的内容价值,从中汲取民族智慧;用现代观念审视作品,评价其积极意义与历史局限。 6(阅读浅易文言文,能借助注释和工具书,理解词句含义,读懂文章内容。了解并梳理常见的文言实词、文言虚词、文言句式的意义或用法,注重在阅读实践中举一反三。 7(进一步提高议论文写作能力,书面表达要观点明确,内容充实,感情真实健康;思路清晰连贯,能围绕中心选取材料,合理安排结构。在表达实践中发展形象思维和逻辑思维,发展创造性思维。 8(在讨论或辩论中积极主动发言,恰当地应对和辩驳。 二、教学目标 1. 教学戏剧单元:首先,介绍关于戏剧的一般知识。其次,引导学生掌握阅读剧本的基本方法。阅读剧本时要注意概括剧中的主要矛盾冲突,把握人物之间的关系,分析、品味人物的语言。最后,要重视学生的文学欣赏活动,以及在这个过程中产生的情感体验和认识。教学时不要脱离学生实际的阅读、欣赏活动,而孤立地进行所谓的人文精神教育。 2. 教学宋词单元:首先,介绍有关词这种文学样式的一般知识。其次,在教学实践中引导学生学习欣赏宋词。其一,整体把握词作的内容以及表达的思想感情;其二,既注意词作中带有作者浓厚的主观色彩的形象性词语,也要注意直接表露作者情绪的词语,以便准确、细致地揣摩作者的感情,并进行欣赏、品味。最后,指导学生诵读。诵读时,要在努力传达词人所表达的感情的同时,处理好节拍与节奏,恰当地把握语调与旋律,减少诵读中的随意性。 3. 教学杂文单元:首先,介绍一些关于随笔、杂文的文体知识。其次,引导学生注意理清文章的思路,提炼和概括文章的观点,教给学生阅读方法,培养、锻炼学生的抽象思维能力。最后,注意培养学生的独立思考的能力。论述类文章所阐述的观点和主张是作者的一家之言,它们有时有历史局限性,有的不一定完全合理、正确,有的在新形势下具有了新的意义,总之,都给学生留下了进一步思考的空间。 教学古代传记单元:首先,介绍传记的基本知识。其次,介绍传记的读法。看看文4. 章中选用了哪些材料,哪些材料是最主要的,从中可以看出传主是怎样的一个人,该怎样评价他。然后,引导学生注意课文多样的叙事写人的手法。最后,积累常用的文言词语和文言句式。 5. 教学表达交流部分:本册的写作教材继续训练学生写作议论文的能力。要求学生更多运用他们的分析、综合、抽象、概括等抽象思维能力,写出有一定独创性的、较有深度的议论文来。本册的口语交际训练是辩论。对高中生来说,辩论的要求不必太高。重视的是学生积极参与的态度及基本的论辩能力。教师在指导学生进行辩论时,应重点要求学生对于辩题形成科学的认识,认识到正反双方各自的合理性和不合理性,做到知己知彼,从而在辩论中能够最大程度的阐明己方的立场,反驳对方的“谬误”。 三、学习评价 在教学过程中,可以结合所教内容,采用学生自评、同学互评、教师评价等方式,对学生的学习情况进行评价,还可以采用同步 检测 工程第三方检测合同工程防雷检测合同植筋拉拔检测方案传感器技术课后答案检测机构通用要求培训 、单元检测、模块考试等方式来评价,以提高评价效率,达到促进全体学生全面发展的目的。 1.对一、二单元文学类文本阅读的评价,要重视评价学生对作品的整体把握,特别是对艺术形象的感悟和文本价值的独到理解,鼓励学生的个体体验和创造性的解读。 2.对三单元实用类文本阅读的评价,着重考察学生对文本内容的准确解读,以及对文本信息的筛选和处理能力。 3.对文言文阅读的评价,重点考察借助语感和必要的文言常识阅读浅易文言文的能力。考察学生对传统文化是否热爱和有兴趣,在文言文阅读中能否有意识地了解文化背景。 4.对议论文写作的评价,应考察是否认识到理论类文本的作用与价值;是否能有意识地通过议论类文本的写作来表达自己的观点。对口语交际的评价,应考察学生对具备良好口语交际素养的必要性和重要性的认识,积极参与口语交际实践活动,在交流中捕捉重要的信息,清楚、准确、自信地表达自己的观点和想法。 四、教学资源 音像资源:录音带,视频、音频材料。 书本资料: ?课本、教辅资料 ?参考书目: 《关汉卿戏曲集》(人民文学出版社1976年版) 《曹禺选集》(人民文学出版社1978年版) 《莎士比亚全集》(人民文学出版社1978年版) 《乐章集》(上海古籍出版社1988版) 《东坡乐府笺》(商务印书馆1958年版) 《稼轩词编年笺注》(古典文学出版社1957年版) 《李清照集校注》(人民文学出版社1979年版) 《且介亭杂文》(人民文学出版社1981年版) 《爱的艺术》(商务印书馆1987年版) 《史记》(中华书局1963年版) 《汉书》(中华书局1962年版) 《后汉书》(中华书局1965年版) 1 窦娥冤 3 《窦娥冤》导学案(一) 高一语文必修四 学习目标: 1(了解作家作品;了解元杂剧的特点以及有关知识。 2(了解课文内容。 重点难点: 1(元杂剧的剧本构成。 学法指导: 1(熟读课文,初步了解文意 2(查阅工具书或电子网络了解作家作品及戏曲有关常识并解决重点字 资料链接: (一)有关元杂剧: 1、 元杂剧的结构形式:“四折一楔子” 元杂剧的“折”相当于现代戏剧中的“幕”,由同一宫调的一套曲子组成,包括多场次。“楔子”相当于“序幕”或“过场戏”,多用在第一幕前的介绍剧情、人物,也有用在两幕之间的。 2、元杂剧的剧本构成: 唱词:按宫调、曲牌写成的韵文; 宾白:即说白。曲词为主,所以说白为宾。 科介:即现代戏剧中的“舞台提示”。 3、 元杂剧的角色: 旦:女角色。包括正旦(女主角)、副旦(女配角)、外旦(老年女角色)、小旦(少年女角色)四类。 末:男角色。也包括正末、副末、外末、小末四类。 净:俗称“大花脸”,多扮相貌、性格上有特异之处者。 丑:俗称“小花脸”。多扮次要角色。 此外,还有“卜儿”(老妇人)、孛老(老头儿)、孤(官员)、徕儿(小厮)、细酸(读书人)等角色。 4、其他:元杂剧多一人主唱,因而又可分为“旦本戏”(女角色主唱)、和“末本戏”(男角色主唱)两类。 (二)有关作者,吴汉卿号斋叟,金末元初大都人。元代杂剧的代表作家,也是我国戏剧史上最早也是最伟大的戏剧作家,他与郑光祖、白朴、马致远齐名,被称为“元曲四大家”。元代的阶级矛盾和民族矛盾十分尖锐,关汉卿不满社会现实,不仅写剧本,有时还登台演唱,借杂剧来揭露黑暗现实,寄托自己的社会理想。他一生创作杂剧有60多部,但大都散失,现仅存15部。《窦娥冤》、《救风尘》、《望江亭》、《单刀会》等流传很广。其中的《窦娥冤》是我国十大古典悲剧之一。1956年,他的名字被列入世界文化名人之列。 (三)东海孝妇的故事:东海有孝妇,少寡,亡子,养姑(婆婆)甚谨。姑欲嫁之,终于肯。姑谓邻人曰:“孝妇事我勤劳,哀其亡子守寡。我老,久累丁壮,奈何,其后,姑自经死。姑女告吏:“妇杀我母。”吏捕 孝妇,孝妇辞不杀姑,吏验治,孝妇自诬服。具狱上府,于公以为此妇养姑十余年,以孝闻,必不杀也。太守不听,于公争之,弗能得。乃抱其具狱,哭于府上,因辞疾去。太守竟论杀孝妇。郡中枯旱三年。(《汉书?于定国传》) (四)写作背景:故事发生在元朝,元代统治者是蒙古贵族和官僚,被统治者是劳苦大众。元统治者不仅对劳动者残酷盘剥,而且实行民族分化的种族歧视政策,以利于巩固统治地位。他们将全国人口分为四等:蒙古人、色目人、汉人和南人。统治者和地主阶级紧密勾结,共同压迫各族人民,可以说,劳动人民与统治者的阶级矛盾,是元朝社会的主要矛盾。杂剧《窦娥冤》反映的就是元朝社会的黑暗现实。 导学过程: 一、知识储备(A) 1(关汉卿,号 , 代戏曲作家,也是我国戏剧史上最早、最伟大的戏剧作家。代表作品有 、 、 、 等。《窦娥冤》全名 ,共 ,是我国十大古典悲剧之一。关汉卿的 和王实甫的 代表了元杂剧的最高成就。 2(元曲四大家为“关”“马”“郑”“白”。其代表作是关汉卿的 ,马致远的 ,白朴的 ,郑光祖的 。 3(元杂剧:元代形成的一种把 、 、 等有机结合起来的戏剧形式。剧本结构一般分为 ,有的还加一个 ,剧本由 、 、 三部分组成。每折用同一宫调的若干曲牌组成套曲演唱,全用北方曲调,由 或 一人独唱至终。 4(正音 刽子手( )、亲眷( )、嗟怨( )、杳无音信( )、前合后偃( )、 ((((( 尸骸( )、冤枉( )、荒阡( )、亢旱( )、鲍老儿( )、瀽( )、 (((((( 5(课文出现 个曲牌,都属于 宫调。 6、元杂剧的四大悲剧:《 》(关汉卿)、 《汉宫秋》( )、 《 》(白朴)、 《赵氏孤儿》( ) 7、元杂剧的四大爱情剧:《拜月亭》( )、 《 》(王实甫)、 《墙头马上》( )、 《 》(郑光祖) 8、说出下列元杂剧中常见的口语、衬字的意义 行动些: 没来由: 不提防: 只合: 哥哥行: 则落的: 也么哥: 二、预习测试(B) 1、关于元杂剧的一些分析判断,有错误的一项是( ) A(元杂剧是在金宫本和诸宫调的直接影响下融合各种表演艺术形式而形成的一种完整的戏剧形式。元杂剧一般是四折,一折相当于现代话剧一幕,是戏剧故事发展的一个大段落,也是音乐组织的单元。 B(元杂剧中的说白包括人物的对白和独白,用以展开剧情和揭示人物性格冲突。白,有散文,也有韵文。科是戏剧动作的总称,包括一般的舞台程式和武打、舞蹈。 C(元杂剧每本戏只有一个主角,男主角称正末,女主角称正旦;此外还有净、丑、杂等。 D(元代杂剧和散曲合称“元曲”。杂剧 规定 关于下班后关闭电源的规定党章中关于入党时间的规定公务员考核规定下载规定办法文件下载宁波关于闷顶的规定 ,每一折戏,唱词一套曲子,每支曲子用一个宫调。 、下列加点字的注音不完全正确的一项是( ) 2 A(盗跖(zhí) 嗟怨(jiē) 前合后偃(yǎn) B(罪愆(qiān) 衔冤(xián) 湛湛青天(zhàn) C(错勘(kān) 亢旱(kàng) 苌弘化碧(cháng) i) 尸骸(hái) 哥哥行(háng) D(埋怨(má 3、对下列加点词语的解释,完全正确的一项是( ) A(可怎生糊突了盗跖、颜渊(混淆) (( 则是看你死去的孩儿面上(只当是) ((( B.念窦娥葫芦提当罪愆(糊涂) (( 你道是暑气暄(温暖) ((( C(也不见得湛湛青天(清明) (((( 此后遇着冬时年节,月一十五(每月十五) (((( D(念窦娥从前以往干家缘(操劳家务) ((( 不知亢旱三年的说话准也不准(抵御旱灾) (( 4、下列人物和典故对应正确的一项是( ) 望帝啼鹃 苌弘化碧 六月飞霜 亢旱三年 A 杜宇 苌弘 窦娥 东海孝妇 B 邹衍 苌弘 杜宇 于定国 C 杜宇 窦娥 邹衍 于定国 D 杜宇 苌弘 邹衍 东海孝妇 5、下列关于元杂剧的术语,解释完全正确的一项是( ) A(外,外末的简称,扮演青年男子;净,俗称“花脸”,扮演性格刚烈或粗暴的男子。 B(科,指示角色动作、表情和舞台效果。 C(正宫,端正好,滚绣球等均是曲牌名。 D(折,相当于现代的“幕”,所有的杂剧都只能是四折。 6、对元杂剧四大家的分析有误的一项是( ) A(关汉卿的主要作品除了《窦娥冤》、《救风尘》、《望江亭》等作品外,还有著名的《桃花扇》。 B(《汉宫秋》是马致远的代表作,他的《天净沙?秋思》也是元曲中的小令珍品。 C(郑光祖的代表作品是《倩女离魂》。 D(《墙头马上》是白朴的代表作。 7、对窦娥临刑前发出三桩誓愿的分析判断不准确的一项是( ) A(东海孝妇的誓愿出自于《汉书?于定国传》。 B(三桩誓愿使主人公的反抗性格达到了顶点,使全剧达到了高潮,悲剧气氛达到了最高点。 C(它塑造了一个勤劳、正直善良而又有强烈反抗精神的古代妇女形象。 D(三桩誓愿层层深入地表现了窦娥强烈的愤怒之情以及坚决的反抗精神。 、提问:在课文中共出现多少曲牌,都属什么宫调, 8 三、理清结构(本折戏由三部分组成) (C) 部分 起至段落 主要内容 第一部分 第二部分 第三部分 本课反思: 《窦娥冤》导学案(二) 高一语文必修四 学习目标: 1(赏析课文,认识窦娥的刚烈性格和反抗精神。 2(体会戏曲的语言特点,理解想像、夸张的艺术手法。 重点难点: 1(现实主义和浪漫主义相结合手法的运用与理解 学法指导: 1(熟读课文,详细了解文意。 2(查阅资料,深刻理解课文内容。 知识链接: 《窦娥冤》是中国十大悲剧之一的传统剧目,是一出具有较高文化价值、以广泛群众为基础的名剧, 据统计,我国约八十六个剧种上演过此剧。 《窦娥冤》 元?关汉卿作。写窦娥被无赖诬陷,又被官府错判斩刑的冤屈故事。全剧四折一楔子。剧 情是:楚州贫儒窦天章因无钱进京赶考,无奈之下将幼女窦娥卖给蔡婆家为童养媳。窦娥婚后丈夫去世,婆媳相依为命。蔡婆外出讨债时遇到流氓张驴儿父子,被其胁迫。张驴儿企图霸占窦娥,见她不从便想毒死蔡婆以要挟窦娥,不料误毙其父。张驴儿诬告窦娥杀人,官府严刑逼讯婆媳二人,窦娥为救蔡婆自认杀人,被判斩刑。窦娥在临刑之时指天为誓,死后将血溅白绫、六月降雪、大旱三年,以明己冤,后来果然都应验。三年后窦天章任廉访使至楚州,见窦娥鬼魂出现,于是重审此案,为窦娥申冤。《窦娥冤》全名《感天动地窦娥冤》,此剧现存版本有:明脉望馆藏《古今名家杂剧》本、《元曲选》壬集本、《酹江集》本、《元杂剧二种》本、《元人杂剧全集》本。 《窦娥冤》是关汉卿的代表作,也是我国古代悲剧的代表作。它的故事渊源于《列女传》中的《东海孝妇》。但关汉卿并没有局限在这个传统故事里,去歌颂为东海孝妇平反冤狱的于公的阴德;而是紧紧扣住当时的社会现实,用这段故事,真实而深刻的反映了元蒙统治下中国社会极端黑暗、极端残酷、极端混乱的悲剧时代,表现了中国人民坚强不屈的斗争精神和争取独立生存的强烈要求。它成功地塑造了“窦娥”这个悲剧主人公形象,使其成为元代被压迫、被剥削、被损害的妇女的代表,成为元代社会底层善良、坚强而走向反抗的妇女的典型。 《窦娥冤》全剧为四折一楔子,课文选的第三折,是全剧矛盾冲突的高潮部分,写窦娥被押赴刑场杀害的悲惨情景,揭露了元代吏治的腐败残酷,反映了当时的社会黑暗,歌颂了窦娥的善良心灵和反抗精神。 作品在艺术上,体现出现实主义与浪漫主义风格的融合。作品用丰富的想象和大胆的夸张,设计超现实的情节,显示出正义的强大力量,寄托了作者鲜明的爱憎,反映了广大人民伸张正义、惩治邪恶的愿望。 关汉卿戏曲的语言通俗自然,朴实生动,极富性格,评论家以“本色”二字概括其特色。课文中的曲词,都不事雕琢,感情真切,精练优美,浅显而深邃。 导学过程: 一、赏析第一部分: 1、《端正好》表现了窦娥怎样的思想感情, 2、.“游魂先赴森罗殿”采用了什么修辞手法,怎样理解, 3、《滚绣球》中有一句是说命运不由自己掌握,这句话是哪一句,它采用了什么诗歌手法, 4、引用“盗跖、颜渊”的典故有何作用, 5、《滚绣球》中有一句话揭露了社会的严重不公,是哪一句, 6、概括地说,《滚绣球》一曲表达了怎样的思想感情, 二、赏析第二部分: 1、窦娥为什么叫走前街而不走后街,这表现了她怎样的性格, 2、作者为什么要表现窦娥善良的性格, 3、作者再次利用婆媳对白,叙说窦娥的冤屈,在情节上起何作用, 4、窦娥临死前反复叮嘱婆婆要祭奠她,是出于怎样的思想感情, 5、窦娥说自己“没时没运,不明不暗,负屈衔冤”,这话怎样理解, 三、赏析第三部分: 1、朗读四支曲子,完成表格。 誓愿顺序 誓愿内容 所用典故 誓愿实质 第一桩 第二桩 第三桩 2、“无头愿”是什么意思, 3、“苌弘化碧、望帝啼鹃”是怎么回事,这两个典故在文中起何作用, 4、监斩官说:“这等三伏天道,你便有冲天的怨气,也召不得一片雪来„„”这一说在文中起何作用, 5、邹衍“六月飞霜”的典故起何作用, 6、窦娥为什么要发下“亢旱三年”的毒誓, 7、“东海孝妇”的典故起何作用, 8、三桩誓愿的实现采用了什么手法,说说这种手法的艺术效果, 9、这三桩誓愿之间,有何内在联系,在全剧中地位如何, 10、思考讨论:为何第一场戏中“怨天”,这里又寄希望于天,怎样理解这一矛盾, 四、窦娥的人物形象:(学生根据赏析概括) 五、概括主题:(学生根据赏析概括) 六、关于本色派的语言特点: 本课反思: 《窦娥冤》导学案(三) 高一语文必修四 学习目标: 1(通过窦娥这一冤案,认识元代社会的黑暗和统治者的残暴,以及人们的反抗精神 重点难点: 1(现实主义和浪漫主义相结合手法的运用与理解 学法指导: 比较阅读 能力提升 (D) 一、阅读《滚绣球》,完成文后题目。 [滚绣球]有日月朝暮悬,有鬼神掌着生死权。天地也~只合把清浊分辨,可怎生糊突了盗跖、颜渊,为善的受贫穷更命短,作恶的享富贵又寿延。天地也~做得个怕硬欺软,却原来也这般顺水推船~地也,你不分好歹何为地~天也,你错勘贤愚枉做天~哎,只落得两泪涟涟。 1(,滚绣球,是( ) A词牌 B宫调 C曲目 D曲牌 2(解释文中加点词 A合 B糊突 C错勘 D怎生 3(对这节文字,分析最恰当的是( ) A(运用了对偶、对比、借代等修辞手法,表现了窦娥呼天唤地,希望天地为她伸冤报仇的心理。 B(运用了对偶、对比、借喻等修辞手法,表现了窦娥在有冤难诉时的悲愤感情和对神权的否定。 C(运用了对偶、对比、借喻等修辞手法,表现了窦娥在有冤难诉时的悲愤感情及对不公平世道的控诉。 D(运用了对偶、对比、借代等修辞手法,表现了窦娥在有冤难诉时的悲愤感情及对黑暗统治的愤怒控诉。 二、《窦娥冤》中有两句唱词,两个版本文字不同,试分析其优劣。 《古名家杂剧》本:“地也,你不分好歹难为地;天也,我今日负屈衔冤哀告天~” 《元曲选》本:“地也,你不分好歹何为地~天也,你错勘贤愚枉作天~” 三、提问式阐述:文学史上公认课文所选部分是全剧的高潮。但在前两折里,已经把窦娥受害的故事情节交待得清楚明白。窦娥与张驴儿的冲突,窦娥在公堂上的斗争,都已在前两折里表现出来,那么高潮为什么出现在第三折里, 四、关汉卿 [ 南宫?一枝花 ] 不伏老 [ 尾 ] 我是个蒸不烂煮不熟捶不扁炒不爆响铛铛一粒铜豌豆,„„我也会围棋,会蹴鞠。会打圈,会插秧,会歌舞,会吹弹,会咽作,会双陆。你便是落了我牙(歪了我嘴、瘸了我腿、折了我手,天赐与我这几般儿歹症候,尚兀自不肯休。 从这"自叙"中,可以看出关汉卿是怎样一个人 ? 熟悉当时市井的多种技艺,性格坚强,追究人性的解放。 五、阅读下面元曲 [ 双调?殿前欢 ] 两首,完成两题。 失题 贯云石 畅幽哉,春风无处不楼台,一时怀抱俱无奈。总对天开,就渊明归去来, 怕鹤怨山禽怪,问甚功名在。酸斋是我,我是酸斋。 次酸斋韵 张可久 钓鱼台,十年不上野鸥猜。白云来往青山在,对酒开怀。欠伊周济世才,犯刘阮贪杯戒,还李杜吟诗债。酸斋笑我,我笑酸斋。 1、对这两首元曲文句的解说,错误的一项是:( ) , . "就渊明归去来",是用陶潜《归去来辞》的典故,意思说自己脱离官场,回归自然。 B. "欠伊周济世才",意思是说自己没有伊尹、周公那样的匡时济世的才干。 C. "犯刘阮贪杯戒"是作者以刘伶、阮籍自比。 D. "还李杜吟诗债"中的"李杜"指李白和杜甫。 2、关于这两首元曲,分析不恰当的一项是:( ) A. "钓鱼台,十年不上野鸥猜"与"就渊明归去来,怕鹤怨山禽怪"相应和, 都表达了二人由于久居樊笼,不觉之中将山林鸥鹤这些自然景物渐渐淡忘的叹惋之情。 B. "白云来往青山在,对酒开怀"与"畅幽哉,春风无处不楼台"也都表达了他们复返自然后的畅快适意。 C. 这两首曲都表现出作者在仕途上不得志后的无奈情怀。 D. 《失题》风格旷达飘逸、豪放淋漓,《次酸斋韵》则于豪迈放旷之中流露出凄清悲凉。 六、思考讨论:除了《窦娥冤》,文学史上还有很多作品运用浪漫主义手法,比如《孔雀东南飞》写刘兰芝和焦仲卿双双殉情,化为双飞鸟;比如《梁山伯与祝英台》《西游记》这些在现实生活中都是不可能的,但是为什么文学作品中会这样安排呢,(提示:可从作家、人物形象、艺术手法、社会现实等角度考虑) 七、趣味探究 窦娥与哈姆雷特 关汉卿是我国文学史上伟大的作家之一,是中国戏曲的奠基人,在世界文学史中被誉为“中国的莎士比亚”,因此关、莎二人的戏剧作品常被用于比较研究。窦娥与哈姆雷特,这两个著名的悲剧形象分别诞生于13世纪的中国和17世纪初的欧洲,在反封建的斗争精神方面存在着一些共性,但二者又有很大不同。 本课反思: 课后阅读: 写作特点 想象、夸张的艺术手法。 作品运用丰富的想象和大胆的夸张,设计了三桩誓愿的超现实情节,显示正义抗争的强大力量,寄托了作者鲜明的爱憎,反映了人民伸张正义、惩治邪恶的愿望。这是本折也是全剧刻画主人公形象最着力的一笔,是作品艺术性的集中体现,使悲剧气氛更浓烈,人物形象更突出,故事情节更生动,主题思想更深刻,既洋溢着浓郁的生活气息,又充满奇异的浪漫色彩,具有震撼人心的艺术力量。 难点点拨 窦娥是被昏官屈判死罪的,她为何在《滚绣球》一曲中指责天地鬼神,却又在最后发下三大奇愿,要感天动地来显示冤情, 在封建社会,人民不仅在人身权利及政治经济上受压迫,而且在思想上受毒害。按当时观念,天地鬼神明察世上是非,主持人间公道。封建官吏为了愚民,也每每以青天自况。窦娥开始也受神权思想影响,相信“青天大老爷”能主持正义,赏善罚恶。在残酷的现实面前,她觉醒过来,看清了“衙门自古向南开,就中无个不冤哉”的社会真相。在《滚绣球》一曲中,她猛烈地指责天地鬼神不分清浊,混淆是非,致使恶人横行,良善衔冤。窦娥对神权的大胆谴责,实质上是对封建统治的强烈控诉和根本否定。她那似岩浆迸射如山洪决堤般的愤激之词,反映了女主人公的觉醒意识和反抗精神,也折射出当时广大人民的反抗精神。 至于窦娥临刑前的三桩誓愿,正是作家世界观矛盾的一种反映。一方面关汉卿关心民情,洞察现实,通过窦娥指斥天地、发下三桩誓愿应验等情节表明自己对封建官吏的憎恨,对被压迫人民的同情,反映人民仇视封建统治,要求变革现实的时代精神。另一方面,作者毕竟是封建时代的剧作家,他尽管关心人民的疾苦,却不能意识到人民自身的力量;他虽然憎恶封建统治,却又找不到变革现实的出路,所以最终还是让天地动容清官平冤来解决问题,这也反映了作家的阶级局限和历史局限。当然,上天被窦娥感动,也说明窦娥冤大恨深。 读后感 关汉卿的剧作深刻揭露了元代社会的黑暗,是元代残酷的民族压迫和阶级压迫的一面镜子。关汉卿的代表作《窦娥冤》写一个弱小无靠的寡妇窦娥,在贪官桃杌的迫害下,被诬为“药死公公”,斩首示众。窦娥的冤案有巨大的典型意义,作家以“人命关天关地”的高度社会责任感,提出了封建社会里“官吏每(们)无心正法,使百姓有口难言”这个带普遍意义的问题,强烈地控诉了封建 制度 关于办公室下班关闭电源制度矿山事故隐患举报和奖励制度制度下载人事管理制度doc盘点制度下载 与民为敌、残民以逞的罪恶。 “有日月朝暮悬,有鬼神掌着生死权。天地也只合把清浊分辨,可怎生错看了盗跖颜渊,为善的受贫穷更命短,造恶的享富贵又寿延,天地也做得个怕硬欺软,却原来也这般顺水推船。地也,你不分好歹何为地,天也,你错勘贤愚枉做天~哎,只落得两泪涟涟。”第三折这〔滚绣球〕一曲,通过窦娥血泪的控诉,引起人们对封建社会的现实秩序与传统观念的怀疑,把窦娥悲剧的意义升华到一个新的高度。 《窦娥冤》导学案(一)参考答案 一、知识储备(A) 6、元杂剧的四大悲剧:《窦娥冤》(关汉卿)、《汉宫秋》(马致远)、《梧桐雨》(白朴)、《赵氏孤儿》(纪君祥) 7、元杂剧的四大爱情剧:《拜月亭》(关汉卿)、《西厢记》(王实甫)、《墙头马上》(白朴)、《倩女离魂》(郑光祖) 8、说出下列元杂剧中常见的口语、衬字的意义 行动些:走快些。 没来由:没缘由,无缘无故。 不提防:没想到。 只合:只应该。 哥哥行:哥哥那边。 则落的:只落得。也么哥:句尾助词,无实在意义。 二、1(D 每套曲子用一个宫调。2(D 3(A 4(D 5(B 6(A 7(C三桩誓愿中没有体现主人公身上勤劳、正直善良的性格特征。 三、理清结构(本折戏由三部分组成) (C) 第一:(开头~“哎,只落得两泪涟涟”)写窦娥指斥天地鬼神,揭露世道不公。 第二:(“(刽子云)快行动些”~“不明不暗,负屈衔冤”)写窦娥告别婆婆。 第三:(“(刽子喝科,云)兀那婆子靠后”)~结束)写窦娥发下三大誓愿。 《窦娥冤》导学案(二)参考答案 一、赏析第一部分: 1、(提示:表现了窦娥满腹冤屈与怨恨。) 2、(提示:讳饰,实际是说被杀头而死。) 3、(提示:“有日月朝暮悬,有鬼神掌着生死权”。采用了兴的手法。) 4、(提示:这里是用以指责天地好坏不分,黑白颠倒。) 5、(提示:为善的受贫穷更命短,造恶的享富贵又寿延。) 6、(提示:既表达了对天地不公的不满,又表现了主人公的反抗精神。) 二、赏析第二部分: 1、(提示:因为她不愿见到婆婆,怕引起婆婆伤心。这表现了她性格中善良的本性。) 2、(提示:窦娥的善良更显出冤屈之深,为后文的反抗精神起积淀作用。) 3、(提示:主要用以激发人物的内心冤屈,表现人物内心愤恨,为后文发三桩毒誓作铺垫。) 4、(提示:这个问题可从这几方面考虑: ?这正是体现窦娥难以舍别婆婆,表现了她对婆婆的挂念; ?安排婆婆做这些事,是让婆婆有所寄托,以免婆婆孤独痛苦或生轻生之念; ?可以增强悲剧气氛; ?白发人送黑发人,白发人祭奠黑发人,更加突出社会黑暗,无公道可言。) 5、(提示:因为现实难合人意,她在这种是非颠倒的社会中感到无能为力,只能感叹自己时运不济;另一方面这样说也可以安慰婆婆,减轻婆婆的悲伤,体现了她的孝顺。) 三、赏析第三部分: 1、朗读四支曲子,完成表格。 誓愿顺序 誓愿内容 所用典故 誓愿实质 第一桩 血溅白练 苌弘化碧 望帝啼鹃 希望刑场上人了解她的冤屈 第二桩 六月飞雪 飞霜六月因邹衍 希望自己的冤屈会在上天得到反应 第三桩 亢旱三年 不仅希望个人的冤屈得到伸张,而且 东海曾经孝妇冤 希望上天能够惩治邪恶 ( “亢旱三年”,楚州自然是颗粒无收,那些贪赃枉法,草菅人命的贪官污吏自然就无可搜刮。 ) 2、(提示:意思是说自己的誓愿无根无据,但正可以表明其冤情。) 3、(提示:这两个典故主要表明案情是假,冤情是真,也是窦娥用以证明第一桩誓愿并非不能灵验,因为她相信自有天理在。) 4、(提示:反衬窦娥的确冤情深重,怨气冲天。) 5、(可以衬托窦娥冤案是实,证明并暗示第二桩誓愿也能实现。) 6、(提示:她希望的不仅是个人的冤情得到伸张,而且希望上天能够惩治恶人。“亢旱三年”,楚州自然是颗粒无收,那些贪赃枉法,草菅人命的贪官污吏自然就无可搜刮。) 7、(提示:除了暗示第三桩誓愿将要实现之外,更能衬托窦娥的反抗精神。) 8、(提示:浪漫主义手法;既能证明窦娥冤情实在是真,有能增强悲剧气氛。从观众来看,符合一般民众善恶有报的心理,反映了人民伸张正义、惩治邪恶的愿望,能引起观众更多的共鸣;也表现了作者鲜明的爱憎。) 9、明确:这三桩誓愿,层层深入地表现了窦娥对自己无辜蒙冤的强烈愤怒、坚决反抗,是她反抗性格的顶点,也是全剧高潮。 10、明确:窦娥的诉说中,“怨天”和依赖于“天”的情感始终交织在一起,这种矛盾实质上反映了作家世界观的矛盾。一方面,他通过窦娥指斥天地鬼神,批判了封建统治者,表达了变革现实的愿望,另一方面,作为封建社会的一个文人,他虽关心人民的疾苦,但不能从根本上看到救民于水火、变革现实的道路和希 望,最终还是要借助“天”来为窦娥昭雪冤枉。 四、窦娥的人物形象:(学生根据赏析概括) 窦娥是一个饱受封建压迫与摧残的充满反抗精神的劳动妇女的形象。他善良勤劳、孝顺贤惠,同时又刚毅顽强,敢于与恶势力拼斗到底。她生于严酷的腐败的封建统治之下,注定要走向悲剧的结局。 五、概括主题:(学生根据赏析概括) 《窦娥冤》是我国十大古典悲剧之一。作者通过窦娥蒙受的千古奇冤,揭露了封建社会的黑暗,统治阶级的昏庸残暴,歌颂了窦娥的美好心灵和反抗精神。窦娥的形象壮美动人,她的悲剧性格,具有深刻的社会意义和强烈的感染力量。 六、关于本色派的语言特点: 《窦娥冤》一剧具有很高的艺术成就。在课文中,凝练而又明白如话的说白,优美而又有节奏的的唱词,深刻地展示了人物的内心世界,集中表现了窦娥的“怨”与“愤”。不少古代白话,如“只合”(只应该)、“怎生”(怎么)等都表述得十分生动。科白等辅助手段运用的也很好,配合唱词生动地表现了人物的复杂心情,推动了情节的发展。 《窦娥冤》导学案(三)参考答案 一、1(D 2(a、合:应该 b、糊突:混淆 C、错勘:错误地判断 d、怎生:怎么 3(D 二、《古》本第一句唱词用的是陈述语气,远不如《元》本用“何为地”这种质问的语气强烈;《古》本第二句是对天的哀告祈求,而《元》本在第二句则是对天的面对面的指责和否定。总之,相比而言,《元》本突出了窦娥不屈服于恶势力的反抗精神,使人物形象显得更加高大,也使作品的主题得到了丰富和深化。这些,正是《古》本的不足之处。 三、提问式阐述:文学史上公认课文所选部分是全剧的高潮。但在前两折里,已经把窦娥受害的故事情节交待得清楚明白。窦娥与张驴儿的冲突,窦娥在公堂上的斗争,都已在前两折里表现出来,那么高潮为什么出现在第三折里, 通过阅读课文,我们发现;第三折里仅有窦娥指天发誓,刑场受戮,故事情节简单,而构成窦娥矛盾冲突的对立面,如社会恶势力的代表张驴儿、元代残酷统治的典型官府都隐藏在幕后,没有冲突,就没有戏剧,更没有高潮,可见,全剧高潮出在第三折里是与戏剧本身的特点分不开的。 戏剧本身的特点是什么呢,那就是剧作家在剧本中,除了精心构思戏剧的矛盾冲突外,还要匠心独运地安排曲牌、唱词、说白,以便创造条件,让演员在舞台上尽情地演出。 四、关汉卿 [ 南宫?一枝花 ] 不伏老 [ 尾 ] 我是个蒸不烂煮不熟捶不扁炒不爆响铛铛一粒铜豌豆,„„我也会围棋,会蹴鞠。会打圈,会插秧,会歌舞,会吹弹,会咽作,会双陆。你便是落了我牙(歪了我嘴、瘸了我腿、折了我手,天赐与我这几般儿歹症候,尚兀自不肯休。 从这"自叙"中,可以看出关汉卿是怎样一个人 ? 熟悉当时市井的多种技艺,性格坚强,追究人性的解放。 五、 1、( B )2、(C ) 六、拓展延伸 思考讨论:除了《窦娥冤》,文学史上还有很多作品运用浪漫主义手法,比如《孔雀东南飞》写刘兰芝 和焦仲卿双双殉情,化为双飞鸟;比如《梁山伯与祝英台》《西游记》这些在现实生活中都是不可能的, 但是为什么文学作品中会这样安排呢,(提示:可从作家、人物形象、艺术手法、社会现实等角度考 虑) 七、趣味探究 窦娥与哈姆雷特 关汉卿是我国文学史上伟大的作家之一,是中国戏曲的奠基人,在世界文学史中被誉为“中国的莎士比亚”,因此关、莎二人的戏剧作品常被用于比较研究。窦娥与哈姆雷特,这两个著名的悲剧形象分别诞生于13世纪的中国和17世纪初的欧洲,在反封建的斗争精神方面存在着一些共性,但二者又有很大不同。 窦娥是被昏官屈判死罪的,她在【滚绣球】一曲中指责天地鬼神,发下三大奇愿,要感天动地来显示冤情。窦娥对神权的大胆谴责,实质上是对封建统治的强烈控诉和根本否定。她那似岩浆迸射如山洪决堤般的愤激之词,反映了女主人公的觉醒意识和反抗精神,也折射出当时广大人民的反抗精神。 哈姆雷特是丹麦王子。他的叔父克劳迪斯毒死了他的父亲,篡夺了王位。老国王的鬼魂显现,把自己被害的经过,告诉儿子哈姆雷特,要他报仇。他弄清真相后,决心杀死奸王,但又顾虑重重。就在下决心动手时,不幸错杀了恋人的父亲。奸王把他送往英国,在去英国途中他逃回丹麦,在一场由克劳迪斯安排的比剑中死去。他在临死前总算刺死了克劳迪斯,但他改变现实的重大理想并未实现。 哈姆雷特的主要对手是克劳迪斯,克劳迪斯身上集中了封建、资本主义社会的许多恶德。从这一点看,窦娥与哈姆雷特悲剧的根源都在社会。 但窦娥的命运从一开始就不能选择,哈姆雷特则不同。他是个有理想、好思索的人文主义者,因苦苦思考“生存还是毁灭”这个问题而迟迟不能采取积极行动,最后只能凭一时冲动,抱着宿命论观点行动起来,以至于最终和所爱的、所恨的人同归于尽。他的悲剧不仅仅在社会,还在人性本身。 2 雷 雨 2 高一语文必修四《雷雨》导学案A 学习目标: 1、知识目标: 了解一些戏剧知识、作家曹禺及作品《雷雨》的相关剧情。 2、能力目标: 鉴赏剧中人物个性化的语言,训练独立阅读能力和分析、鉴赏、评价能力。 3、情感目标: 了解具有封建阶级和资产阶级两重特点的周朴园在家庭和社会上的罪恶。 教学重、难点: 1、 在剧本的矛盾冲突中通过分析人物的语言来把握人物形象。 3、 认识尖锐的戏剧冲突和个性化的戏剧语言 学法指导: 、概括情节。 1 2、了解主要矛盾冲突和主要人物关系。 3、分析品味人物语言:A品味富有个性化的语言B品味富有动作性的语言C品味人物语言中丰富的潜台词。 4、把握和欣赏人物形象。 资料链接: 介绍戏剧常识(了解、掌握) 戏剧是一种综合的舞台艺术。我们这里讲的戏剧实际上是剧本,它是文学体裁的一种。戏剧离不开戏剧冲突,它表现在人物性格的冲突上,具体表现为一系列的动作。戏剧语言包括人物语言 (即台词) 和舞台说明。 按照不同的标准,戏剧可以分为不同种类: 按艺术形式和表现手法分为话剧如《雷雨》、歌剧如《白毛女》、舞剧如《丝路花雨》; 按剧情繁简和结构分为独幕剧、多幕剧 如《雷雨》; 按题材所反映的时代分为历史剧如《屈原》、现代剧如《雷雨》; 按矛盾冲突的性质分为悲剧如《屈原》、喜剧如《威尼斯商人》、正剧如《白毛女》 介绍作者(了解、识记) 曹禺(1910,1996),原名万家宝,湖北潜江人,我国杰出的剧作家和“当代语言艺术大 师”,和“巴金”“老舍”合称“巴老曹”。作品有《雷雨》《日出》《原野》《北京人》《王昭君》《蜕变》《明朗的天》《胆剑篇》(与人合作)等剧本。 曹禺擅长以现实主义的笔触,深入挖掘人物的内心世界,展现紧张、尖锐的戏剧冲突。戏剧氛围浓重,语言富有诗意。 学习过程: 一、速读课文,梳理情节(A级) ,学法指导:分析剧本的情节结构~一般都以场面为基本单位~划分场面的依据~一般是主要人物的上下场或主要场景的转换。请按这一标准给课文划分层次~归纳层意。, 明确: 第一部分: 第二部分: 二、分析戏剧中的矛盾冲突,揣摩人物的性格特点。(B级) (学法指导:欣赏戏剧作品~首先应了解戏剧的发展等有关知识。在对剧本这种文学体裁有了一定的认识之后~应着重阅读剧本~了解作品的戏剧冲突~看哪些冲突是主要的~冲突的实质是什么~进而明确这样的冲突表现了怎样的主题。, 课文节选部分主要刻画了四个人物,其关系为: 1、学习第一部分: (1)开头一段舞台说明有什么作用, 明确: (2)开始的时候,是鲁侍萍的什么引起了周朴园的注意呢, 明确: (3)文中有四处周朴园问鲁侍萍身份的地方,如 : “你——你贵姓,” “,抬起头来,你姓什么,” “,忽然立起,你是谁,” “,徐徐立起,哦~你~你~你是——” 与周朴园相对的,这四次表明身份的机会,鲁侍萍都没有揭穿身份。但是,从台词和舞台说明中,我们又看出,鲁侍萍并没有躲闪或者急于离开,反而希望引起周朴园的注意。由此看来,鲁侍萍一方面欲语还休,另一方面又欲罢不能,当时到底抱着什么样的心态呢, 明确: (4) 周朴园知道站在面前的正是三十年前被自己赶出家门的鲁侍萍的时候,为什么会惊恐万状,(提示:结合人物的身份性格) 明确: (5)、 周朴园保留家具、熟记生日、关窗习惯说明什么,周朴园是否是真的良心发现,深切地怀念鲁侍萍, 明确: (6)、周朴园毕竟是几十年来压榨工人的资本家,他在惊恐万状之后对鲁侍萍的态度有什么变化,(态度变化过程) 明确: 二、学习第二部分的内容 (B级) 1、 周朴园对亲生儿子鲁大海的态度怎样,(找出代表性的语言分析) 明确: 2、周鲁两家的矛盾冲突的根源是什么,(从社会身份地位等角度分析) 明确: 三、 人物形象分析: (C级) 1 、周朴园 ,提示:周朴园是一个由封建地主转化而成的资本家~他是这个大家庭的统治者。对于这个人物的评价~剧作者是这样说的:“周朴园这个人可以说是坏到家了~坏到连自己都不认为自己是坏人的程度。”通读课文~请找出最能体现其性格特点的语句来加以总结。, 明确: 对鲁侍萍始乱终弃,大年三十从家中赶走?, , 故意淹死二千多小工,发昧心才? , , 对鲁侍萍由“爱”到“赶”到“怀念”到“凶狠”? , , 总结:周朴园是个 2、鲁侍萍 来自乡下的温柔貌美贤惠的侍女? , , 默默承受三十多年的悲惨? , , 面对周朴园含血带泪地控诉三十年的悲惨遭遇?, , 撕毁周朴园的支票? , , “命,是不公平的命叫我来的”? , , 对周朴园的生日哄骗心软,不敢当众揭周朴园的底?, , 总结:鲁侍萍是个 3 、鲁大海 代表广大工人群众面对面地同周朴园谈判,斗争。他义正辞严地揭穿周朴园软硬兼施镇压工人的罢工的阴谋,揭露他制造事故淹死两千三百个小工以发横财的罪 恶。?, , 与周朴园的斗争暂告失败?, , 总结:鲁大海是个 学习小结: 高一语文必修四《雷雨》导学案B 一、 探究性学习:(C级) 1、个性化语言特点 语言是构成剧本的基础,戏剧的人物语言也叫台词,包括对话、独白、旁白等,是人物心理、动作的外现。因此,鉴赏剧本,品味人物语言至关重要。而品味人物语言的第一步就是要学会品味个性化的人物语言。所谓个性化,是指受人物的年龄、身份、经历、教养、环境等影响而形成的个性特点。由此我们可知,个性化语言能够准确地表达人物的思想感情。 ?、高度个性化 (1)示例: 周朴园对侍萍的身份有四次发问: 1(“你——你贵姓,” 2(,抬起头来,“你姓什么,” 3(,忽然立起,“你是谁,” 4(,徐徐立起,“哦~你~你~你是——” 明确:周朴园的第一问,用词文雅。为了显示自己的知识和教养而特意用“贵姓”来问,以突出自己的身份。当他怀疑对方的真实身份时,便一改文雅问法而为直接询问,“你姓什么,”表现出对下人一副不屑的样子。当他相当怀疑此人就是当年的梅侍萍时,便像疯狗一样跳了起来,怀着一丝侥幸心理问,“你是谁,”这充分显现了他的怀疑和紧张。当他断定眼前这人就是梅侍萍时,他完全被吓呆了,甚至连跳起来的力气也没有了,慢慢站起语不成句地问:“哦~你~你~你是——”这更表现了周朴园的恐惧、焦虑和失态。这四问,语言高度个性化,从开始极力显示自己的贵族风度,到最后的失态表演,步步深入,一层一层地揭去了周朴园的伪装,深刻揭露了他的虚伪性。 (2)试分析下列台词的含义: “,大哭,这真是一群强盗:,走到周萍面前,你是萍……凭——凭什么打我的儿子,” 学习反思: ?、丰富的潜台词 潜台词即是言中有言,意中有意,弦外有音。它实际上是语言的多意现象。“潜”,是隐藏的意 思,即语言的表层意思之内还有含有别的意思。通过潜台词可以窥见人物丰富的内心世界。 (1) 示例: 周朴园 (汗涔涔地)哦。 鲁侍萍 她不是小姐,她是无锡周公馆梅妈的女儿,她叫侍萍。 周朴园 (抬起头来)你姓什么, 鲁侍萍 我姓鲁,老爷。 明确:“你姓什么,”的言外之意就是:你怎么知道的这么多,也是同样的道理。再看“你姓什么,”的言外之意就是:你怎么知道得这么清楚, (2)试分析下列潜台词的言外之意 A鲁侍萍 老爷问这些闲事干什么,? 周朴园 这个人跟我们有点亲戚。 鲁侍萍 亲戚,? 周朴园 嗯,——我们想把她的坟墓修一修。 鲁侍萍 哦,——那用不着了。? 周朴园 怎么, 鲁侍萍 这个人现在还活着。? 言外之意就是: ? ? ? ? B可是她不是小姐,她也不贤惠,并且听说是不大规矩的。? C周朴园:好~痛痛快快的~你现在要多少钱吧~ 鲁侍萍:什么,? D周朴园:什么?鲁大海,他~我的儿子,? 言外之意分别是: ? ? ? 学习反思: 2、《雷雨》中有多出破折号~这些破折号起着不同的作用~请试分析下列各处破折号的作用。 ,提示:有解释说明、转移话题、声音延长、阻断停顿等作用) ?“哦。——老爷没有事了,”?, , ?“你——你贵姓 ,”?, , ?“不过是——他很大了~——并且他以为他母亲早就死了的。”?, , ?“我——我——我只要见见我的萍儿” ?, , ?“你从前在哈尔滨包修江桥~故意叫江堤出险~——”“你现在还——”??“我看你的性情好像没有大 改~——鲁贵像是个很不老实的人。”?,, ?“你叫警察杀了矿上许多工人~你还——”?, , 二、基础联系(B级) 1(下列加点字的读音,有错误的一项是„ ( ) A郁热(yù) 沉吟(yǐn) 汗涔涔(jīn) (((( B伺候(cì) 惊愕(â) 昧良心(mâi) ((( C离间(jiàn)卑鄙(bǐ) 哭哭啼啼(títí) (((( D谛听(dì) 烦躁(zào) 规规矩矩(juju) (((( 2(下列句子,没有错别字的一项是„„„( ) A(你看这些傢俱都是你从前顶喜欢的东西。 B(顺便把那箱子里的几件旧衬衣也捡出来。 C(侍萍的像貌有一天也会老得连你都不认识了。 D(侍萍走头无路,跳河自尽,幸而被救,从此流落他乡。 三、课外延伸(D级) 阅读《雷雨》第一幕中的一个片断,回答后面的问题 周朴园 (向四凤)倒了来。 四凤走到左边倒药。 周 冲 爸,妈不愿意,您何必这样强迫呢? 周朴园 你同你母亲都不知道自己的病在哪儿。(向蘩漪,低声)你喝了,就会完全好的。(见四凤犹 豫,指药)送到太大那里去。 蘩 漪 (忍顺地)好,先放在这儿。 四凤放下药碗。 周朴园 (?)你最好现在喝了它吧。 蘩 漪 (忽然)四凤,你把它拿走。 周朴园 (?)喝了它,不要任性,当着这么大的孩子。 蘩 漪 (声颤)我不想喝。 周朴园 冲儿,你把药端到母亲面前去。 周 冲 (反抗地)爸! 周朴园 (?)去! 周冲只好把药端到蘩漪面前。 周朴园 说,请母亲喝。 周 冲 (拿着药碗,手发颤)爸,您不要这样。 周朴园 你说什么? 周 萍 (低头,至周冲前,低声)听爸爸的话吧。爸爸的脾气你是知道的。 周 冲 (含泪,向着母亲)您喝吧,为我喝一点吧,要不然,爸的气是不会消的。 蘩 漪 (恳求地)留着我晚上喝不成吗? 周朴园 (?)蘩漪,当了母亲的人,处处应当替孩子着想,就是自己不保重身体,也应当替孩子做 个服从的榜样。 蘩 漪 (望望周朴园,又望望周萍,拿起药又放下)不!我喝不下! 周朴园 萍儿,劝你母亲喝下去! 周 萍 爸!我—— 周朴园 去,跪下,劝你的母亲。 周 萍 (走至蘩漪,向周朴园,求恕地)爸爸! 周朴园 (高声)跪下! 周萍望着蘩漪,蘩漪泪痕满面,周冲气得发抖。 周朴园 叫你跪下。(周萍正要下跪) 蘩 漪 (望着周萍,急促地)我喝,我现在喝!(喝了两口,眼泪又涌出来,望一望周朴园峻厉的眼 光和苦恼着的周萍,咽下愤恨,一气喝下)哦„„ (哭着,由右边饭厅跑下。) 7(文中?,?处是舞台说明,选出依次填人最恰当的一项是( ) A(忽然严厉地 怒视 冷峻地 不高兴地 B(不高兴地 忽然严厉地 怒视 冷峻地 C(怒视 不高兴地 忽然严厉地 冷峻地 D(冷峻地 怒视 不高兴地 忽然严厉地 8(请简要概括本片断剧情。(不超过10字) 答:,,,,,,,,,,,,,,,, 9(结合舞台说明,谈谈蘩漪的心理变化过程。 答:,,,,,,,,,,,,,, 10(“不要任性,当着这么大的孩子”这句台词有什么深层含意? 答:,,,,,,,,,,,,,, 11(简析周朴园的性格特征。 答:,,,,,,,,,,,,, 课后小结: 高一语文必修四《雷雨》导学案A参考答案 一、明确:第一部分:梅、周狭路相逢?家庭内部冲突第二部分:鲁、周针锋相对?社会阶级斗争 二、1、明确:交代了故事发生的地点,渲染了气氛,烘托了人物的烦躁、郁闷不安的情绪。 (2)明确:鲁侍萍的口音和关窗的举止 (3)明确: A鲁侍萍带着30年的积怨,想见一见当初摧毁自己命运的罪魁祸首。用自己的出现拷问这个人的灵魂,想看看他是什么样,自己在他眼中又是什么样。 B本以为抛弃自己的人三十年来一定逍遥快活,置自己于脑后,没有想到这里还保留30年前的痕迹,些许有些感动。 C自己要作为周当初犯罪的活证据,控诉他的罪行。 (4)明确:因为鲁侍萍的出现,将威胁到他的名誉、地位、家庭,他以前的种种罪恶将昭然于天下。他以为搬了家,离开了无锡,从前的罪恶无人知道,可鲁侍萍的突然出现,却打他个措手不及,惊恐万状正是他内心虚伪的表现。 2、明确:?说明周朴园没忘旧情,多少年来一直在“纪念”着她。周朴园的感情很复杂,他确实曾对温柔美丽的鲁侍萍动过情,但他对欺辱鲁侍萍的行径必须掩饰,因为在他看来,同“下人”有一些什么是有损门第的。到了晚年,身边的妻子繁漪很不驯服,儿子也对他敬而远之,他时时感到家庭生活不如意,感到寂寞孤独,因此怀念鲁侍萍,借以弥补他灵魂的空虚,使精神得到解脱。?其实,他的怀念是廉价的,是十分自私的。因此,当他知道鲁侍萍就是眼前的鲁妈,就立即声色俱变,以至于最后凶相毕露,辞退四凤和鲁贵,开除鲁大海,并声称“以后鲁家的人永远不许再到鲁家来。”他对鲁侍萍的欺辱、摧残,他对鲁侍萍的忽冷忽热、软硬兼施,充分显露了他的残忍、自私、冷酷、虚伪,是一个地地道道的伪君子。 4、明确:?他认出鲁侍萍后,首先是翻脸不认人,厉声责问“你来干什么,”“谁指使你来的,”妄图恐吓鲁侍萍,但鲁侍萍从几十年的压迫中已经把自己磨练得顽强勇敢,周朴园的责问只能引起鲁侍萍的反驳;?周朴园见硬的不行,又以“现在你我都是有子女的人”旧事“又何必重提呢”想稳住鲁侍萍;?然后又采用哄骗的手段,口口声声地表白他没有忘记旧情,——屋中陈列着从前她喜欢的家具,每一年都记得她的生日,保留着关窗的习惯,多少年来一直在“纪念”她;?最后,他拿出五千元的支票给鲁侍萍,希望平息三十年来的旧恨新仇。 二、1、明确:依然摆出资本家对待工人的冷酷的面孔和傲慢态度。他明知故问鲁大海“叫什么名字”“有什么事”,继续教训鲁大海:“只凭意气是不能交涉事情的”,讽刺、挖苦、嘲笑鲁大海说“傻小子,没有经验只会胡喊是不成的”,以显示他的老练和雍容。 2、明确:周朴园是带有浓厚封建性的资本家,鲁家母子是善良的下层劳动人民。他们之间阶级地位和思想意识的对立,使他们必然产生矛盾冲突。从整个戏剧来看,在错综复杂的矛盾冲突中,以周朴园为代表的资本家与以鲁侍萍为代表的下层劳动人民,资本家与工人之间的阶级冲突是最本质的冲突,这些矛盾的存在、发展,决定了其他矛盾的存在和发展。 三、1 、明确:对鲁侍萍始乱终弃,大年三十从家中赶走?冷酷、残忍、自私 故意淹死二千多小工,发昧心才?残忍、奸诈 对鲁侍萍由“爱”到“赶”到“怀念”到“凶狠”?虚伪、冷酷、自私 总结:(1)周朴园是一个资本家,他做事冷酷无情,对人冷淡,并且十分虚伪。他似乎在深情地怀念“已死”的侍萍,他打听侍萍的坟墓并表示要进行修理,但当知道眼前的人就是侍萍时,他却签了一张五千块的支票递给侍萍,想以此来了结所有的事,这正是周朴园伪善丑恶心理的曝露,集中得表现了他得处世哲学。 (2)周朴园是个由封建地主转化而成的资本家。他残忍、冷酷、自私、贪婪、而又虚伪。 鲁侍萍与周朴园的对话,从私生活的领域揭露了周朴园的反动阶级本性。 鲁大海与周朴园的对话,从 社会生活的领域 揭露了周朴园的反动阶级的本性。 从连续紧凑的戏剧冲突可见: 周朴园对侍萍先是爱,后是赶,赶走之后又怀念,他怀念的侍萍活脱脱地站在他面前时,他竟是那样的凶狠,对此应如何理解? 周朴园正是富有个性的活生生的资本家形象。他那矛盾的行为,以其性格的复杂性、多面性,显示出他作为董事长的周朴园的基本定性?? 真诚的伪善,伪善的真诚 。 2、明确:来自乡下的温柔貌美贤惠的侍女?心地善良、正直 默默承受三十多年的悲惨?刚强 面对周朴园含血带泪地控诉三十年的悲惨遭遇?倔强 撕毁周朴园的支票?骨气和尊严 “命,是不公平的命叫我来的”?尚不知道造成自己悲惨命运的根本原因 对周朴园的生日哄骗心软,不敢当众揭周朴园的底?斗争不坚决 总结:侍萍是一个受侮辱,被损害的女子,是旧中国 劳动妇女 的形象,正直、善良、刚毅、倔强 。 她自己被遗弃以及三十年挣扎的痛苦经历中,认清了周朴园的真面目。她虽然不懂得自己的悲惨命运是阶级压迫造成的,但已意识到她同周朴园之间事实上存在着阶级的壁垒。她知道自己的亲生儿子周萍是不会认她这个生母了,也断言自己的儿子,被周朴园称为闹得最凶的人鲁大海是不会认那个董事长做父亲的。她对过去的事,有的只是满腔的悲愤。她对周朴园不抱幻想,痛惜自己的女儿又走上了自己的老路,来伺侯周朴园的少爷。她把这归之于命运的安排,但最后又发誓以后永远不想再见到周朴园,表现了对自己的命运的抗争,对不公平的社会的控诉,从自己痛苦的经历和现实的磨练中坚强起来了,表现了她的自尊与刚强。( 分析侍萍不要钱的情节) 由于反动统治的毒害,在侍萍身上还存有 封建伦理观念 和宿命论的思想,因而认识不到自己的不幸是那个腐朽的社会制度造成的,却把原因归结为命运的安排。对周朴园,她不去公开揭露,反而在他表示他怀念她的时候心软了,这些都说明了旧社会不仅摧残了劳动人民的躯体,而且毒害了他们的灵魂 ,从另一方面 暴露了旧社会的黑暗 。 3 、鲁大海与周朴园,从血缘上看,他们是父子;从阶级关系上看,他们是你死我活的敌人在同周朴园的 斗争中表现出他是一个觉醒了的工人 ,代表广大工人群众面对面地同周朴园谈判,斗争。他义正辞严地揭穿周朴园软硬兼施镇压工人的罢工的阴谋,揭露他制造事故淹死两千三百个小工以发横财的罪 恶。他坚定、勇敢、无私、求实。他对资本家有着清醒、透彻的认识,资本家的威胁、讹诈、利诱等卑鄙 手段在他面前无计可施。他义无反顾地在反抗斗争的道路上走下去。( 反抗精神、坚强不屈的性格) 斗争中 显出经验的不足、鲁莽等弱点。 高一语文必修四《雷雨》导学案B参考答案 一、(1)明确:周朴园的第一问,用词文雅。为了显示自己的知识和教养而特意用“贵姓”来问,以突出自己的身份。当他怀疑对方的真实身份时,便一改文雅问法而为直接询问,“你姓什么,”表现出对下人一副不屑的样子。当他相当怀疑此人就是当年的梅侍萍时,便像疯狗一样跳了起来,怀着一丝侥幸心理问,“你是谁,”这充分显现了他的怀疑和紧张。当他断定眼前这人就是梅侍萍时,他完全被吓呆了,甚至连跳起来的力气也没有了,慢慢站起语不成句地问:“哦,你,你,你是——”这更表现了周朴园的恐惧、焦虑和失态。这四问,语言高度个性化,从开始极力显示自己的贵族风度,到最后的失态表演,步步深入,一层一层地揭去了周朴园的伪装,深刻揭露了他的虚伪性。 (2)(1)侍萍目睹了一场父亲对亲生儿子的陷害,哥哥对弟弟施淫威的人间悲剧,肝胆俱裂,于是她悲愤地指出“这真是一群强盗~”只有侍萍这个多重身份、曾多次受到周家伤害的人才能有此体会,也才能一针见血地指出周家是一群强盗。(2)侍萍想认分离多年的儿子周萍,但要信守不相认的承诺而使她欲言又止,加上周萍打了亲弟弟大海,面对如此令人痛心的场面,使得侍萍的感情由“想念”一下子变成了“愤慨”。一道森严的阶级壁垒已横在她面前,她的“萍„„凭——凭什么打我的儿子,”反映了她内心的矛盾与痛苦。也只有这个特殊身份的人才能说出这样的话。人物的身份、地位和性格决定了人物的语言。 学习反思:人物的语言反映了人物的身份、地位和性格特点。周朴园与鲁侍萍的对话中,周朴园开始的盛气凌人、漫不经心到怀疑犹豫、猜度游移,直至惊恐万分、惊慌失措;而鲁侍萍从开始的平和恬淡到闪烁其词,直到悲愤控诉,这些都完全是通过语言来表现的。这种语言特点的确是值得深入体察,反复玩味的。 (1)明确:“你姓什么,”的言外之意就是:你怎么知道的这么多,也是同样的道理。再看“你姓什么,”的言外之意就是:你怎么知道得这么清楚, (2)言外之意就是:?她现在与你已经没有什么关系了。?根本就无所谓什么亲戚。 ?她没有死,现在就立在你面前。?那次她母子被人救起了。 2、(提示:有解释说明、转移话题、声音延长、阻断停顿等作用) ?“哦。——老爷没有事了,?”转移话题 ?“你——你贵姓 ,”?声音延长 ?“不过是——他很大了,——并且他以为他母亲早就死了的。”?转移话题 ?“我——我——我只要见见我的萍儿” ? 声音延长 ?“你从前在哈尔滨包修江桥,故意叫江堤出险,——”“你现在还——”?阻断停顿 ?“我看你的性情好像没有大改,——鲁贵像是个很不老实的人。”?转移话题 ?“你叫警察杀了矿上许多工人,你还——”?阻断停顿 二、1( ( A ) 2( ( B ) 三、7(( B ) (例如:?处的台词是“你最好现在喝了它吧”,可以看出此时此刻的周朴园还带有一种劝说的态度,表情不可能是“忽然严厉地”,也不可能是“怒视”。?处是在蘩漪不想喝药而让四凤把它拿走之后,周朴园大概没有想到蘩漪会这样做,所以“忽然严厉地”才符合剧情的发展。) (答:周朴园威逼蘩漪喝药 8 9(答:周朴园逼迫蘩漪喝药,蘩漪内心非常痛苦,但仍然“忍顺”地消极反抗,“声颤”“恳求”显示了她不愿屈从于丈夫淫威的努力,但在周朴园“怒视”、强迫之下,内心的桀骜不驯使得她表现得很激烈,“忽然”叫四风拿走,“拿起又放下”。最终,面对周朴园叫周萍下跪,强忍住心头怨愤“一气喝下” 10(答:,我在家是说一不二的,你必须听我的。要不我不仅让你的孩子劝你,我还会收拾你的孩子,你会看在孩子的面上给我喝下去的。(这句潜台词含意深刻,应结合周朴园的性格特征来揣摩。) 11(答:(周朴园是一个十足的封建家长,他为了维护自己的权威身份,说一不二的地位,上下尊重的秩序,不惜用威逼手段使妻子就范,让儿子下跪。 3 哈姆雷特 2 《哈姆莱特》导学案(一) 1. 了解莎士比亚及其剧作 2. 了解《哈姆莱特》剧情 3. 掌握文中的重要字词. 4. 掌握课文紧张激烈的矛盾冲突,初步分析人物个性 【学习重点】目标3、4 【学习方法】诵读、讨论。 【知识链接】1.莎士比亚及其剧作 威廉?莎士比亚(1564—1616),文艺复兴时期英国杰出的戏剧家,诗人。1564年出生于一个富商家庭。他曾经在“文法学校”读书,后因父亲破产,中途辍学。21岁时到伦敦剧院工作,很快就登台演戏,并开始创作剧本和诗歌。他创作的大部分是诗剧,主要作品有《李尔王》《哈姆莱特》《奥赛罗》《罗密欧与朱丽叶》《威尼斯商人》等。他的作品是人文主义文学的杰出代表,在世界文学史上占有重要的地位。 他的创作生涯分为三个时期。第一个时期是1590—1600年,包括十部喜剧(如《仲夏夜之梦》《威尼斯商人》等),九部历史剧(如《理查三世》等),三部悲剧(《罗密欧与朱丽叶》作于此时)等;第二 个时期是1601—1608年,包括七部悲剧(四大悲剧《哈姆莱特》《奥赛罗》《李尔王》《麦克白》作于这个时期),四部喜剧和一些十四行诗;第三个时期是1609—1613年,包括三部喜剧(主要是传奇剧)和一 部历史剧。 2. 《哈姆莱特》背景及简介 莎士比亚是欧洲文艺复兴时期英国伟大的戏剧家和诗人。他生活在欧洲历史上封建制度日趋没落、资本主义兴起的交替时代。哈姆莱特是丹麦古代的王子。莎士比亚故意以超越时代的误差将哈姆莱特搬到伊丽莎白统治未年的英国现实中来。此时的英国,社会矛盾激化,宫廷生活挥霍浪费,社会动乱不堪,王室与资产阶级的矛盾越来越尖锐。莎士比亚借哈姆莱特之口,无情地揭露了当时社会的黑暗与不平,充分表现了他的人文主义思想。 《哈姆莱特》全剧是以哈姆莱特和克劳狄斯之间你死我活的斗争为线索展开的。主要剧情是:从老国王哈姆莱特的鬼魂那里,哈姆莱特得知了克劳狄斯阴谋杀害兄长、篡夺王位的罪行;哈姆莱特装疯,克劳狄斯怀疑哈姆莱特知道些什么,便派人刺探他;哈姆莱特用一出有“谋杀”情节,并且所用手段和克劳狄斯杀害老国王的手段相类似的戏剧,来试探克劳狄斯,以此判断鬼魂所说的话是否真实;克劳狄斯果然露出了马脚,心里对哈姆莱特十分忌惮;不明底细的王后乔特鲁德找哈姆莱特谈话,哈姆莱特误杀躲在旁边偷听的大臣波洛涅斯,克劳狄斯借机立即派他到英国去,并在公文里让英国国王处死哈姆莱特;哈姆莱特半路上回来了,克劳狄斯已经设计好毒计,让哈姆莱特和波洛涅斯的儿子雷欧提斯比剑。课文节选的是第五幕第二场(未完),主要情节是哈姆莱特和雷欧提斯比剑,这是全剧的高潮。剧情发展到这里,克劳狄斯和哈姆莱特都清楚知道了对方要置自己于死地,他们谁都不能再回避、耽搁,形势刻不容缓。老辣狠毒的克劳狄斯抢占了先机,安排了看上去万无一失的“比剑”的圈套。 【学习过程】 一( 完成下列字词检测,并解释词义。 写汉字:(guǐ)计,,,,, 拾人牙(huì),,,,, 给下列加点字注音: 戕害,,,, 蟊贼 ,,,, 繁文缛节 ,,,, 战栗,,,, (((( 枭下,,,, 藩属,,,, 棕榈,,,, 国玺,,,, ((((( 嗣位,,,, 阿谀,,,, 禀赋,,,, 词藻,,,, ((((( 吮吸,,,, 揩干,,,, 嗜杀,,,, 拘捕,,,, (((( 二( 整体感知,概述情节,回答下列问题。 1. 哈姆莱特在船上找到公文后,采取了什么措施,这表现了他什么个性, ——————————————————————————————————————————— ——————————————————————————————————————————— ——————————————————————————————————————————— 2. 哈姆莱特是怎么样捉弄奥斯理克的,这一情节并非剧情发展必须的,怎么看待这类情节的存 在, ——————————————————————————————————————————— ——————————————————————————————————————————— ——————————————————————————————————————————— 3. 哈姆莱特比剑结局如何,这种结局他是否能避免,从中可以看出他什么个性, ——————————————————————————————————————————— ——————————————————————————————————————————— ——————————————————————————————————————————— 4. 仔细阅读哈姆莱特比剑这一片段,说说几个主要人物之间的复杂关系是如何推动情节发展的。 ——————————————————————————————————————————— ——————————————————————————————————————————— ——————————————————————————————————————————— 5. 一千个读者就有一千个哈姆莱特,纵观全文情节的发展,谈谈你对哈姆莱特的看法。 ____________________________________________________________________________________________ ____________________________________________________________________________________________ _________________________________________________________________________________________ 三( 拓展阅读。 1.《关于哈姆莱特》(索天章) 《哈姆莱特》这部戏不一定是莎士比亚最伟大的作品,但是它受到最多数人的重视。几个世纪以来,关于此剧的评论不可胜数,各种解释都有,几乎不可能再有什么新的解释了。只有一点似乎还没有人仔细研究过,那就是,《哈姆莱特》这部戏虽然是与英国历史剧完全不同的伟大悲剧,它却与它们有着密切的关系,是一脉相承的。莎士比亚在写英国历史剧时,是从宣传都铎王朝的政治思想开始的。他在前几本英国历史剧里阐述了英明的君主的必要性和弑君篡位必然招致灾祸的论点。这些我们在前面已经详细谈过。莎士比亚写《亨利四世》上下集和《亨利五世》的时候,他的认识深入了一步,他的写作艺术也随之提高。他进一步发现了许多问题,其中之一便是他对于像亨利五世这样的“模范君主”开始持有保留的态度,对于他所耍的权术有所怀疑。在《裘力斯?恺撒》一剧中他同样研究这一问题,开始感觉到它并不像他原先所想象的那样简单。等到他写《哈姆莱特》的时候,虽然篡位弑君是重要的背景,他看出哈姆莱特所面临的乃是十分复杂的社会问题,简单地为他父亲报仇是没有多大用处的。莎士比亚这时开始将眼光放在整个社会上面,同时仔细分析了哈姆莱特在极端复杂的条件下的复杂的内心世界。这样,《哈姆莱特》便超出了历史剧的狭窄范围,而莎士比亚的视野更加广阔了,他的观察更加深入了,同时他的写作技巧和语言艺 术都达到了新的高度。 四:另类思想 真的有“一千个哈姆雷特”, 这是“创造性教育”的一个根据,也是多元解读的一个根据。难道所有的文本都能读出“一千个哈姆雷特”, 首先,不同类型的文本有不同的阅读过程和结果。 就文本而言,大概有文学文本、科学文本(纯科学文本、准科学文本、泛科学文本)、一般文本。 文学文本特点:(1)“文本所指”具有极大的不确定性,同样的客观对象由不同的人去描述,或者同一个人在不同环境下或不同心情下写出的客观对象也会不同,所谓情景相生就是这个道理。(2)文学作品所反映的是作者对客观世界的观察理解和感受,不同的人因为阅历、学识不同,对同一世界的反映就会不同;同一作者处在不同的历史阶段、不同环境、不同心态下写出的同一世界也不会哦相同。(3)描述的语言具有极大的灵活性,每个作者有自己的语言风格和描述方式。 文学的目的不是揭示概念,而是用文字塑造形象,因此文学文本的语言具有极大模糊性和灵活性。由此造成了文学文本的“空白点” “未定点”,于是就有了“一千个读者就有一千个哈姆雷特”。 在《项链》的教学中,以前我们对玛蒂尔德的分析通常定位在“虚荣心”上,如果有其他的认识则是不可以的。这种评判很不适合文学阅读。玛蒂尔德有许多可爱之处,怎一个“虚荣心”了得,即使是虚荣心吧,有一点虚荣心又有什么不可以呢,对于学生的评价教师应作出恰当的判断,不能用很好来一概而论。 科学文本中纯科学文本主要包括自然科学领域的教科书、学术论著,也包括自然科学与人文科学之间的一些交叉学科,入数理逻辑、教育统计学等。这类文本的特点是确定性、精确性和严密性。无论怎么样读不出“一千个哈姆雷特”。 泛科学文本主要指人文科学论著,哲学、历史、美学等,虽然有概念,但没有那么严密,因而也有空白点,也许也会读出一些“哈姆雷特”。 准科学文本,主要指科普作品。有文学性强一点的,也有严谨一点。严肃的科普作品所介绍的科学知识、科学原理应该是准确的,不应该让读者误解。 一般文本,包括政府文件、法律法规、新闻报道以及生活中的各种实用文体。这类文本在语言表述上应该是确定和惟一的。通知、启事应具有惟一性。 从读者角度对这些文本归类的话,可以概括为:(1)硬性文本,包括纯科学文本,一般文本;(2)软性文本,包括文学文本,泛科学文本;(3)中性文本,包括准科学文本。 不同类型的文本有不同的特点,因而阅读过程就不一样。不是所有的阅读都是创造的,而且不是所有的阅读都是可以创造的,不是所有的文本都能读出一千个“哈姆雷特”。 其次,即使是软性文本,也不一定能读出“一千个哈姆雷特”。 1、未必所有的形象都像“哈姆雷特”一样丰富 莎士比亚创作的戏剧多达37部,他塑造的人物形象更是数以千百计,而哈姆雷特是个性最丰富的形象之一。因为他出身在王室却又接受了人文主义教育,面对父王被杀,自己身负复仇的重任而困难重重时,他感到茫然、忧郁、失望、烦恼„„但是,为了复仇,他仍然孤军奋战,最后与敌人同归于尽。哈姆雷特自身形象的丰富性使得有不同的认知水平、情感体验、生活经历的读者必然会产生不同的独特体验。这就是所谓的“一千个读者就有一千个哈姆雷特”。 语文课本中并不是每个形象都如此丰富。如《董存瑞舍身炸暗堡》中的董存瑞就是一个为了党和人民的利益不惜牺牲自己的英雄形象;《狐狸和乌鸦》中的狐狸就是一个为了自己的利益而想方设法吹捧别人的狡猾的形象,而乌鸦则是一个喜欢听奉承话的形象„„这些人物形象,并不会引起读者的独特体验。即使是一万个读者,他们心中仍然只会有一个“哈姆雷特”。 2、形象丰富的“哈姆雷特”仍然是哈姆雷特 世界文学中的著名人物形象数不胜数。但每个形象都有他与众不同的地方:巴尔扎克笔下的葛朗台是一个贪婪吝啬、投机取巧的守财奴形象;海明威笔下的老渔夫桑提亚哥是一个永不服输,坚持与命运抗争的“硬汉”形象,鲁迅笔下的“阿Q”又是一个精神胜利的代表„„这些人物形象并不会因为有不同的读者而发生根本变化。就说哈姆雷特,绝不会有任何一位读者把他与阿Q或葛朗台等同起来。否则,哈姆雷特这个艺术形象就没有存在的价值。 因此,我们在尊重学生的独特体验时,千万不能脱离了文本的价值取向以及社会道德所宣扬的真善美。否则学生的独特体验不但不能认为是“独特”,而且要加以修正。 3、自主感悟中的“一千个哈姆雷特”都有道理吗 “一千个读者就有一千个哈姆雷特”就是接受美学的经典名言。 相对于过去那种严重扼杀学生个性和创造力的“阅读教学”,接受美学不失为一帖对症的良方,因此正在得到越来越多的语文教师的认同和响应。 但问题是:接受美学的研究对象是一般读者阅读文学作品的审美过程,而阅读教学则是学生在教师指导下的一种有目标的学习的行为,两者之间固然有不少共同点,但能不能就此画上等号,教学过程中的学生能不能等同于一般读者,一千个读者可以从一个哈姆雷特读出成千上百个哈姆雷特,但如果一个班级的学生从一篇课文中也读出了千差万别的结论,我们要不要全部肯定,按照某些研究者的意见,凡学生的“发现”都是可贵的,当然都应该予以肯定。有一个典型的个例。某班学生读朱自清《背影》,什么也没发现,就“发现”了“父亲违反交通规则”这一条,有位教授先生当即指教我说:学生能够发现“父亲违反交通规则”,也是好的嘛~ 其实,阅读的自由与阐释的可能范围一直也是接受美学关注的课题。不同的语境和读者势必带来意义阐释和阅读期待的五花八门,这种状况可能给文本带来丰富的视野,但同时也可能导致阐释的混乱不堪。因此,接受美学的代表人物伊瑟尔在肯定作品意义不确定性的同时,也在寻找意义相对的“确定性”在他看来,文本中的“空白”虽然指向文本中未曾实写出来的部分,但文本已经实写出来的部分却为读者提供了“重要的暗示或提示”。他把这个能给予读者暗示或提示召唤读者参与创造的部分叫“召唤结构”。一方面,文本的空白召唤、激发读者进行想像和填充;另一方面,“召唤结构”又暗示乃至决定着解读的自由度和意义阐释的可能范围。(如《项链》的课件) 可见,伊瑟尔也并不认为读者可以随心所欲、毫无限制地阐释文本。所谓“一千个读者就有一千个哈姆雷特”,只是不同读者从不同角度解读哈姆雷特的结果,但哈姆雷特还是哈姆雷特,不会是李尔王或奥赛罗,可惜我们在强调学生作为阅读的主体时,往往忽略了文本给予读者的暗示或提示对解读自由度的限制。这种剑走偏锋对阅读教学的损害,其实不亚于“标准答案”。 《哈姆莱特》导学案(二) 【学习目标】 1. 品味语言,把握哈姆莱特语言的动作化和性格化 2. 了解哈姆莱特的天命思想。 3. 了解《哈姆莱特》的艺术价值。 4. 对比《窦娥冤》,了解中西戏剧的不同。 【学习重点】目标1 【学习方法】诵读、讨论。 【学习过程】 一( 品味哈姆莱特的语言,举例分析动作化和性格化。 ———————————————————————————————————————————————————————————————————————————————————————————— 二( 阅读下面文字,做题。 国王 我想我的判断不会有错。你忙两人的技术我都领教过;但是后来他又有了进步,所以才规定他必须多赢几着。 雷欧提斯 这一柄太重了;换一柄给我。 哈姆莱特 这一柄我很满意。这些剑都一样长吗, 奥斯理克 是,陛下。(二人准备比剑) 国王 替我在那桌子上斟下几杯酒。要是哈姆莱特击中了第一剑或是第二剑,或者在第三次交锋的时候争得上风,让所有的城堡上一齐鸣起炮来;国王将要饮酒慰劳哈姆莱特,他还要拿一颗比丹麦四代国王戴在((( 王冠上的更贵重的珍珠丢在酒杯里。把杯子给我;鼓声一起,喇叭就接着吹响,通知外面的炮手,让炮声震彻天地,报告这一个消息,“现在国王为哈姆莱特祝饮了~”来,开始比赛吧;你们,裁判员,留心看啊。 ((((( 哈姆莱特 请了,先生。 雷欧提斯 请了,殿下。(二人比剑) 哈姆莱特 一剑 雷欧提斯 不,没有击中。 奥斯理克 中了,很明显的一剑。 雷欧提斯 好;再来 国王:且慢;拿酒来。哈姆莱特,这一颗珍珠是你的;祝你健康~把这一杯酒给他。 哈姆莱特 让我先赛完这一局;暂时把它放在一边。来。又是一剑,你怎么说, 国王:我们的孩子一定会胜利。 1. 文中加点的“国王”和“他”都是国王自指,换成“我”好不好, —————————————————————————————————————————————— 2. “这一个消息”指什么消息, ((((( —————————————————————————————————————————————— 3. 国王对哈姆莱特说他看好雷欧提斯,而对王后说“我们的孩子一定会胜利”,为什么, —————————————————————————————————————————————— 4. 从中我们可以看出国王有什么样的性格特点, —————————————————————————————————————————————— 三( 从文中找出哈姆莱特相信天命的句子,谈谈你的看法。 —————————————————————————————————————————————— 四( 《窦娥冤》和《哈姆莱特》是中外戏剧的代表作,试对比二者的不同之处。 点播:从人物性格、身份、悲剧原因去对比。 ——————————————————————————————————————————— 五( 读哈姆莱特经典的独白,回答问题。 生存还是毁灭,这是一个值得考虑的问题;默然忍受命运的暴虐的毒箭,或是挺身反抗人世的无涯的苦难,在奋斗中结束了一切,这两种行为,哪一种是更勇敢的?死了;睡着了;什么都完了;要是在这一种睡眠(((( 之中,我们心头的创痛,以及其他无数血肉之躯所不能避免的打击,都可以从此消失,那正是我们求之不得的结局.死了;睡着了;睡着了也许还会做梦;嗯,阻碍就在这儿:因为当我们摆脱了这一具腐朽的皮囊以后,在那死的睡眠里,究竟将要做些什么梦,那不能不使我们踌躇顾绿.人们甘心久困于患难之中,也就是为了这个缘故;谁愿意忍受人世的鞭挞和讥嘲,压迫者的凌辱,傲慢者的冷眼,被轻蔑的爱情的惨痛,法律的迁延,官吏的横暴,和微贱者费尽辛勤所换来的鄙视,要是他只要用一柄小小的刀子,就可以清算他自己的一生? 谁愿意负着这样的重担,在繁劳的生命的迫压下呻吟流汗,若不是因为惧怕不可知的死后,那从来不曾有一个人回来过的神秘之国,是它迷惑了我们的意志,使我们宁愿忍受目前的折磨,不敢向我们所不知道的痛苦飞去? 这样理智使我们全变成了懦夫,决心的炽热的光彩,被审慎的思维盖上了一层灰色,伟大的事业在这一种考虑之下,也会逆流而退,失去了行动的意义. 1. 选文中“两种行为”分别指什么, —————————————————————————————————————————— 2. 下列不属于“顾虑重重”范围的一句是: A死了;睡着了;睡着了也许还会做梦 B在那死的睡眠里,究竟将要做些什么梦 C若不是因为惧怕不可知的死后,那从来不曾有一个人回来过的神秘之国, D伟大的事业在这一种考虑之下,也会逆流而退,失去了行动的意义. 3对这段独白理解正确的是: A哈姆莱特为“生存和毁灭”所困,渴求抹平创伤,平静死去。 B生动地写出哈姆莱特内心的复杂,但是他还是走出了顾虑重重。 C(无论是生存还是毁灭,哈姆莱特都未走出那顾虑重重。 D这只是独白而已,没什么深刻性。 六( 拓展阅读 《哈姆莱特》情节的魅力(高文斌) 亚里士多德认为,在戏剧的情节、性格、言语、思想、形象和歌曲等六大要素中,“最重要的是情节,即事件安排”。这是两千多年前产生的欧洲传统的“情节中心”的戏剧观,与初步发展的古代希腊戏剧是相适应的。进入近代社会,由于人的价值和尊严的发现,人们对戏剧中的人物投去了更多的关注,“情节中心”的戏剧观自然而然地发生了变化,艺术应以表现人和人的思想感情为中心的“性格中心”说越来越有影响。然而,即使是“性格中心”说也承认,情节对于戏剧是至关重要的,它是戏剧的基础,是其他要素赖以生存发展的土壤,因此一部好的戏剧需要一个好的情节,没有好的情节戏剧很难在观众心中扎下根。莎士比亚在创造情节结构方面是杰出的,他的每部戏剧几乎都有引人入胜的情节故事。19世纪60年代,恩格斯在谈悲剧的一封信中高度评价莎剧的情节特征,他写道,“德国戏剧具有较大的思想深度和意识到的历史内容,同莎士比亚剧作的情节生动性和丰富性的完美融合”,那将是“戏剧的未来”。《哈姆莱特》作为莎剧的典范,也是“情节生动性和丰富性”的典范,正是生动丰富的情节与典型的人物性格,进步的思想倾向,优美的艺术语言的有机融合,相得益彰,使《哈姆莱特》成为世界戏剧史上的一部伟大的经典。 那么《哈姆莱特》情节的生动性和丰富性是如何体现的,莎士比亚又是怎样设计安排《哈姆莱特》情节的呢,设置错综复杂的情节线索,扩大悲剧反映社会生活的广度和深度是《哈姆莱特》安排情节的突出 特征。 莎士比亚创作的前前后后,欧洲一直流行着“情节整一”的戏剧原则。这个原则是亚里士多德在总结古希腊戏剧经验的基础上提出的,简单地说,就是要求一部戏剧只能有一条情节线索,而且要完整,长短适度。亚里士多德称之为只模仿“一个完整的行动”。“情节整一”的原则支配着18世纪之前的欧洲戏剧创作,并且成为古典主义法规“三一律”的一部分。莎士比亚强调戏剧模仿自然,创作从实际出发,走上了与“情节整一”完全相悖的道路 。他的每部戏剧都有两条或两条以上的情节线索,每条线索都包含着一 定的社会内容,多角度,多层面地展现了他所生活的时代和社会。 《哈姆莱特》设置了三条情节线索,以哈姆莱特为父复仇为主线,以雷欧提斯为父复仇及挪威王子福丁布拉斯为父报仇为辅助线索,三条线索对比交错,互相映衬,构成了戏剧的主要情节。与一条线索的情节相比,这个悲剧故事不仅反映了更广阔的生活,展示了从丹麦宫廷到贵族家庭,从守望的城堡到凄惨的 墓地等生活的画面,而且包容了更丰富的内容。 悲剧的故事情节主要有三个方面的内容: 第一,哈姆莱特与外部世界的冲突:父王被害,母亲改嫁,叔叔篡位,廷臣效忠窃国者,童年朋友背叛,意中人被利用,王子装疯导演“戏中戏”,放弃复仇良机,误杀波洛涅斯,争取母亲,国王两次借 刀杀人,哈姆莱特手刃仇人,中毒身亡。 第二,哈姆莱特的内心矛盾:家庭变故后的苦闷,把复仇和“改造时代”相连,精神危机引发人文主义理想的动摇,第一次自责和自我激励,生与死的思考和批判意识的增强,放弃复仇良机的矛盾和第二 次自责,第三次自责和坚定斗志,宿命论和虚无主义思想的发展。 第三,展示气氛、环境和背景:鬼魂不祥地出现,宫廷内部混乱不堪,丹麦强邻剑拔弩张,关心国家的青年忧心忡忡,封建宗法家庭的面貌,天真的姑娘成为无辜牺牲品,小人物的不满,朝廷新老贵人的嘴 脸。 众所周知,莎士比亚的文学观念是现实主义的,就在这部戏剧中,他还强调演员是“时代的缩影”,“演戏都是为了给自然照镜子”。因此我们完全可以认为,悲剧的这些故事就是全面地给莎士比亚时代“照镜子”,广泛地反映那个时代的社会历史生活。通过这面镜子,我们既看到了封建社会向资本主义社会转型时期新旧力量的矛盾冲突,以哈姆莱特为代表的新兴力量向以克劳狄斯为代表的腐朽势力的冲击、斗争,又看到了新兴力量在思想政治上还不成熟,哈姆莱特彻底战胜克劳狄斯的时代还未到来,同时,我们也看 到了英国在伊丽莎白统治后期矛盾重重、危机四伏、社会动荡的时代特征。 总之,《哈姆莱特》反映的社会历史内容如此丰富,是“情节整一”的戏剧无法胜任的。 冲突是戏剧的基础,也是戏剧情节的基础。人们常说“没有冲突就没有戏剧”,同样,我们也可以说,没有冲突就没有好的戏剧情节。这是一个戏剧常识,也是一个戏剧的重要原则。莎士比亚历来重视这一原则。他的戏剧,特别是悲剧,总是贯穿着尖锐激烈的戏剧冲突,斗争针锋相对,场面惊心动魄,使情节充满了戏剧性。在这方面《哈姆莱特》是十分成功的。它不仅安排了三条交错对比的线索,编织了生动丰富的悲剧情节,而且设计了两个尖锐激烈、贯穿全剧的戏剧冲突,在生死较量和痛苦抉择中,进一步增 强了戏剧情节的生动性和丰富性。 《哈姆莱特》的第一个冲突是哈姆莱特与克劳狄斯的矛盾冲突。这不只是与杀父淫母夺位者的冲突,而且是一场新旧力量的较量,是以哈姆莱特为代表的人文主义新人与克劳狄斯为首的封建腐朽势力的殊死的搏斗,是你死我活的斗争。第二个冲突是哈姆莱特内心的矛盾冲突,从表面上看,这是人性、人生、生死、道德等哲理问题的思考,是何时复仇,怎样复仇的焦虑,实际上它是哈姆莱特的人文主义理想的动摇和坚守,是新旧思想的抉择,同样是无法调和的。这两个冲突相互交织构成了悲剧情节的主线,虽然发展缓慢,渐次展开,但是冲突的过程环环紧扣,因此前后照应,紧密联系,一步一步地走向高潮,走向结局, 而悲剧的情节也就波澜起伏,生动丰富,始终充满了巨大的艺术魅力。 《哈姆莱特》的悲剧情节随着戏剧冲突的展开大致经历了三个阶段: 第一幕是悲剧的开端。这是一个极为精彩的开端,脍炙人口。在城堡若隐若现的鬼魂,不仅给悲剧营造了神秘、阴森、紧张的气氛,交待了背景和人物关系,而且揭露了克劳狄斯的“可怕罪恶”,既使哈姆莱特与克劳狄斯的矛盾骤然升级,又让哈姆莱特的忧郁更加沉重。同时,矛盾冲突的升级也制造了一个贯穿全剧的悬念,忧郁的王子能够与奸诈的国王抗衡吗,他会以怎样的方式完成命运交给他的“双重使命”呢,这个悬念成为后来冲突展开、情节发展的基础,引发了观众的强烈期待。难怪柯勒律治对鬼魂的戏那 么赞不绝口。 在第二幕和第三幕的一、二场,随着冲突双方斗争和哈姆莱特内心矛盾的发展,悲剧情节有了戏剧性发展。克劳狄斯做贼心虚,急于弄清对手的虚实,哈姆莱特小心谨慎,需要证实仇人的罪行,矛盾双方在试探和反试探的行动中开始了真正的碰撞。最初的碰撞还是遮遮掩掩的,“戏中戏”使冲突由暗到明,由隐蔽到公开。克劳狄斯摸清了哈姆莱特的底细,但也暴露了自己,由主动变为被动。哈姆莱特印证了克劳狄斯的罪恶,本可以变被动为主动了。可是他正在精神危机中挣扎,在思辨中焦虑,似乎还无力投入行动。这段情节非常有戏,虚虚实实,真真假假,最后终于剑拔弩张,戏剧的节奏也由慢而快,急速地向高 潮发展。 从第三幕第三场到结尾,是悲剧的第三部分,也是悲剧情节的高潮和结局。矛盾冲突的激化使戏剧情节出现了两次起伏。一次是不杀克劳狄斯,误杀波洛涅斯;一次是偷换秘信,死里逃生。“不杀”和“误杀”使克劳狄斯得以处置哈姆莱特,哈姆莱特又处下风。绝处逢生,潜回丹麦,让哈姆莱特多少又夺回了一点主动。与此同时,斗争形势的变化反过来又进一步激化了矛盾冲突。冲突双方的决战在即,克劳狄斯已经设下圈套,而哈姆莱特内心冲突更加尖锐,发挥才智、有所作为的渴望和人生如梦、成败靠天的虚无感,使他毫无准备,听天由命地钻入了敌人的圈套。击剑、服毒、刺杀和四人死亡,是悲剧的高潮,也是结局。这个结局是双方较量的结果,既然哈姆莱特还不具备彻底战胜对手的力量,那么同归于尽就是最好的结局。这个结局也是人物性格的必然,至死还没有摆脱内心矛盾的哈姆莱特,恐怕只能有这样的命运了。 在这一部分,尖锐的冲突,复杂的性格,带动生动丰富的情节,几乎达到了水乳交融。 《哈姆莱特》答案 答案(一) 一( 诡 慧 Qiāng máo ruì lì Xiāo fān zōnglǜ xī Sī ěr yú bǐng zǎo Shǎn kǎi shì bǔ 二(1.另写一通公文,代替原有公文,要英王处死送信人。 机智敏感 2.讽刺他是水苍蝇 丰富戏剧内容 3.死亡 不能 草率 冒失 4.哈姆莱特自信 国王鼓动 王后关心 5.言之有理即可 答案(二) 一((略) 、 二。1.不好 用“他”和“国王”而不用“我”,说明国王设诡计而心虚。 2.哈姆莱特获胜 3.欺骗王后 4.阴险狡诈 三((略) 四(窦娥是平民,悲剧原因是外部因素 汉姆莱特是贵族,悲剧原因是性格 五(1.生存和毁灭 2 A 3 C 4 解读时间(作文) 1 解读时间 学习横向展开议论 导学案 一、教学目标 1(培养学生对客观事物、客观事理的观察能力、分析能力。侧重学习横向展开议论的写法。 2(引导学生关注时间,珍惜时间,了解时间对人生的意义,以便更科学有效地利用时间。 二、重点与难点 培养学生对客观事物、客观事理的观察能力、分析能力、判断能力,能对中心论点进行科学合理的切分,形成分论点的能力。 通过训练使学生能灵活运用横向展开的写法写出有一定独创性的、较有深度的议论文。 三、教学过程 1(感受认知时间 (1)时间是什么呢,古今中外有许多人对时间作过精辟的阐述,下面来让我们一起来读读课本,看一看都有哪些阐述,(学生读课本,感受他人对时间的认识。) (2)同学们还能说出哪些有关时间的诗文、名言警句, 明日复明日,明日何其多,我生待明日,万事成蹉跎。世人若被明日累,春去秋来老将至。朝看水东流,暮看日西坠。百年明日能几何,请君听我明日歌。(文嘉《明日歌》) 三更灯火五更鸡,正是男儿读书时。黑发不知勤学早,白首方悔读书迟。(颜真卿《劝学》) 不饱食以终日,不弃功于寸阴。 少壮不努力,老大徒伤悲。 已去之浪不回流,已去之时不再来(意大利) 无人能唤回已去的时辰(英国) (3)读了这些诗文、名言警句后,同学们对时间又有怎样的认识呢,(让学生各抒己见,培养学生口头表达交流能力;并借此引导学生归纳时间的性质、特点,加深对时间认识,了解时间对人生的意义,养成对待时间,对待人生的正确态度。) 学生发言,师板书学生对时间的认识 (短暂、宝贵、稍纵即逝、一去不返) 教师小结:时间的脚步是无声的。冬去春来,天回地转,稍不留意,岁月就会从你身边悄悄溜走。它不会给延误时间的人以任何宽恕,也不因任何人的苦苦哀求而偶一回顾。它能使红花萎谢、绿叶凋零;会让红颜变成白发,让童稚变成老朽。时间是无情的,又是有情的。对于珍惜时间的人,它却馈赠以无穷的智慧和财富。对于风华正茂的青少年来说,青春的脚步如行云流水,青春的岁月容不得半点浪费。莫要为昨天叹息,当你为虚掷年华而叹息的时候,年华又在叹息中虚掷。珍惜时间,请从眼前开始,从今天开始。今天是未来的地基,未来是今天的延续。走向美好的未来,必须争分夺秒地抓紧今天。向往辉煌的未来,必须更加热爱,珍惜宝贵的时间。谨记:我们不是时间的富翁,我们要做时间的主人。 2(写法指导 这次的写作训练是“解读时间,学习横向展开议论”,实际上是要求对学生进行发散性思维的训练。 发散性思维训练的特点是由一个中心话题引发出多种不同的思考点。在这种训练中,应当注意把握两点:一是思考的辐射面要宽,各辐射点之间的关系是并列的,但又要有一定的跳跃性,每一个辐射点都应该是可以独立展开论述的分话题;二是思考要有一定的深刻性。为了帮助学生尽快理解和掌握这种思维方法,教师可以用生活中常见的事物作为中心话题,引导学生进行练习。 1)横向展开议论主要表现为并列式结构,即在论证思路中,对中心论点进行条分缕析,分解出几个分论点,共同来阐述文章的中心论点,这样就是并列式。 各个分论点平行列举,各个阐述。并列结构的外部语言标志有“首先”、“其次”、“再次”,或“一方面”、“另一方面”,或“其一”、“其二”等 。几个并列的分论点的内容不可交叉,不可重复,不可包容,不可矛盾。 并列式结构模式: 引论(提出中心论点、分论点) 论证分论点1:论据,分析论证 并 模式: 本论 论证分论点2:论据,分析论证 论证分论点3: 论据,分析论证 列 结论(照应全文) 参考示例: 2)横向展开议论的关键是提取分论点,那么如何提出分论点呢,主要方法有: (1)追问类属法——即从“是什么”的角度展开。针对话题本身(或话题关键词),运用横向思维, 根据它不同的内涵或外延进行多角度分析 。常用的表述方式是:“某某,是一种„„” 示例: 以“肩膀”为中心,进行分解: ?肩膀是亲情的牵挂(肩膀装载着幸福); ?肩膀是友情的寄托(肩膀承载着快乐); (2)追问对象法——即从“关涉谁”的角度展开。就是看话题的两端可涉及到哪些对象(人,事,物)。 例:以“纪念”为中心论点,分解分论点: ?纪念靖节先生(让我懂得了在浮躁纷扰中独守心中一方乐园和净土)。 ?纪念苏轼(让我懂得了在困难逆境中应保持有奋勇向前的心)。 (3)追问原因法——即从“为什么”的角度展开。就是把中心论点作为结论去追溯这个结论产生的条 件和原因。主要是回答原因和目的方面的问题。常用的分论点表述方式是:“因为„„,所以„„” 示例:以“答案是丰富多彩的”为中心论点,进行分解: ?世界千变万化,事物是复杂的,因此答案是丰富多彩的。 ?看问题的方法、看问题角度不同,因此答案是丰富多彩的。 训练示例: 以“选择诚信 ”为中心,进行分解。 ?选择诚信,因为它比美貌来得可靠; ?选择诚信,因为它比机敏来得憨实; ?选择诚信,因为它比金钱更具内蕴; (4)追问办法法——即从“怎么办”的角度展开。主要是追寻解决或达到的方法和途径,分论点常用 的表达方式是“要„„就应该„„” 示例:以“学会欣赏别人”为中心,进行分解: ?学会欣赏别人,要求自己要有坦荡的胸怀。 ?学会欣赏别人,要努力发掘别人身上的闪光点。 (5)追问结果法——即从“会怎样”的角度展开。 主要是回答结果和效能方面的问题(意义、作用、 价值、影响),常用表达方式,做到„„就会„„。 示例 :以“热爱生活”为中心,进行分解: ?热爱生活,就会乐于为生活付出; ?热爱生活,才会有奋斗的力量; ?热爱生活,才会创造美好的生活。 3)课堂训练:请以“直面挫折”为话题,运用以上方法提出分论点。 示例:(1)直面挫折是一种深沉认知自我的心态。 (2)直面挫折是一种豁达的人生境界 。 (3)直面挫折,翻过肢体表达的屏障。 (4)直面挫折,冲破心灵沟通的隔膜。 (5)个人直面挫折,才能不断进取。 4)教师小结:(1)横向展开的几种角度(方法) ?追问类属法——即从“是什么”的角度展开。 ?追问原因法——即从“为什么”的角度展开。 ?追问办法法——即从“怎么办”的角度展开。 (2)拟写分论点的要求是:分而有范,分而有理,分而有序: ?分而有范:几个分论点应该按同一标准(同一角度)回答同一问题。(“扣得住”) ?分而有理:分论点之间界限清楚,不互相交叉重叠,互不包容。(“分得开”) ?分而有序:分论点的排列的先后顺序要合乎逻辑,合乎情理。(“排得顺”) 5)阅读这篇范文,把握横向展开的写法。 干事业需要激情 激情是吹动船帆的风,没有风船就不能行驶;激情是火箭的推进剂,没有推进剂,火箭就难以飞向蓝天。生活告诉我们,灵感可以催生不朽的艺术,激情能够创造不凡的业绩;缺乏激情,疲沓涣散,很可能一事无成。因此,我们对待工作必须始终保持高昂的激情,有了激情,工作才能轰轰烈烈地进行。 对待工作的激情不是心血来潮,兴之所至,而是一种觉悟、追求和境界。在实际工作中,有许多胸怀大志、开拓进取、顽强拼搏、埋头苦干的人,他们始终保持高昂的工作热情和旺盛的革命干劲,因而工作成效明显,事业日新月异。 高昂的激情来自崇高的理想。没有理想,人就会失魂落魄。一块手表可能有最精致的指针,可能镶嵌了最昂贵的宝石,然而如果缺少了发条,它仍然一无用处。同样,一个人无论是学富五车,还是健壮高大,如果对工作毫无激情,甚至连热情都不足,生命就会黯然失色。昆虫学家法布尔正是因为有献身昆虫学的崇高理想,正是因为对事业有着火热的激情和满腔的热忱,才创造出了骄人的成就。 高昂的激情来自强烈的责任感。责任感是对党和人民事业的忠诚和热情。一个具有高度责任感的人,会把工作看成追求和奉献,满怀激情地投入工作;一个丧失责任感的人,会把工作当做一种负担,自然就会失去工作的乐趣。田家英同志曾经写过一首诗:“十年京兆一书生,爱书爱字不爱名。一饭膏粱颇不薄,惭愧万家百姓心。”这就是一种高度责任感的写照。由责任感激发出来的力量是巨大的。有了强烈的责任感,才会有奋发有为的精神状态,开拓创新,干好事业。 高昂的激情来自自强不息的追求。“天行健,君子以自强不息。”自强不息是激情不断迸发的动力,是推动事业发展的加速器。我们所处的时代是一个强手如林、竞争激烈的时代,是一个日新月异、你追我赶的时代。我们应当保持清醒的头脑,与时俱进,自强不息,克服知足常乐的思想惰性,向着更快更高更强 的目标前进,不断研究新情况,解决新问题,开辟新境界。 让我们始终保持奋发向上的精神状态,把高昂的激情投入到工作中去,才能用勤劳的双手创造幸福的生活和美好的未来。 点评:这是一篇结构规范的议论文。本论部分又设置了三个分论点“高昂的激情来自崇高的理想”、“高昂的激情来自强烈的责任感”、“高昂的激情来自自强不息的追求”,三个分论点共同论证了中心论点,呈现出并列式结构。整篇文章结构严密,层次分明,脉络清楚。 6)教师强调横向展开议论应注意的要点:(屏幕显示以下文字) ?考虑轻重关系、主次关系、先后关系、时间关系。 ?形式要一目了然,即每个分论点应在段首作中心句。 ?多角度地观察、分析、认识事物。 ?分论点角度要统一、不能交叉包容。 3(写作实践 (1)把有关时间的古诗、名言警句印发给学生。 (2)阅读这些材料,自选角度,自拟标题,运用自己刚刚归纳的写法,写一篇不少于800字的议论文,谈谈自己对时间的理解。 珍惜生命》) (参考范文:教材精析精练《珍惜时间 三、评价与反思 “时间”这个对象本身很玄妙,难以捉摸。通过让学生仔细阅读“话题探讨”部分和“写法借鉴”部分,将关于“时间”的一些说法加以概括,提醒学生注意那些引起他们的兴趣、引发了他们进一步思考的说法,然后在班上交流、讨论,这样一来,既加深了学生对时间的认识,又活跃了学生的思路,能解决“写什么”的问题。 “写法指导”这一环节,通过让学生阅读写法为横向展开议论的文章,主要归纳总结横向展开议论的写作规律,能较好的解决“怎么写”的问题。 5 柳永词两首 2 《柳永词两首》导学案,一, 望海潮 【学习目标】 1、 知识和能力:了解柳永词的特点,感受评赏词的意境、词的情韵,学习写景、叙事、抒情相结合的手法; 2、 过程和方法:反复吟咏,感受两首词的节奏美、音韵美;感受柳永慢词的意境美 3、 情感态度和价值观:领会作者的真挚感情和对杭州西湖的热爱之情。 【学习重、难点】 领悟作者对杭州西湖的热爱之情; 感受词的意境,情韵;学习将写景、叙事、抒情相结合的写法。 【学法指导】诵读法,鉴赏法,探究法 【知识链接】 1作者简介:柳永(约987,约,,,,),北宋词人。原名三变,字景庄。后改为名永,字耆卿,因排行第七,又称柳七。崇安(今属福建)人。景佑进士。官屯田员外郎。世称柳屯田。他一生在仕途上抑郁不得志,独以词著称于世。他为人放荡不羁,终身潦倒。其词多描绘城市生活的繁华,歌妓悲欢、愿望及男女恋情,尤长于抒写羁旅行役之情。创作慢词独多。铺叙刻画,情景交融,语言通俗,音律谐婉,在当时流传很广,对宋词的发展有一定影响。《雨霖铃》、《八声甘州》、《望海潮》等颇有名。有《乐章集》。 2 历来认为,这首《望海潮》是作者献给两浙转运使孙何的。孙何是柳永的友人,当时驻节杭州。文中表现杭州繁华富庶的题旨,是为了称颂祝愿。来日升迁到朝廷做官,虽然杭州城无法带去,但把这西湖美景绘成图画,可带回京城去欣赏。 相传,后来金主完颜亮听唱“三秋桂子,十里荷花”以后,便羡慕钱塘的繁华,从而更加强了他侵吞南宋的野心。为此,宋人谢驿还写了一首诗:“莫把杭州曲子讴,荷花十里桂三秋。岂知草木无情物,牵动长江万里愁。”(见罗大经《鹤 林玉露》)说金主亮因受一首词的影响而萌发南侵之心,原不可信。但是,产生这一传说,却可以印证这首词的艺术感染力是很强的。 3知识概述 词始于唐代,鼎盛于宋代。它原是配乐歌唱的一种诗体,句的长短随歌调(词牌)而改变,句子长短不一,所以有“长短句”之称;又有“诗余”“倚声”“填词”之谓。词依字数的多少分为小令、中调、长调,58字以内称小令;59,90字为中调;91字以上称长调。一般分上下阕。从风格看,宋词大致可分为豪放与婉约两派。明代张綖说:“婉约者欲其词调蕴藉,豪放者欲其气象恢弘。”本周学习婉约派最有影响的两位词人的作品。 【学习过程】 一 学生在熟读的基础上背诵全词,给下列加点字注音,A级, 参差, ,堤沙, ,天堑, , 珠玑, , 罗绮, ,叠巘, , 嬉嬉, ,萧鼓, , 二 名句默写,A级, 东南形胜,,,,,,,,,,。 ,,,,,,,,,参差十万人家。 ,,,,,,,,,户盈罗绮,竞豪奢。 重湖叠巘清嘉,,,,,,,,,,。 千骑拥高牙,,,,,,,,,,,,,,,,,,,。 ,,,,,,,,归去凤池夸。 三 鉴赏《望海潮》,B级, ? 《望海潮》一词描绘了怎样的一种生活景象, ? 作者是从哪些方面描写杭州的繁荣和美丽的,抒发了他怎样的感情, ? 结尾的几句写了什么内容呢? 最后两句是写给谁的,从哪里可以看出, ? 艺术手法 四 背诵全词,A级, 五 总结 这首词上阕极力赞美杭州自然风光和都市的繁华;下阕极力描写杭州黎民百姓安居乐业的生活景象,歌颂了杭州山水的美丽风光,赞美了杭州人民安定欢乐的生活,反映了北宋结束五代分裂割据局面以后,所显现的繁荣太平景象。杭州的湖光山色令人神往,读了柳永的这首词,领略了作者高超的写作技巧,美丽的西湖更加令人心旷神怡,相信你一定会有机会领略那里美好的风光的。 五 当堂检测、默写全词,A级, 课外拓展 赏析《八声甘州》 ?柳永 对潇潇暮雨洒江天,一番洗清秋。渐霜风凄紧,关河冷落,残照当楼。是处红衰翠减,苒苒物华休。惟有长江水,无语东流。 不忍登高临远,望故乡渺邈,归思难收。叹年来踪迹,何事苦淹留,想佳人妆楼望,误几回、天际识归舟。争知我,倚阑干处,正恁闲愁。 【赏析】 这是一首抒写游子思乡感情的作品。词里把人生漂泊的悲愁,思乡怀人的凄苦深挚地表现出来。开篇以江天为背景,暮雨潇潇,暝色漠漠,一个“洒”字,让人联想起利落而过的风声、雨声和涛声。正清秋时节,秋意萧疏,一番风雨洗罢,凸显凄清悲凉的气氛。“洗”字鲜明而精当,让人回味起韩愈的“长安雨洗新秋出”。 紧接着,作者笔下洒脱而苍劲地推出一片撩人心扉的秋色:“渐霜风凄紧,关河冷落,残照当楼”,从听觉、视觉,远景、近景的交叠中传递着开阔博大的空间感受,雄浑刚健的音乐震撼。苏轼曾为此赞赏说,“于诗句不减唐人高处”。苍凉的秋,几乎泯灭了一切生机,处处群芳凋零,落叶纷飞,美好的景物在漫无声息中无奈地消亡,连浩浩长江,也哽咽失语,默默东流。作者在字里行间微妙地渗透着游子登高临远的悲戚与感伤。 词的下半片真切地抒发作者曲折的心情,写来跌宕多姿,别有韵致。“不忍”二字,逆势而起,表现内心世界缠绕不解的矛盾。远望故乡,回家的念头总要浮上心头,难以抑制,却愈来愈加强烈。自叹近年来漂泊江湖,百无聊赖,空有归思,有家难回。其间的苦楚和怨艾,欲说还休。作者巧妙地用“何事若淹留”,把多少辛酸、不平和痛切隐在心底。这一笔委婉地刻画了内在感情,读来有缠绵, 而“叹”,由“叹”而“想”,词人层次分明地展示一幕幕情有深悲。由“望” 景,想到佳人对自己的期盼,由于主客的转换,场景的拓展,韵味更为醇厚。传神的家人妆楼颙望,这一幕油然让人忆起“过尽千帆皆不是”的惆怅和失落。“误几回”三字,直白中显精到,率真而寓深沉,亦是妙笔。结语回到作者的深情诉说,那满腔的乡愁和眷念,满腹的忧伤和悲凉,谁能体味又该如何倾诉呢, 这首词状写景物,不尚藻饰,白描中风神俱现,得大巧之朴,可谓高妙;抒写情感,流连顿挫,反复间淋漓尽致,兼动静之美,可见高超。如果配以谐婉的音律唱歌,必将曼妙缠绵,令人悱恻,使人陶醉。 《雨霖铃》导学案 教学目标 1. 感受词中浓浓的别离之情。 2. 鉴赏名句,理解善于铺叙、融情与景、虚实相济的艺术特点。 3.分析意境,提高学生理解和鉴赏词的能力。 教学重点、难点 把握意象,探究意境,体味情感 教学方法:感悟法、鉴赏法、讨论法 【学习过程】 一 学生一起背诵《雨霖铃》 二 给下列词注音 骤雨, ,暮霭, ,凝噎, , (((( 三 名句默写 寒蝉凄切,,,,,,,,,,骤雨初歇。 ,,,,,,,,,,,,,,,,,。念去去, 千里烟波,,,,,,,,,。 多情自古伤离别,,,,,,,,,,,,,,,,,,。 今宵酒醒何处? ,,,,,,,,,。 四 鉴赏《雨霖铃》 ? 本首词分为上、下两阙,上阙主要写,,,,下阙主要写,,,。 ? 本词的基调是什么, ? 本词按什么顺序来写的,那么离别可以分为几个阶段 ? 分析“寒蝉凄切,对长亭晚,骤雨初歇。” 主要渲染了一种什么样的气氛, ? “都门帐饮无绪。留恋处,兰舟催发。执手相看泪眼,竟无语零噎。”体会这句词的感情。 ?“念去去千里烟波,暮霭沉沉楚天阔” 这句话在上阙的结尾表达怎样的感情,放在此处有何作用, ?.“多情自古伤离别,更那堪冷落清秋节。”在这两句中,有三个字点明了这 首词的主旨,是哪三字, ?:“此去经年,应是良辰好景虚设。便纵有千种风情,更与何人说~”这是作者设想的别后孤苦无聊的生活情景。请问词人这里为什么说“虚设”, 五 小结 情景交融的艺术手法:这首词以冷落的秋景作衬托,精心刻画难分难舍的惜别场景,进而想象别后的离愁别恨,情景相生。虚实结合的艺术手法:上阙重点在记别,下阙重点在述怀,前后呼应,浑然一体。(词作在表现离人分别时,通过离别场面的描写、铺叙,让读者深切感悟离情)。 从这两首词中,我们体会了诗人柳永婉约的创作风格,体会了词的特色。由此得知:词,声韵谐婉,意境优美,是我国抒情文学的最高形式之一。大家多去品味词,增强自身美的感悟力。 柳永的词,声韵谐婉,意境优美,清代词人冯煦曾这样评价他:“耆卿词曲处能直,密处能疏,傲处能平,状难状之景,达难达之情,而出之以自然,自是北宋巨手。” 希望大家学会声情并茂地品读这些优美的文学作品,你定会受益匪浅。 六 背写这首词 七当堂检测: ? 词中哪句话最能体现这个内容呢, ? 本词有哪些意象的词,这些意象的词起到怎样的作用, ? 默写本词。 课外拓展 赏析《雨霖铃》 词的起头三句点明时间、地点、景物,事件是与自己心爱的人饯别。晚上,阵雨才停,知了发出凄切的鸣叫,长亭送别,叫人如何能够忍受这离别的痛苦~这蝉鸣助添悲凉,而一开始即道出“凄切”,为这首词定了调子。这一层展开了一个凄凉的氛围。“都门”两句,极写饯别时的心情,委婉曲折。两情依依,难舍难分之际,客船却不断催促。心理矛盾,欲饮无绪,欲留不得。由此可窥见留恋之情深。“执手”两句,再加深涂抹,在“执手”、“相看”、“无语”中更使人伤心失魄。这一层极写留恋之情。以上两层极尽回环、顿挫、吞吐之能事,足以使人为之呜咽。“念去去”以后,则大气包举,一泻千里,似江流出峡,直驰平川,词亦直抒胸怀。以“念”这一领字带起,表明是设想别后的道路辽远,“千里烟波,暮霭沉沉楚天阔”,浩渺的烟波,沉沉的暮霭,辽阔的天空,全是写景,实际上全含的是情,衬托出旅人前途茫茫,情人相见无期,景无边而情无限。换头以情起,叹息从古到今离别之可哀,“伤离别”点明这首词的主旨。“更那堪冷落清秋节”句又将推进一层,更何况我正在冷落清秋的时节呢,这是多么难以忍受啊~这是把江淹《别赋》中“黯然销魂者唯别而已矣”和宋玉悲秋的情思两者结合起来,提炼出这两句。把古人这种感受融化在自己的词句中,更赋予以新的意义。“今宵”二句,又进一步推想别后的凄凉,然而景物清丽真切,真象别者酒醒后在船中之所见。这一句妙在景中有情。“昔我往矣,杨柳依依。 这首词写将别、临别以及别后的种种设想,以白描的手法铺叙景物,倾吐心情,层次分明,语意明确,绝少掩饰假借之处。尤其是把别后的情景描写得比真的还真,又以景视之,使人不觉得是虚构的,足见柳永的艺术手法之高妙。所以有人称其“微妙则耐思,而景中有情。„„‘杨柳岸晓风残月’,所以脍炙人口也。”。”(谢章铤《赌棋山庄词话》)又有人认为“‘千里烟波’,惜别之情已骋;‘千种风情’,相期之愿又赊。真所谓善传神者。”(李攀龙《草堂诗余隽》)这都道出这首词的妙处的。但我觉得刘熙载在《艺概》中的“点染”之说,更是值得称述的。 柳永词两首 《望海潮》 cēncīdīshātiānqiànzhūjīluïqǐdiã一参差 堤沙 天堑 珠玑 罗绮 叠yǎnxīxīxiāogǔ 巘 嬉嬉 萧鼓 二 三吴都会,钱塘自古繁华。 烟柳画桥,风帘翠幕, 市列珠玑, 有三秋桂子,十里荷花。 乘醉听萧鼓,吟赏烟霞。 异日图将好景, 三 ?太平、富庶、安定、祥和的都市生活图景; ?“东南形胜”三句从地理条件、悠久历史来说的,“烟柳画桥”六句是从风景美丽和人口众多来说,“市列珠玑”三句是从人们生活的城市的商业发展来看的。(地理位置、历史传统、自然景观、市井面貌、百姓生活); 抒发了对杭州风物的惊叹、赞美、艳羡。对两浙转运使孙何的奉承。 ?结尾处词人达官贵人到此游乐的场景,以达官贵人的不思离去,烘托出西湖之美。成群的马队簇拥着高高的牙旗,缓缓而来,一派煊赫气势。诗人笔致洒落,音调雄浑,仿佛令人看到一位威武而又风流的地方长官,饮酒赏乐,啸傲于山水之间。 词中“图将”是指把杭州美景画出来,“凤池”指朝廷。这样一来,此句的意思就十分明显了。这里其实暗含对长官日后飞黄腾达、不断高升的美好祝愿。它是作者纯粹的奉承之辞。 ?本文是慢词,所以在艺术手法上与小令有所不同。本词重在用铺叙的手法,例如“烟柳画桥,风帘翠幕,参差十万人家。云树绕堤沙。怒涛卷霜雪,天堑无涯。市列珠玑,户盈 罗绮,竞豪奢”,前面一句是总说,这几句是对前面的分述;又如“羌管弄晴,菱歌泛夜,嬉嬉钓叟莲娃。千骑拥高牙。乘醉听萧鼓,吟赏烟霞。 《雨霖铃》 一 略 zhîumùǎiyē 二骤 暮 霭 噎 三 对长亭晚 执手相看泪眼,竟无语凝噎 暮霭沉沉楚天阔 更那堪、冷落清秋节 杨柳岸、晓风残月 四 ?离别时的情景 离别后的情景 ? 凄凉、伤感、悲凉、忧伤、痛苦. ? 时间顺序 离别时(离别情景)、离别后(别后想象) “寒蝉” “长亭” “骤雨”词人一开头就捕捉到有特征的秋景来渲染气氛,“寒蝉”透着秋意,而? 蝉的叫声更让人感觉的悲凉,骤雨过后天气陡然变凉,更使人感到凄切冷落,所以这首词不仅交代了离别的时间、地点、更是在写凄凉心情。 ? 前一句是实写不忍别离又不能不别的情况。这六个字明显地写出地点、动作和情绪,是高度压缩的精练的写法。“留恋处,兰舟催发”,是说正在留恋不舍的当儿,舟子已经催促他出发了。从“催发”中可以看出他们多么依依不舍。在这时候,真是纵有千言万语也给喉咙噎住说不出口了。只有紧握着手,泪眼相对而已。这一形象的刻画,看来似很简单,实则是情感的集中表现,是很真挚动人的。这句要读得“哽咽”欲哭,欲言又止。 ? 邈远之景,衬托出情人相见无期,自己前途茫茫的惆怅;承上启下,既是总结上阙的离别情景,又是为下阙写别后情景作铺垫。 ?伤离别。词作下阕一落笔就点明了全词的主题是“伤离别”。自古以来,多情的人,有感情的人,都会因为离别而感;人生当中,什么最令人痛苦,那就是生离和死别。离别本来就是人生中最令人悲伤难过的事,更何况正处在冷落的清秋节呢~秋天本来是草枯叶落,《雨霖铃》中,词人把离别之事与清秋之气叠加在一起,融合在一起,非常悲苦,再加上特别凄凉,这就使情感的表达更加沉重。正是“此情无计可消除,才下眉头,却上心头”啊。 ?因为没有人陪伴,无论怎样的良辰好景,对词人来说都是没有意义的。请大家注意,词人在这里只写到良辰好景,而没有写到平常时日。为什么呢,因为连良辰好景都形同虚设,那平常日子中的度日如年就可想了,在漫长的日子里,再好的良辰,再妙的美景,没有知心人相伴也是形同虚设。就算有千种深情,万般爱意,又向谁去说呢。 五 略 六 略 七 ? 多情自古伤离别,更那堪、冷落清秋节~这首词就是在冷落清秋的景色中抒发他离别的伤感之情。 ? a 意象:蝉、长亭、雨、兰舟、烟波、暮霭、楚天晚、杨柳、晓风、残月;b作用:渲染凄清意境,烘托人物感情 ? 略 6 苏轼诗两首 2 六中 胡金鹏 《念奴娇•赤壁怀古》导学案 教学目标: 1、了解苏轼的生平和思想,了解词的写作背景。 2、诵读吟咏,感受诗词恢宏开阔的意境。 3、学习品析,感受词人壮志难酬的心怀。 4、理解词人复杂的心情,丰富学生的情感世界。 教学重点:教学目标2、3 教学难点:情与景的自然结合 教学方法:诵读法 讨论法 知识链接: 1、宋词的风格流派 豪放派和婉约派是宋代词坛上的两大流派,其作品分别代表了不同的风格。豪放派作品气势豪放,意境雄浑,词中充满豪情壮志,给人一种积极向上的力量,代表作家是苏轼、辛弃疾。婉约派作品语言清丽含蓄,词中抒写的感情婉转缠绵,情调或轻松活泼,或婉约细腻,体裁较狭窄,多是写个人遭遇,男女恋情,也间有写山水,融情于景的。婉约派代表词人有柳永、秦观、李清照等。 2、关于苏轼 生平: (1037-1101)北宋文学家、书画家。字子瞻,号东坡居士,北宋文学家,画家,眉州眉山(今属四川)人。一生宦海沉浮,诗与黄庭坚并称“苏黄”,词与辛弃疾并称“苏辛”,散文与欧阳修并称“欧苏”,书法与黄庭坚、米芾、蔡襄并称“宋四家”。与其父苏询、其弟苏辙并称“三苏”。 政治上苏轼是失意的,但就个人创作成就而言,许多人认为苏轼堪称“中国古代第一全才。”词发展至北宋中期,在人们心目中仍为“小道”,不能登大雅之堂。苏轼一出,扭转百余年的词坛颓风,开拓了词的意境和表现手法。 思想:苏轼思想较复杂,虽以儒家思想为主,但老庄哲学和释道思想也很浓厚。苏轼为人耿直,才意迈峻,重节操。 3、写作背景: 北宋元丰五年,苏轼因“乌台诗案”被贬为黄州团练副使,游赤鼻矶,作词抒怀。本首词就是苏轼贬官黄州后的作品。43岁时因作诗讽刺新法,被捕下狱,出狱后贬官为黄州团练副使。这是个闲职,他在旧城营地辟畦耕种,游历访古,政治上的失意滋长了他逃避现实和怀才不遇的思想情绪,但由于他豁达的胸怀,在祖国雄伟的江山和历史风云人物的激发下,借景抒情,写下了一系列脍炙人口的名篇,此词为其代表作。 学习过程: 一、 基础巩固(A级) 1(下列各组词语中加黑字的字形、注音都正确的一组是:( ) A. 纶(lún)巾 酹(lâi)酒 樵(qiáo)悴 崩摧(cuī) B. 纶(lǔn)巾 酹(lēi)酒 樵(qiáo)悴 崩催(cuí) C. 纶(guān)巾 酹(lēi)酒 樵(qiáo)悴 崩摧(cuī) D. 纶(guān)巾 酹(lâi)酒 憔(qiáo)悴 崩摧(cuī) 2(判断下面正误,对的画?,错的画×。 A(词有词牌,词牌是指词的题目。一般来说,一首词为一阙,“阙”在文字上,指词的分段。( ) B(词牌是指词的曲调名称。词的题目指填词用的题目,词的分段为上片、下片, 或称上阙、下阙,因为一曲便是一阙。( ) C(词牌指填词用的曲牌名,“念奴娇”、“永遇乐”之类便是;它们又是词的题目,而“赤壁怀古”、“京口北固亭怀古”之类只不过是副标题。一曲为一阙,阙在文字上指词的曲调。( ) D(词,是歌词,是一种按照乐谱的曲调和节拍来填写,歌唱的文学作品,它和音乐有密切的关系。所以,词的种种特点,大都是由它的性质所规定的。( ) 二、深层理解(B级) 3.从下列说法中选出正确的几项:( ) A(“大江东去”中的“大江”指长江;“长河落日圆”中的“长河”指黄河。 B(“故国神游”一句中,“故国”指旧国,旧地,即今天的中原地区。 C(“卷起千堆雪”和“樯橹灰飞烟灭”两句都运用了比喻修辞格。 D(“羽扇”“纶巾”都是古代武将的装束。 4.“羽扇纶巾,谈笑间,樯橹灰飞烟灭”。对这诗句理解正确的一项是( ) A. “羽扇纶巾”,指诸葛亮。诗句表现出诸葛亮衣着儒雅,仪态从容,大破曹军。 B. “羽扇纶巾”,指诸葛亮。诗句表现出诸葛亮衣着儒雅,仪态从容,协助周瑜大破曹军。 C. “羽扇纶巾”,指周瑜。诗句表现出周瑜衣着儒雅,仪态从容,在诸葛亮协助下大破曹军。 D. “羽扇纶巾”,指周瑜。诗句表现出周瑜衣着儒雅,仪态从容,大破曹军。 5.“故国神游,多情应笑我,早生华发”。对诗句理解正确的一项是 ( ) A. 周瑜神游当年破曹旧地,应该笑我(苏轼)倾慕他的破曹伟业,而早已长出白头发了。 B. 我(苏轼)神游当年周瑜破曹的战场,周瑜应该笑我倾慕他的破曹伟业,而早已长出白头发了。 C. 我(苏轼)神游当年周瑜破曹的战场,人们应该笑我倾慕周瑜的破曹伟业,而早已长出白头发了。 D. 我(苏轼)神游当年周瑜破曹的战场,应该笑自己倾慕周瑜的破曹伟业,而早已长出白头发了。 三、问答题(C级) (1)“乱石穿空,惊涛拍岸,卷起千堆雪”。试分析这几句诗在景物描写上的表现力量。 (2)苏轼为什么自讥“多情”,并且说“人生如梦,一尊还酹江月”, (3)这首词塑造了词人自身什么样的形象,创造了什么样的意境,抒发了什么样的感情, 四、课外阅读(D级) 蝶恋花 苏轼 花褪残红青杏小。燕子飞时,绿水人家绕。枝上柳绵吹又少,天涯何处无芳草。 墙里秋千墙外道。墙外行人,墙里佳人笑。笑渐不闻声渐消,多情却被无情恼。 1、“绿水人家绕”中的“绕”字,有的版本写作“晓”。你认为哪个字更恰当,为什么,请简要赏析。(4分) 2、俞陛云在《宋词选释》中对这首词的上阕作过这样的整体评价:“絮飞花落,每易伤春,此独作旷达语。”你同意他的看法吗,为什么,请结合词的内容简要赏析。(4分) 五、课堂小结 这首词塑造了词人立足万里长江岸上,面对古代战场,倾慕周瑜伟业,慨叹自己无成的形象,创造出开阔深沉、壮丽奇险的意境,抒发了渴望建功立业而不可得的抑郁感情。 六、作业布置 背诵: 念奴娇赤壁怀古鉴赏 古典诗歌中咏史、怀古一类的作品,一般都是古为今用,借对史事的评论、对古迹的观赏来抒发自己的怀抱。这首词也不例外。他想到古代“风流人物”的功业,引起了无限的向往,同时就引起了自己年将半百,“四十五十而无闻焉,斯亦不足畏也已”(《论语》)的感慨。 一起头二句,是词人登高眺远,面对长江的感受。江水不停地东流,波涛汹涌,气势奔放,自然使人不可能不想起过去那些历史上留下了丰功伟绩,因而与祖国的壮丽山河同样永远保留在后人记忆里的英雄们。当然,这些人是属于过去的了,就像沙砾被波浪所淘汰了一样。但是不是他留下的历史遗产也被“淘尽”了呢,那可不是的。“风流人物”的肉体虽已属于过去,而他们的事功却是不会磨灭的,它属于现在,也属于将来。这两句,江山、人物合写,不但风格雄浑、苍凉,而且中含暗转,似塞实通,有“山重水复疑无路,柳暗花明又一村”之妙。否则,我们一看,“风流人物”都被“浪淘尽”了,那就没有什么可说的了,还有什么下文呢, 正因为暗中有此一转,所以才可由泛泛的对于江山、人物的感想,归到赤壁之战的具体史迹上来。未写作战之人,先写作战之地,因为是游其地而思其人的。江、汉一带,地名赤壁的有好几处。发生在汉献帝建安十三年(208)那一场对鼎足三分的政治形势具有决定性作用的大战,事实上发生在今湖北武昌境内,而不在黄州。博学如苏轼,当然不会不知道。但既然已经产生了那次战争是在黄州赤壁进行的传说,而他又是游赏这一古迹而不是来考证其真伪的,那么,也就没有必要十分认真地对待这个在游赏中并非十分重要的问题了。其地虽非那一次大战的战场,但也发生过战争,尚有旧时营垒,所以用“人道是”三字,以表示认为这里是“三国周郎赤壁”者,不过是传闻而已。“赤壁”而冠以“三国周郎”,为的是突出其历史意义,并为下面写周瑜先伏一笔。 这首词在内容上,表现了作者用世与避世或入世与出世思想之间的矛盾,这是封建社会的知识分子具有的普遍性的矛盾,既然没有机会为国为民做出一番事业,就只有在无可奈何的心情之下,故作达观。所以它在赞赏江山、人物之余, 最后仍然不免趋于消极。但总的说来,最后这一点消极情绪,却掩盖不了全词的豪迈精神,所以读者还是可以从其中吸收一些有益的成分。 在艺术上,这首词也有它的独特成就。其中最突出的一点就是它将不同的、乃至于对立的事物、思想、情调有机地融合在一个整体中,而毫无痕迹。这里面有当前的景物与古代人事的融合,有对生活的热爱、对建功立业的渴望与达观、消极的人生态度的融合,有豪迈的气概与超旷的情趣的融合。而描写手段则虚实互用,变幻莫测,如:“人道是,三国周郎赤壁”,是实的地方虚写;“遥想公瑾当年”,是虚的地方实写。有“人道是”三字,则其下化实为虚,对黄州赤壁并非当日战场作了暗示。有“遥想”二字,则其下虽所咏并非原来的战场,而且还掺入了虚构的细节,仍然使人读去有历史的真实感。 《定风波》导学案 教学目的: 1、了解苏轼的生平和思想,了解词的写作背景。 2、鉴赏词的写作技巧,学习从生活小事中提炼哲理的方法。 3、反复诵读,采用“知人论世”的方法,理解词的内容。学习词人豁达的胸襟和乐观的人生态度。 教学重点: 教学目标2、3 教学难点: 教学目标3 学法指导:诵读法、讨论法 知识链接: 1(苏东坡是在苦难中完成了自己的一个人物,他追求的是一个可以无待(依靠)于外的完成。客观上不得意,还能不落迂腐消极,能够有积极的生活志趣,有持守的一种修养,这是苏东坡了不起的地方。 2(东坡先生非心醉于音律者,偶尔作歌,指出向上一路,新天下耳目,弄笔者始知自振。(王灼《碧鸡漫志》 3(东坡的父亲,喜欢到处边四方去游学,常常不在家,苏东坡小时受他母亲的教训。有一次读到《后汉书?范滂传》,范滂是东汉党锢之祸时被迫害的一个人,而他不逃避,为了理想付出了生命。东坡问母亲:他日儿要做范滂,母亲能做范滂的母亲吗,他母亲说,你如果能做范滂,我怎么不能做范滂的母亲呢~欧阳修的母亲画荻教子,岳母刺字教子。 4(乌台诗案。他被贬到湖州时写了谢上的表文“臣愚不识时,难以追陪新进,老不生事,或可牧养小民”。表中的话被人摘取,以为他有诽谤朝廷之意。于是下到御史台狱,那里有柏树,所以也叫柏台,上有乌鸦,故称乌台。他写的诗中有“根到九泉无曲处,此心唯有蛰龙知”,差一点被处死。在这九死一生之后,东坡写了《念奴娇》(大江东去),在黄州写了《定风波》(莫听穿林打叶声)。所以经过忧患苦难,苏东坡还能写出这样飞扬,这样潇洒,这样开阔,这样博大,这样超旷风格的作品来,这是苏东坡的修养。 要以无生(忘记自己的得失利禄)的觉悟,才能够成就更伟大的有生事业。 学习过程: 一(给画线的字注音(A级) 同行xíng 吟啸xiào 一蓑suō 烟雨 二(划出朗读节奏(A级) 莫听穿林打叶声,何妨吟啸且徐行。竹杖芒鞋轻胜马, 谁怕,一蓑烟雨任平生。 料峭春风吹酒醒,微冷,山头斜照却相迎。回首向来萧瑟处,归去,也无风雨也无晴。 三(基础理解(B级) 1.下列画线词语与现代汉语相同的一项是(D) A大江东去 B千古风流人物 C回首向来萧瑟处 D同行皆狼狈 2.下列词语解释不正确的一项是(B) A一尊还酹江月 酹:将酒洒在地上,以表示凭吊 B料峭春风吹酒醒 料峭:温暖 C回首向来萧瑟处 萧瑟:风吹雨落的声音 D樯橹灰飞烟灭 樯橹:代指曹操的水军 3.填写下列各句名篇中空缺的部分 故垒西边,人道是,三国周郎赤(1) , , 。 壁。 (2)《念奴娇赤壁怀古》中描绘周瑜的句子:小乔初嫁了, 。 ,谈笑间, 。 (3) ,谁怕, 。 (4) ,归去, 。 四(理解题(C级) 你认为所列举的这些答案该打多少分,请简要分析。 1、在风雨中,“同行皆狼狈”,而词人却能“吟啸且徐行”“竹杖轻胜马”,表现了他怎样的心情, ?表现了作者很乐意,很享受当时的雨景,同时表现作者当时勇于面对现实、乐观无拘无束的心情。 ?表现了诗人阔达、平静、与世无争的轻松心情。 ?运用反衬表达了作者对雨的喜爱和轻松闲逸、自由自在的心情。 ?表现了作者的无奈、悲观的心情。 2 、“一蓑烟雨任平生”的意思是什么,表达了诗人怎样的人生态度, ?表现了作者很乐意,很享受当时的雨景,同时表现作者当时勇于面对现实、乐观无拘无束的心情。 ?表现了诗人阔达、平静、与世无争的轻松心情。 ?运用反衬表达了作者对雨的喜爱和轻松闲逸、自由自在的心情。 ?表现了作者的无奈、悲观的心情。 3.、“也无风雨也无晴”中“风雨”“晴”的含义是什么, ?“风雨”指官场上的争斗,“晴”指官场外的闲适生活。 ?“风雨”指官场上的互相倾轧,,“晴”指一帆风顺,没有灾害,暗含官场的争斗。 ?“风雨”既指刮风下雨,又指对词人的贬官;“晴”既指雨后初晴,又指词人重新被皇上重用。 4、这首词与《念奴娇》“人生如梦,一樽还酹江月”的感情基调是否相同,为什么, ?不同。《念奴娇》的基调极为雄壮、激昂,而这首词的基调较为悲凉。 ?不同。这首词的基调是乐观恬淡;而《念奴娇》则比较潇洒、豪放。 ?“人生如梦”多少表达了诗人无奈之情,也包含了对官场的些许期盼等待之情。而此词,应是后期的作品,是诗人思想内容的转折,显示了其归隐之心,是看淡官场,向往田园之情。 五(拓展练习(D级) 通过两首词的学习,大家一定对苏轼有了较深入的理解,请写一篇小作文:我眼中的苏轼 定风波赏析 词开头的第一句,“莫听穿林打叶声”,“穿林打叶声”是指风雨穿过树林在叶子上发出的沙沙声响。用“穿林打叶声”来描写风雨声,很形象,给人以十分真切的感受。风雨来临时,作者正在野外出游,身边并没有雨具,一般的人在这样的境遇下一定很狼狈,会急于慌慌张张地去寻找个避雨场所。事实也是这样,作者的小序就提到:“同行皆狼狈”。可是苏轼却一反常人之所为,不但没有一点惊慌狼狈之态,而且显示了少有的从容不迫、悠然自在的神态。他出人意料地来了个“何妨吟啸且徐行”。“何妨”是“不妨”。“吟啸”,是指吟诗长啸。“徐行”,是慢慢地走。苏轼在风雨之中独自漫步吟诗长啸,这种表现是何等地与众不同~极富于浪漫色彩,通过这句,就把苏轼鲜明独特的个性一下子突现出来了。 “芒鞋”是草鞋,可见作者这次出外郊游是一身野服打扮,他手持竹杖,脚穿草鞋。在苏轼看来这种打扮比起穿了官服骑着马要强得多,这里从一个侧面,透露了作者一贯喜好自然、无拘无束的性格。在上片结束时,作者用了这么一句:“谁怕,一蓑烟雨任平生。”这是个不同凡响的惊人之笔~它画龙点睛般地表现出了作者的胸怀、抱负,体现了全词的中心思想。这句从字面上解释,无非是说,“怕什么呢,自己的一生就是披着蓑衣在风雨之中过来的,对此我早就习以为常、处之泰然了。”“任平生”三字是指平生饱经风雨,早已听其自然的意思。当然, 这里的“风雨”,不仅是指自然界的风雨,更重要的是指政治上的风雨。 下片中,自然界情况发生了新的变化。换头后的第一句:“料峭春风吹酒醒,微冷。”从“吹酒醒”三字中,可以看出,苏轼是在带有醉意的情况下出游的。在被贬黄州期间,苏轼处境艰险、内心苦闷,因此借酒浇愁就成了常事,有时竟喝得酩酊大醉,不省人事。“料峭春风吹酒醒,微冷”是说经略带寒意的春风一吹之后,酒醒了,这时身上微微地感到有些寒冷。紧接着来的是“山头斜照却相迎”一句,它写出了自然界天气变化之快,刚刚自己还在风雨中行进,现在迎着他的却是山头的斜阳了。自然界忽晴忽雨,变化不定;而政治舞台上的晴雨表也是升沉不定。社会上政局犹如自然界的气候一样,变幻莫测。“回首向来萧瑟处”,这里的“萧瑟处”,是指作者刚才遇雨的地方。天气的突然放晴,引起了作者“回首向来萧瑟处”的兴趣,看看原来下雨的地方,现在又发生了什么新的变化呢。全词以“归去,也无风雨也无晴”作结束。这样结束,初看似乎不太好理解,但仔细一琢磨就会感到这样写实在太好了。含蓄隽永,耐人寻味,发人深思。对此究竟应作怎样的理解呢,有的解释是这样的:“政治场合的晴雨表是升沉不定的,不如归去,做一个老百姓,不切实际地幻想着‘也无风雨也无晴’”。这样的解释当然也不失为一家之言,但似乎和前面的“谁怕,一蓑烟雨任平生”中所表现的情绪,以及苏轼其人一贯的气质并不太吻合。看来还不如作这样的解 释为好:“回去,对我来说既没有晴天也没有雨天。”也即无所谓晴天、雨天。意思是晴天也好,雨天也好,对我说来都是无所谓的。这样就同前面的“谁怕,一蓑烟雨任平生”是前后呼应的,通过这种写法进一步强调了自己的心胸、志向以及对人生的态度,从而作者的个性也就表现得更鲜明了。 总之,这首《定风波》通过生活中的一件平常小事——途中遇雨,借题发挥,表达了作者在种种打击和挫折面前不退缩、不丧气,坦然处之的旷达心境。作者巧妙地把自然界的风雨和政治变化中的风风雨雨联系起来,给人们以多方面的联想,大大增强了词的韵味。 苏轼词两首 参考答案(一) 1(D 2(AC 3. A(×) B(?) C(?) D(?) 4(D 5. D 三((1)以“乱”修饰“穿空”之“石”,表现出岩峰错列,直插云霄的态势;以“惊”修饰“拍岸”之“涛”,表现出拍击江岸的波涛汹涌奔腾,“卷起千堆雪”,“卷起”这个动词,“千堆雪”这个比喻,形象地表现出汹涌的波涛猛拍陡峭江岸,翻卷而起形成簇簇浪花,宛如堆堆白雪。这几句诗表现出赤壁古战场的壮丽奇险,体现了词人神驰赤壁古战场的心情。 (2)苏轼神驰当年周瑜破曹的战场,倾慕周瑜的破曹伟业,渴望像周瑜那样建功立业。但周瑜三十四岁就破曹操,立大功,他年已四十有七,却事业无成;所以自讥“多情”??倾慕周瑜,渴望建功。这自我讥笑当中,包含着浓重地岁月蹉跎、壮志难酬的苦涩心情。“人生如梦,一尊还酹江月”,则更是在坎坷遭际中的强自排遣,在消极情调之中也包含有报国无路的感慨。 (3)这首词塑造了词人立足万里长江岸上,面对古代战场,倾慕周瑜伟业,慨叹自己无成的形象,创造出开阔深沉、壮丽奇险的意境,抒发了渴望建功立业而不可得的抑郁感情。 四1、(1)“绕”字好,因为它切实具体地描绘出了绿水环抱人家的场景,生动形象,具有动态美。 (2)“晓”字好,因为它既点明了时间,又渲染了早晨的清新氛围,能够使读者有更自由、更广阔的想像空间。 2、答案可以有两种,答其中一种即可。 ?同意。起句“花褪残红青杏小”,虽写了花之凋零,却又写了青杏新生,显示出生机与活力;二、三句则又移向更广阔的空间,燕子轻飞,给画面带来了盎然生气,而绿水绕人家也饶有情趣,这样一来,人的心情也自然随之敞阔;末句虽言萋萋芳草,却以“天涯”起笔,意境开阔。总之,词的上片虽写“絮飞花落”的暮春之景,却处处可见旷达之语。 ?不同意。起句写花之凋零,青杏酸涩,为整首词投下了悲凉的阴影;二、三句写 燕子翻飞、绿水绕人家,虽富情趣,也不乏暖意,但却是以乐景衬哀情;四句说柳絮飘 飞,这一“又”字,则又表明词人之看絮飞花落,非止一次,伤春之感、惜春之情自然 流出;“芳草”在古诗词中常用来写愁情,此处亦然,“天涯何处无芳草”即言愁情无限。 因此,这首词中对“絮飞花落”等景物的描写,依然浸透着伤春之情,并非旷达之语。) 参考答案(二): 一(同行xíng 吟啸xiào 一蓑suō 烟雨 二(莫听\穿林\打叶声,何妨\吟啸\且徐行。竹杖\芒鞋\轻胜马, 谁怕,一蓑烟雨\任平生。 料峭\春风\吹酒醒,微冷,山头\斜照\却相迎。回首\向来\萧瑟处,归去,也无风雨\也无晴。 三( 1.D 2.B 3.略 四( 1、参考答案:闲适、轻松、自如。 2、参考答案:披着蓑衣在风雨中过一辈子也处之泰然。反映作者不避风雨,听任自然的生活态度。 3、参考答案:处境的好坏;官职的升降;地位的得失等。 4、参考答案:不同。前者表现词人豁达、乐观、积极的心境;后者则反映其消极悲观的人生态度。 五(略 7 辛弃疾词两首 2 辛弃疾词两首导学案 学习目标: (一)情感目标: (1)引导学生以真挚的感情品读品 (2)培养爱国主义情感 (二)知识目标: (1)通过反复诵读,感知作品内容和思想感情 (2)掌握咏史怀古词的特点 (3)理解词中所用典故的含义 (三)能力目标:通过课堂教学进行自主式、探究式学习,开拓知识面,丰富教学内容,提高诗歌鉴赏能力。 学习重点:知识目标(1)、(2) 难点:知识目标(3) 学习指津 : 以学生活动为主,点拨启发,引导讨论,朗读背诵,检查落实 比较学习法 本课文所选辛弃疾的两首词,主题相同,都表现了作者御敌抗金的爱国思想和壮志难酬的愤慨之情。总体风格是雄健、豪放。都大量使用了历史典故,借古讽今。 两首词作于不同时期。《水龙吟》是早期作品,作者时任江东安抚司参议官,词中虽然用了蓝天、绿水、青山、红日等景象来陪衬、点染,但没有柔媚繁缛之感。《永遇乐》是晚期作品,辛弃疾当时以65岁的高龄在镇江任职,他本以为可以积极备战,积蓄军事力量,但很快发现韩侂胄为了邀功,只想草率出兵,于是,作者有了更多的悲愤。可见,他早期的作品多些昂扬、激奋;晚期作品则转向了苍劲、悲壮。 同为豪放派的词人,苏轼与辛弃疾的风格也同中有异。苏轼天性忠爱,自如畅达;辛弃疾气势沉雄,豪壮悲愤。苏轼词风旷达,辛弃疾词风豪健。评论者总是将苏、辛并称,说:“东坡之词旷,稼轩之词豪。 学习过程: 《水龙吟 登建康赏心亭》导学案 一、预习感知 1、作者: 辛弃疾,南宋词人。字 号 ,山东历城人。作品题材广阔,风格多样,以 (风格)为主,关于用典,也善于白描,开拓了词的疆域,提高了词的表现力。 背景:本词作于宋孝宗乾道五年,辛弃疾通判建康,当时他正值壮年有为,但朝士都庸俗之辈。词人此时已南归八年,虽几番上书奏御敌之策,但都无下文。英雄失意,年华虚掷,难得一遂报国之愿。这年,他登临建康赏心亭,写下这首词,借以倾吐自己的抑郁悲愤之情。 2、注音: 玉簪( )螺髻( ) 鲈鱼堪脍( ) 揾英雄泪( ) 3、写下你的疑问: 二、探究学习 1、整体感知: (1)你感觉词的风格怎样, (2)作者表达的主要情感, 2、分片讨论: (1)上片的主要表现手法是什么,试加以分析。 (2)下片的主要表现手法是什么,试加以分析。 3、总结整合: 用简练的语言总结这首词。 三、自测训练 1、“遥岑远目,献愁供恨,玉簪螺髻” 一句中“玉簪螺髻”是形容 ,运用了 修 辞。 2、“落日楼头,断鸿声里,江南游子”中“落日”二字,比喻 。“断鸿”是失群的孤雁,比喻 。 3、“把吴钩看了,阑干拍遍”一句。“把吴钩看了”,借吴钩闲置身旁,烘托出词人 的苦闷,“阑干拍遍”形象地表现出作者 的情态。 《永遇乐 京口北固亭怀古》导学案 一、【基础知识梳理】: 1、给下列加点的字注音 孙仲谋( )舞榭( )巷陌( )金戈( )元嘉( )狼居胥( )仓皇( )可堪( )佛狸祠( )拓跋焘( ) 张浚( )王玄谟( ) 2、解释下列词语,掌握在词中的意思: 风流: 寻常: 草草: 北顾: 可堪: 二、研习课文,探讨用典的意义和作用。 (一)所谓用典,就是引古事、古人来比喻今事、今人以抒发情怀,是古代诗文中常见的一种写作手法,统称“用典”。恰当地用典,可使诗文文情隽永,含蓄深刻。用典或仰慕古人;或以古人自况,感慨身世;或借古讽今等等。言简意赅,以一当十。 (二)辛弃疾在文学史上被评论家讥之以“掉书袋”,原因是他的词作好用典故。全词用了哪几个典故,你认为每一个典故的用意是什么,(对照注释、材料分组讨论) 明确: 上阙用了 、 两个典故, 下阙用了 、 、 、 三个典故。 上阙: 1. 分析研讨前两个用典。 ? 说说这两个人的共同点, ? 用此二人典,蕴含了什么用意, 下阙: 2. 分析研讨第三个用典。 辛弃疾引用宋文帝北伐惨败的故事目的是什么,(用典的用意何在,) 3. 分析研讨第四个用典。 诗人写佛狸祠下祭神庙会的热闹景象,是什么心情, 4.、有人说最后一段洋溢的是满腔豪情,也有人说是悲情怨语,说说你的理解。分析研讨用廉颇典故的用意。 辛弃疾词两首学案导学答案 《水龙吟 登健康赏心亭》学案导学答案 一、1、字幼安,号稼轩,以豪放为主。2、zān,jì,kuài,wèn 二、1、(1)这首词,境界阔大,气魄雄浑,寄意遥远,感慨深沉。豪放。 (2)作者登上建康城(今南京)的赏心亭,面对辽阔山河,听到鸿雁悲鸣,生出无限忧思。自伤抱负世无知已,得不到理解与慰藉;浩叹英雄徒有雄心壮志,却无处施展才华。 2、(1)借景抒情 。 词人由水写到山,有无情之景写到有情之景,“楚天千里清秋,水随天去秋无际”是词人在赏心亭上看到的江景,水天一色的辽远和如簪似髻的山影便牵动了他久蓄的愁苦。 遥岑远目,献愁供恨,玉簪螺髻”三句,是写山。“遥岑”即远山。举目远眺,那一层层、一叠叠的远山,有的很象美人头上插戴的玉簪,有的很象美人头上螺旋形的发髻,景色算上美景,但只能引起词人的忧愁和愤恨。 “落日”本是自然景物,辛弃疾用“落日”二字,含有比喻南宋国势衰颓的意思,亦表达了作者的愁苦心情。 “断鸿”是失群的孤雁,比喻自己飘零的身世和孤寂的心情。 2)用典。就本词来看,词人用典意在对比,前两个是反例,后两个是正例,正反对比表达了词人的荣辱( 观、价值观。“尽西风、季鹰归未,”既写了有家难归的乡思,又抒发了对金人、对南宋朝廷的愤懑,“求田问舍,怕应羞见,刘郎才气” 表明了词人为国忘私的广阔胸怀,也批判了那些不顾国事、钻营私利的人。 “可惜流年,忧愁风雨,树尤如此”, ,辛弃疾用这个典故,抒发自己不能为抗击敌人、收复失地而效力,徒然虚度年华的愤慨和苦痛。 3、这首词,上片写景抒情,下片则是直接言志。境界阔大,气魄雄浑,寄意遥远,感慨深沉。通过所见所闻所感,层层展开主题。结构曲折而严谨。而且典故使用贴切精妙,天然浑成。不愧为传世之作。 三、1、一层层、一叠叠的远山,比喻。 2、比喻南宋国势衰颓;比喻自己飘零的身世和孤寂的心情。 3、把作者虽有沙场立功的雄心壮志,却是英雄无用武之地的苦闷也烘托出来了;雄心壮志无处施展的急切悲愤的情态。 《永遇乐?京口北固亭怀古》导学案 一、1、孙仲谋( móu )舞榭(xiè )巷陌(mò )金戈(gē )元嘉( jiā )狼居胥(xū )仓皇(huáng )可堪(kān )佛狸祠(fó )拓跋焘(tāo ) 张浚(jùn )王玄谟( mó ) 2、风流: 寻常:平常。草草:草率。北顾:北望。可堪:不忍。 二、(二)上阕:“千古江山,英雄无觅、孙仲谋处”; “斜阳草树,寻常巷陌,人道寄奴曾住”。 下阕:“元嘉草草,封狼居胥,赢得仓皇北顾”;“四十三年,望中犹记,烽火扬州路” ;“佛狸祠下,一片神鸦社鼓”。 1、?都是能够建立功业的英雄人物,而且他们的事业都是在京口起步的。 ?表达自己力主抗金和决心恢复中原的宏大抱负,同时借古代帝王来讽刺南宋统治者屈辱求和的无耻行径。 2、借鉴历史,伐金必须做好准备,不能草率从事。 3、作者的心情是沉重的,表达了自己的隐忧:如今江北各地沦陷已久,不迅速谋求恢复,民俗就安于异族统治,忘记了自己是宋室臣民。表示对南宋政府不图恢复中原的不满。 4、辛弃疾这首词最后用廉颇事作结,是作者到老而爱国之心不衰的明证。廉颇虽老,还想为赵王所用。他在赵王使者面前一顿饭就吃了一斗米作的饭、十斤肉、又披甲上马,表示自己尚有余勇。辛弃疾在这词末 了以廉颇自比,也正表示自己不服老、还希望能为国效力的耿耿忠心。雄心不减当年,切望为国效力,叹无人前来问讯徒有英雄豪情 8 李清照词两首 2 《李清照词两首》导学案 【学习目标】 1、了解李清照的生平与创作。 2、鉴赏李清照作品的婉约风格,比较婉约与豪放两种词风的异同。 【目标难点】 1、诵读两首词,了解词作风格与社会环境、个人遭遇的关系。 2、体味词作中的意象和由意象营造的意境,以及词句中凝聚的感情。 3、理解词中“愁”字的由来,在词的艺术境界中陶冶美好的情操。 【学法指导】 诵读法,背景解读法 【资料链接】 公元1084年出生的李清照,距今快一千年了,但我们对她一点都不感到陌生。她天生丽质犹如一朵带露的花,美丽的双眸总是渗满迷人的清泪;她娇花照水好像一弯遮云的月,妩媚的面庞总是挂着深沉的愁绪。曹雪芹笔下同样多愁善感的林黛玉有资格和她媲美,但林黛玉只是儿女情长的女儿愁,太单一也太狭隘。李清照为爱情而愁,为婚姻而愁,为家国而愁,为民族而愁。她像一缕带香的风,越过历史的时空朝我们款款走来,走进人们空旷而幽深的心灵;她又像一片轻盈的云,隔着岁月的清流离我们盈盈远去,可永远也走不出人们追随的视线。 作者 李清照(1084——1155),号易安居士,济南(今山东济南)人,宋代女词人,著名学者李格非之女,幼有才藻,十八岁嫁给金石家赵明诚,夫妻感情甚笃。南渡后,赵明诚病故,她颠沛流离于江浙皖赣一带,在孤寂中度过晚年。她工诗,能文,更擅长词。李清照是宋代婉约派大家。她的词婉约而不流于柔靡,清秀而具有逸思,富有真情实感,语言清新自然,流转如珠,音调柔美,故名噪一时,号为“易安体”。 作品 有《漱玉词》。前期多为闺情词,反映大家闺秀的生活情趣,清新明丽,意境优美。有《如梦令》二首、《凤凰台上忆吹箫》《一剪梅》《醉花阴》等。词后期充满了凄凉、低沉之音,主要抒发悼亡之悲和怀旧之思,以寄寓家国之痛和故土之思。如《菩萨蛮》《永遇乐》《武陵春》《声声慢》《清平乐》等。 背景信息 《醉花阴》写于词人生活的前期。当时赵明诚在山东莱州做官,李清照留在老家青州居住。李清照形单影只,居室寂静,再不闻评诗论文、查书赌茶、鉴赏碑刻的欢声笑语。李清照写了这首词给明诚,以寄相思。 《声声慢》是李清照晚年的作品。这时她不但连续经历了国家败亡、远离故乡、丧失丈夫的灾祸;在南方到处辗转逃亡避难中,她丢失了珍爱的文物、古籍,还遭到政治上的诬陷。她晚年的处境极其凄惨,心境极其恶劣。她用这一曲抒情长调,艺术地表现了自己晚年的生活状况和内心情感。 【学习过程】 一、 品读全文,要求读准字音,看准字形 戚戚( ) 乍暖还寒( )( ) 有谁堪摘( ) ((((( 二、 研读课文,对译课文并将加点字词解释出来。 薄 雾 浓 云 愁 永 昼 , 瑞脑 ((((( 销 金 兽。 佳节又 重阳, 玉枕 纱厨 , (((((((( 半夜凉 初透 。 东 篱 把 酒 黄昏后, 有 暗 香 盈袖。 莫 道 不 销魂, ((((( 帘 卷 西风, 人比黄花 瘦。 (((( 三、 文段分析思考 1. 《醉花阴》这首词写出了词人的哪些愁绪,请概括一下。 2、这首词哪几句为全词最精彩之笔,请作简要分析。 3、古人常常爱用花比喻人之美貌,如“芙蓉如面柳如眉”“人面桃花相映红”等,而李清照却说“人比黄花瘦”。赵明诚的朋友也认为“莫道不消魂,帘卷西风,人比黄花瘦”三句绝佳。请分析一下这个比喻的妙处。 点拨:分析词人与“黄花”的相似之处,明确词人的情感。 四、研读课文,对译课文并将加点字词解释出来。 寻 寻 觅 觅, 冷冷 清清, (((( 凄凄 惨惨 戚戚。 乍 暖 还寒 时候,最难 将 息。 (((((((( 三杯两盏淡酒, 怎敌他、 晚来风急~ ( 雁 过也, 正 伤心, 却 是 旧时相识。 满地 黄花 堆积, 憔悴 损, 如今 ( 有谁 堪 摘, 守 着 窗儿,独 自怎生得 黑, 梧桐 ((( 更兼细雨, 到黄昏, 点点滴滴 。 这 次第,怎 一个 (( 愁 字 了得~ 五、 文段分析思考 1、“雁过也,正伤心,却是旧时相识”一句。词人为什么强调是“旧时相识”,表现的是作者什 么样的思想感情, 2、“梧桐更兼细雨,到黄昏,点点滴滴”这几句在写法上有何特点,请简要分析。 3、“寻寻觅觅,冷冷清清,凄凄惨惨戚戚”这几句包含几层意思,有什么作用,这几句在用语上有什么特点, 【知识梳理】 薄雾浓云愁永昼( ) 怎敌他、晚来风急( ) ((((乐郊乐郊,谁之永号( ) 大王必欲急臣( ) (( 永为仪则( ) 县官急索租,租税从何出( ) (( 【思维导图】 愁永昼:薄雾加浓云,瑞脑销金兽——孤独寂寞 醉花阴 思行人:佳节又重阳,半夜凉初透——思念难免 闺愁 人更瘦:东篱黄昏酒,暗香销魂后——自怜愁闷 冷清:黄花堆积,黄昏细雨,点点滴滴 颠沛之苦 声声慢 凄惨:乍暖还寒,最难将息,憔悴不堪 丧夫之痛 忧戚:雁过伤心,怎敌风急,怎生得黑 亡国之恨 【整体感知】 李清照的创作生活始于北宋末,终于南宋初。她既享受过安逸、宁静的生活,也遭遇了国破、家亡、夫死的灾难与不幸。本课所选的两首词反映了李清照不同时期的生活与心态,呈现出不同的韵味、格调。请比较两首词的异同。 明确: 词名 节令 意象 意境 感情 写法 风格 薄雾、浓云、瑞脑、轻烟袅袅、天气百无聊赖 借景抒情 《醉花阴》 重阳 纱橱、玉枕、西风、初凉和菊花吐孤单寂寞 情景交融 委婉 黄花 蕊时的清寂、寂相思之苦 比拟映衬 含蓄 静的意境 晚风送寒、秋雨颠沛之苦 直抒胸臆 不加雕 大雁、黄花、梧桐、连绵、黄花零丧夫之痛 借景抒情 饰 《声声慢》 暮秋 细雨 落、北雁南飞的亡国之恨 对比烘托 朴素自 凄惨、悲凉的意然 境 【妙言妙语】 李清照诗词名句 (1)生当作人杰,死亦为鬼雄。至今思项羽,不肯过江东。 (《夏日绝句》) (2)此情无计可消除,才下眉头,却上心头。 (《一剪梅》) (3)闻说双溪春尚好,也拟泛轻舟,只恐双溪舴艋舟,载不动许多愁。 (《武陵春》) (4)枕上诗书闲处好,门前风景雨来佳。 (《摊破浣溪沙》) (5)天接云涛连晓雾。星河欲转千帆舞。 (《渔家傲》) (6)九万里风鹏正举。风休住,蓬舟吹取三山去。 (《渔家傲》) (7)卖花担上,买得一枝春欲放。泪染轻匀,犹带彤霞晓露痕。 (《减字木兰花》) (8)醉里插花花莫笑,可怜春似人将老。 (《蝶恋花?上巳召亲族》) (9)不如随分尊前醉,莫负东篱菊蕊黄。 (《鹧鸪天》) (10)寂寞深闺,柔肠一寸愁千缕。惜春春去,几点催花泪。 (《点绛唇》) 【写作妙招】 中国古典诗歌的创作十分讲究含蓄、凝练。诗人在处理情感时一般不是直接的抒情,而是言在此而意在彼,叙事则因事抒情,写景则借景抒情,咏物则托物言志,记史则咏史抒怀。 李清照就是抒情的能手。“薄雾浓云”布满整个天宇,用环境的凄冷,衬托出词人内心的凄凉。“瑞脑销金兽”,写出了时间的漫长无聊,侧面写词人独自一人的寂寞无聊。“半夜凉初透”这一个“凉”字,也不仅仅是秋风的凉爽,更是词人心情的反映。上篇不言相思,尽用相 思之物、景来衬托。作者在自然景物的描写中,加入自己浓重的感情色彩,景为情设,情因景生,使客观环境和人物内心的情绪融合交织。“守着窗儿,独自怎生得黑~梧桐更兼细雨,到黄昏、点点滴滴。这次第,怎一个愁字了得~”以极寻常浅白的语言。自然流露出时光难捱的无奈之感。点点滴滴的秋雨,其凄凉的况味,引起作者复杂的感伤情绪,哪一个是“愁”字能概括地了呢, 【品味经典】 渔家傲 天接云涛连晓雾,星河欲转千帆舞。仿佛梦魂归帝所,问天语,殷勤问我归何处, 我报路长嗟日暮,学诗谩有惊人句。九万里风鹏正举。风休住,蓬舟吹取三山去。 问题设计 1、 整首词都是描写梦境,请说说词人的用意。 2、 梁启超评价这首词说:“此绝似苏辛派,不类《漱玉集》中语。”你同意这种说法吗,说明理由。 推荐理由:李清照是一位婉约派的女作家,《醉花阴》《声声慢》等词多半写闺情幽怨,风格是含蓄、委婉的。但《渔家傲》却是她豪放风格的佳作,她用《离骚》、《远游》的感情来写小令,不但是五代词中所没有的,就是北宋词中也很少见。其所以有此成就,无疑决定于作者的实际生活遭遇和她那种渴求摆脱这种生活的思想感情,这绝不是没有真实生活感情而故作豪语的人所能写出的。 【课文翻唱】 千古绝唱凄苦情 文/佚名 金兵入侵,北宋灭亡。志趣相投的丈夫病死他乡。南渡避难,半生收藏的金石文物,遗失殆尽。夜半起床,屋外,寒风刺骨。在寻觅些什么,在这么冷清的夜中,难免独自泪下。反复吟唱,徘徊低迷,婉转凄楚,有如听到一个伤心之极的人在低声倾诉,那种伤感的情绪久久不散。 心情不好,再加上这种乍暖还寒天气,连觉也睡不着了。如果能沉沉睡去,那么尚可在短暂的时间内逃离痛苦,可是越想入眠就越难以入眠,于是她就很自然念起亡夫。也罢~命运至此,还有什么不可忍受的呢,披衣起床,喝一点酒暖暖身子再说吧。可是寒冷缘孤独而起,酒入愁肠,都是相思泪,凄凉孤独的感觉,才下眉头,却上心头。 就这样守着一杯淡酒,在在这天暗云低,冷风正劲的时节,耳畔传来孤雁的声声悲鸣,那种哀怨的声音划破天际,也再次划破了她心头未愈的伤口。头白鸳鸯失伴飞,唉,雁儿,你叫得这样凄凉幽怨,难道也像我一样,暮年失去了相濡以沫的伴侣吗,也像我一样,余生要独自一人面对万里层山,千山暮景吗,心绪在寒风中瑟瑟,泪光迷蒙了双眼,蓦然觉得那只孤雁不就是往昔为自己传递彩笺的那只吗,唉,这似曾相识的归雁~ 那些无可奈何的菊花儿早已憔悴不堪,花雨浸湿了小园的香径,再无当年那种"东篱把酒黄昏后,有暗香盈袖"的雅致了。以前和丈夫在一起的日子多么美好,和唱诗词,整理金石,可现在呢,只剩下自己一个人承受这无边无际的孤独了。故物依然,人事全非。独对着孤雁残菊,更感凄凉。手托香腮,珠泪盈眶。怕黄昏,捱白昼。正在为落花惋惜、为自己忧苦,偏偏又下起淅淅沥沥的秋雨。雨点打在梧桐叶上,浸湿了我的愁绪,一直到黄昏都没停止。对着这阴沉的天,一个人要怎样才能熬到黄昏的来临呢,漫长使孤独变得更加可怕。独自一人,时间也仿佛变慢了起来。湿冷的雨点还在心头滑落,一滴滴,一声声,敲打着满怀愁绪的心扉,简直让人不能自抑~ 然而心中的哀愁又何止这些呢,身世之痛,家国之痛,孤独之病„„仅用一个“愁”字又如何包括得尽,概括得了呢, 一试身手 试从下列角度“翻唱(改写、仿写、续写„„)”这篇课文吧~ (1)将两首词改写成散文 (2)人比黄花瘦 (3)不让须眉 (4)月满西楼 (5)李清照——我想对你说 《李清照词两首》导学案 答案 二、 研读课文,对译课文并将加点字词解释出来。 整天都在发愁。永,长。 一种龙脑,香料,又称冰片。 薄 雾 浓 云 愁 永 昼 , 瑞脑 ((((( 一天从早到晚,天空都布满着薄雾浓云,这种阴沉沉的天气使人整天都在发愁,瑞脑 兽形的铜香炉 节令名,又称重九,为农历九月初九日。 磁枕的美称。 纱帐,蚊帐。 销 金 兽。 佳节又 重阳, 玉枕 纱厨 , (((((((( 香在金兽炉中焚烧着。又是重阳佳节, 玉枕孤眠,纱窗独寝, 半夜凉 初透 。 到半夜凉意袭来,透入帐中枕上,很难将息。 东 篱 把 酒 黄昏后, 有 黄昏在种有菊花的篱边端起酒杯饮酒, 菊花的清淡 菊花的清香。 充满。 形容极度愁苦。 暗 香 盈袖。 莫 道 不 销魂, ((((( 香气充满衣袖。 不要说人不极度愁苦, “西风卷帘”的倒文。 帘 卷 西风, 人比黄花 瘦。 (((( 西风卷起了帘子, 帘中人比菊花还瘦。 三、 文段分析思考 2. 《醉花阴》这首词写出了词人的哪些愁绪,请概括一下。 重阳无阳,有高难登,诗兴难发,此一愁也;丈夫不在,夜晚难眠,独饮闷酒,此二愁也;秋风飒飒,菊残人老,此三愁也。 2、这首词哪几句为全词最精彩之笔,请作简要分析。 “莫道不消魂,帘卷西风,人比黄花瘦”为全词最精彩之笔。“莫道”一句直接对“东 篱把酒”而言,使“人比黄花瘦”的比喻与全词的整体形象结合得十分紧密,而且极 有情思;“帘卷西风”更直接为“人比黄花瘦”句做环境气氛的渲染,使人想象到一 幅西风瘦菊,佳节冷落,佳人对花兴叹,怜花自怜的图画。 3、古人常常爱用花比喻人之美貌,如“芙蓉如面 柳如眉”“人面桃花相映红”,而李清照却说“人比黄花瘦”。赵明诚的朋友也认为“莫道不消魂,帘卷西风,人比黄花瘦”三句绝佳。请分析一下这个比喻的妙处。 点拨:分析词人与“黄花”的相似之处,明确词人的情感。 菊花不止外形上雅淡、清秀,与作者因相思而消瘦的体态相近,而且在菊花品格的传统象征意 义上,也酷似作者清高、淡泊的精神,这样的比喻比较确切地反映了当时作者由于离开丈夫而 孤独、愁闷的生活状态和内心情感。表达是时摒绝浮花浪蕊,而选择不求浓丽、自甘素淡的菊花加以自比也反衬出作者不同凡俗的逸韵。 四、研读课文,对译课文并将加点字词解释出来。 意谓不想失去的一切都找不回来,表现出极度空虚惆怅、迷惘失落的心态。 寻 寻 觅 觅, 冷冷 清清, (((( 内心空虚,想找回那些失去的物事,却只觅得满心的冷清 悲愁、哀伤的样子。忽暖忽冷,气候变化无常。 调养,保养。 凄凄 惨惨 戚戚。 乍 暖 还寒 时候,最难 将 息。 ((((((((和忧伤凄楚。在这种天气忽而回暖又忽而转冷的时候,最难调养休息。 对付、抵抗。 三杯两盏淡酒, 怎敌他、 晚来风急~ ( 三两杯酒无法抵挡住愁情,再加上傍晚秋风迅疾,又加重了愁思。 作者以北方人流离南方,大雁也从北方飞来,故说旧时相识。 雁 过也, 正 伤心, 却 是 旧时相识。 北雁南来,当空飞过,看到后更加伤心,也许正是昔日为我传书的那只雁吧。 极,表示程度高,这里相当于“煞”。 满地 黄花 堆积, 憔悴 损, 如今 ( 院子里的黄花遍地盛开,(而我自己)却因忧伤而这样憔悴, 到如今 可。 怎么,怎样。 有谁 堪 摘, 守 着 窗儿,独 自怎生得 黑, 梧桐 ((( 有谁一起来欣赏它、采摘它呢,孤单无聊独坐窗前,怎么才等到天黑(让我结束这一天的愁苦), 光景,状况。 更兼细雨, 到黄昏, 点点滴滴 。 这 次第,怎 一个 (( 细雨敲打着梧桐,直到黄昏还点点滴滴没有停止。这种滋味,怎么能用 愁 字 了得~ 一个“愁”字说尽, 五、 文段分析思考 1、“雁过也,正伤心,却是旧时相识”一句。此人为什么强调是“旧时相识”,表现的是坐着什么样的思想感情, 作者正在酒后感到寒气袭人之时,最让她伤心的是又见到了飞过的鸿雁,而这南来 的大雁,却是当年在北方的旧相识~作者这时是感叹:大雁可以南来北往,而自己却只滞留在南方不能北归~表现了作者强烈的思乡之情和一种沦落异地的流浪之感。 2、“梧桐更兼细雨,到黄昏,点点滴滴”这几句在写法上有何特点,请简要分析。 这几句是环境描写,用以烘托作者内心的孤独和寂寞。点点滴滴的雨打在梧桐叶上,可在凄苦的作者听来却像打在自己心上,是那么强烈的敲击着、震撼着作者的心扉。 3、“寻寻觅觅,冷冷清清,凄凄惨惨戚戚”这几句包含几层意思,有什么作用,这几句在用语上有什么特点, (1)这几句包含三层意思:第一句,写人的动作神态,似乎在寻找失去的东西,追索过往的陈迹;第二句,写环境的凄凉、寂寞;第三句,写内心世界的愁苦、哀伤。(2)这几句直接抒情,表达的感情哀婉凄切,为全词奠定了凄清的基调。(3)七组叠字,很好的统率了全词的内容,展示出一种凄凉萧条的意境;选用叠字,造成一种回环往复的效果,为全词的抒情定下基调;使用叠字更显示感情之强烈;平中见奇,字字传神。 【知识梳理】 薄雾浓云愁永昼( 长 ) 怎敌他、晚来风急( 快,急速 ) (( 乐郊乐郊,谁之永号( 通“咏”,歌唱 ) 大王必欲急臣( 逼迫 ) (( 永为仪则( 永远,无休止 ) 县官急索租,租税从何出( 迫切,紧急 ) (( 【品味经典】 渔家傲 1、 整首词都是描写梦境,请说说词人的用意。 词人不安于社会给她安排的命运,她渴望自由,追求光明。但这种愿望在她生活的时代的现实中是不可能实现的,因此她只有把它寄托于梦中虚无飘渺的神仙世界,在这境界中寻求出路。 2、 梁启超评价这首词说:“此绝似苏辛派,不类《漱玉集》中语。”你同意这种说法吗,说明理由。 梁启超的评论是正确的。词人把生活感受融入梦境,把《离骚》、《逍遥游》和神话传说化入词中,通过神奇的想象使梦幻与生活、历史与现实融为一体,构成气势恢宏、格调雄奇的意境。它与词人其它作品的清丽、婉约确是很不相同。 《直面挫折 学习描写》导学案 【学习目标】 1、明确描写的含义、特点,了解描写与记叙的不同。 2、认识描写是为刻画人物性格和表现作品主题思想服务的。 3、掌握描写的方法。 4、学会抓住事物特点,选取恰当的描写方法,生动形象有感情的描写事物。 【学习过程】 一、学习描写 1.什么是描写 描即描绘,写指摹写。描写,就是用生动形象的语言,对人物、事件、环境作具体入微的描摹绘写,给人以真切的感受。 2.描写的方法 A.从描写的对象分,一般分为人物描写、环境描写和场面描写。 a人物描写,主要有肖像描写、行动描写、语言描写和心理描写等。 对话描写:《范进中举》:胡屠户凶神走到跟前,说道:“该死的畜生~你中了什么,”一个嘴巴打将去…… 肖象描写:《孔乙己》:他身材很高大;青白脸色,皱纹间时常夹些伤痕;一部乱蓬蓬的花白的胡子。穿的虽然是长衫,可是又脏又破,似乎十多年没有补,也没有洗。 动作描写:《范进中举》:范进不看便罢,看了一遍,又念一遍,自己把两手拍了一下,笑了一声道:“噫~好了~我中了~”说着,往后一交跌倒,牙关咬紧,不醒人事……笑着,不由分说,就往门外飞跑,把报录人和邻居都吓了一跳。走出大门不多路,一脚踹在塘里,挣起来,头发都跌散了,两手黄泥,淋淋漓漓一身的水。 细节描写:《孔乙己》:他从破衣袋里摸出四文大钱,放在我手里,见他满手是泥,原来他便用这手走来的。 心理描写:《范进中举》:自己心里懊恼道:“果然天上„文曲星?是打不得的,而今菩萨计较起来了。”想一想,更疼的狠了,连忙问郎中讨了个膏药贴着。 对比描写:(1)《曹刿论战》:以乡人对国事的冷漠映衬曹刿的爱国、勇于承担;又以鲁庄公的“鄙”映衬曹刿的深具“远谋”。(2)《廉颇蔺相如列传》:以廉颇的胸襟狭窄映衬蔺相如的胸襟广阔。人物描写的主要作用是,刻画人物性格、形象,使读者如闻其声、如见其人。 b环境描写,包括自然环境和社会环境的描写。 c场面描写。场面,一般指由人物活动构成的生活画面,有时也泛指一定场合的情景。所以,描述场面主要是写人的活动,写人物之间的关系。 B.描写的角度分,有正面描写(又称直接描写)和侧面描写(又称间接描写)。 正面描写是作者直接对描写对象(人物、事件、环境)所作的刻画和描绘。 侧面描写是作者通过对周围人物或环境的描绘来表现所要描写的对象,即间接地对描写对象进行刻画、描绘,使其鲜明突出。 C.从描写的手法分,有白描和细描。 白描,即运用最简练的文字,不加渲染烘托,没有浓烈色彩的描写,不借助比喻、拟人等修辞手法,也不用或少用形容词,抓住描写对象的主要特点,描写出事物的本质特征或人物的精神面貌。叙事,线条分明,言简意赅;写人,三言两语则揭示出人物的状态,人物栩栩如生。 细描,即细致具体地描绘对象,运用比喻、拟人、夸张等修辞手法和象征、衬托、渲染等表现手法,使所写对象栩栩如生,逼真动人。 3.描写应注意的问题: 第一,目的明确。我们写文章,不能为描写而描写,要有明确的目的。什么地方需要描写,什么地方不需要描写;什么地方需要详细描写,什么地方不需要详细描写;都要从刻画人物、表现主题出发,而不能兴之所至,就大肆描写一番。 第二,特点突出。不论描写人物还是景物,都要抓住特点、突出特点,这样,给读者的印象是鲜明的、突出的。如果泛泛而谈,虽然具体、细致,也不能给人留下深刻的印象,反而使人感到烦琐、累赘,写得多而没有特点,结果等于什么也没有写。 第三,鲜明而生动。描写要力求鲜明、生动。作者的感情,要在描写中显露出来,表现出鲜明性。通过描写来抒情的文字,自然是饱含感情的;一般的描写,也要把倾向性表现出来。通过对客观事物的描写,表现出褒贬、爱憎,态度力求鲜明。描写人物、景物,都力求使读者得到生动、具体的形象,给人以身临其境之感。这种生动性,并不是借助于过分的雕琢、堆砌,而是在朴实自然中再现事物的形象。 二、话题探讨 1.在你的成长道路上,难免会遇到挫折。其中哪一次挫折给你留下了深刻的印象,面对挫折,你采取了什么样的态度, 2.面对挫折,我们应该采取什么样的态度,说说你自己的看法。 3.在你所了解的人物中(文学作品或真人真事),哪一个直面挫折的人物令你感动,请你对他(她)进行描述。 三、名人面对挫折的故事 贝多芬的故事 贝多芬从4岁起,就受到父亲严格的音乐训练,他整天被关在屋子里练琴。贝多芬很早就学会了演奏钢琴、管风琴、小提琴、中提琴和长笛。由于家境贫困,贝多芬小小年纪便承担起家庭重担,因而没能受到很好的教育。他是靠自学成长起来的。他17岁时,母亲去世了,年轻的贝多芬作为一家之主,同生活顽强地作斗争。他先是被公认为灵感丰富的即兴演奏家和维也纳最好的钢琴家,后来又被公认为优秀的作曲家。但痛苦一直伴随着贝多芬。从27岁起,他的耳朵渐渐地聋了。这对于一个音乐家来说是多么沉重的打击~同时,他还遭受着失恋的折磨,心爱的姑娘一个个离他而去。也正是在这个时期,贝多芬表现出极其旺盛的创作力,写出了许多杰出的作品,被誉为“交响乐之王”,最终获得巨大成功。 富兰克林的故事 雷电是一种自然现象,遇到这样的天气,一般人会想到要下雨了,快回家吧~也有一些人甚至感到有些害怕。但是有一个人专门挑这样的天气去放风筝,你相信吗, 他是谁,他就是作为美国的政治家、外交家、科学家、发明家而闻名于世的本杰朋•富兰克林。作为科学家,他的主要科学成就是在电学方面。富兰克林从事电学研究,进入电学领域,在很大程度上是偶然的。 1746年的某一天,富兰克林走在街头,看到一位欧洲人正在表演称作“电气魔术”的电学实验。他感到极为新鲜,产生了浓厚的兴趣,心中激起强烈的探求欲望。他买下了全部实验品。不久,一位他在伦敦英国皇家学会结识的朋友柯林森得知后,又给他寄来了大批书籍、电学著作和某些摩擦起电的设备。富兰克林立即动手重复在街头所看到的实验,另外还加上了一些新实验。此时,富兰克林已经年近40岁,事业有成,是一位社会名人,工作繁忙,但他全力投入了电学研究。他在自传里写道:“我以前在任何研究上,从没有像现在这样全神贯注过。以至几个月来,没有余暇顾及其他任何事情。这些实验涉及到我完全陌生的事,所以使我感到惊奇,使我得到了满足。” 在开始电学研究不久,根据所观察到的现象,富兰克林认为太空中的闪电和摩擦起电产生的电火花是同一种东西,他认为闪电是带电的云的大量放电造成的,为了证明这一大胆猜测,1752年6月,富兰克林冒着生命危险进行了一次震动世界的风筝实验。富兰克林的风筝实验揭开了雷电神秘的面纱,显示了雷电的本质,对人们的思想产生了极大的震撼,是人类认识自然的历史上一个划时代的进步。实验轰动了全世界。 在风筝实验的启示下,富兰克林又发明了避雷针。避雷针是电学研究给人类带来的第一项有实际应用价值的发明。200余年来,它不知使人类避免了多少次生命和财产的损失。 问题:贝多芬和富兰克林是怎样面对挫折的,从他们的态度中你学到了什么, 四、有关资料 作家谈描写 1.“画眼睛” 画人像,眼睛是很重要的关键。其他轮廓具备,眼睛画得像了,那人物就灵活起来了。 因此,关于画眼睛, 在画史上就流传着许多佳话,有时甚至衍变而成神话。 晋代的顾恺之,画过《女史箴图》《洛神赋图》等名画,一向被人推为中国画史上的第一位画人物的宗师。据说,他画的人物,往往好几年不点眼睛。人家问他为什么这样,他说:“四体妍媸,本无关于妙处,传神写照,正在阿堵(这个)中。” 南北朝时的画家张僧繇,擅于画龙,民间故事说他在金陵安乐寺的壁间画了四条龙,不点上眼睛的时候一切如常,一点上眼睛,龙便破壁飞去了。“画龙点睛”这句话,就由此产生。 现代的湘绣,也很讲究绣眼睛的技巧。据说,当年最著名的湘绣,绣眼睛成为“家传之秘”。人像的眼睛、动物的卷毛这些细活,总是放在内室,由湘绣名家的儿媳们自己做,不传外人。绣线也异常考究,一根丝线,要分成16股;一种颜色,从最浅到最深,要分成13种。用这种丝线和这种技艺绣出来的人物,栩栩如生,那眼睛竟像会动一样。 透过神话的迷雾,透过“玄妙”的气氛,我们看到了艺术上一项重要的真理:画龙必须点睛。不点上眼睛,龙始终活不起来。(秦牧《眼睛的奥妙》,《艺海拾贝》上海文艺出版社1962) 2.人物描写 大凡写得最好的人物,不是用叙述方法来介绍他的面相和性格;而是写他的声音笑貌,一举一动,使人读完后能够想像出这个人物的形貌。《红楼梦》的人物就是用这个方法来描写的,甚至可以从人物的说话中想像出他(或她)的丰采,辨出是男或是女。中国社会上,新知识分子与旧知识分子,资产阶级与地主,工农与职员,男女老少,各色人等,谈吐的腔调都不一样;闻其声,如见其人;写人物而写到这样境界,才算技巧之上乘。 茅盾《怎样阅读文艺作品》,《茅盾论创作》,上海文艺出版社1980年版) ( 3.环境描写 人物不得不在一定的环境中活动,因此,作品中就必须写到环境。作品中的环境描写,不论社会环境或自然环境,都不是可有可无的装饰品,而是密切地联系着人物的思想和行动。作家常常要从各方面来考虑,在怎样的场合应该有怎样的环境描写。不适当的环境描写会破坏作品的完整性,至少也要破坏作品的气氛。一段风景描写,不论写得如何动人,如果作家只是站在他自己的角度来欣赏,而不是通过人物的眼睛、从人物当时的思想情绪,写出人物对于风景的感受,那就会变成没有意义的点缀。风景(或其他自然现象的)描写,和室内装饰布置的描写,时常被用来加强特定的气氛。而为了达到这目的,有时会觉得正面渲染的方法不如对比的手法能够产生更强烈的效果。烦恼的人恰恰落进作乐的场合,表面上不得不强颜欢笑,心里却加倍痛苦??这在作品中是常常看到的,在生活中也常常看到,不过,由于作家的加工,作品中所表现的,比在生活中所发生的,就要强烈得多。 (茅盾《关于艺术的技巧》,中国青年出版社1959年版) 俄国作家契诃夫说过:如果一个剧本在第一幕的布景里,墙上挂着一柄腰刀,到最后一幕就得让刀子出鞘,要不然,那是柄多余的刀子,一开始就不应该挂上去。对于写作,我以为这是个有益的提示,每一个初学写作者都可以从这里得到很大的启发。 就我接触到的初学写作者的创作来说,下笔时没有扣紧主题,墙上挂着太多始终不见出鞘的刀子,还是一个比较普遍的现象。譬如说写景,有些同志似乎对自然风景有很大的兴趣,即使是一篇短短的速写,也喜欢花呀、月呀、山呀、水呀的写上一大堆;又譬如说人物的行动,也往往来去无常,随兴所之,看不出一点必要的约束。我不是说一切风景和行动,都必须和主题直接结合,我是说这些风景和行动,甚至对故事本身也很少关涉,很少作用。其实风景也罢,行动也罢,只有当它们成为全部情节或者主要场面一个有机的部分,和主题联系与呼应的时候,才能发出耀眼的光彩,收得激动人心的效果。先说写景,《红楼梦》 第三回记黛玉初进贾府,一路雕梁画栋,彩屏玉觚,堂上一几一案,廊边一山一石,都显得精妙绝伦,豪华之至;直到元春归省,大观园建成,更是富丽堂皇,穷奢极欲。小说精心刻画,一笔不苟,通过这些写出了贾府极盛时期的面貌。这是作者卖弄才情,故意表示自己的博物吗,一点也不!这是柄挂在墙上的腰刀,到了抄家以后,这柄刀子终于出鞘了:树倒猢狲散。往日豪华,只换得一片凄凉。草木无情,到此仿佛都在说话,读起来更觉惊心动魄。可见作者的描写景物,预先都有安排,所谓为后文“张本”,紧紧地扣住封建贵族家庭最后终于没落的这个主题。能够说这里有多余的笔墨吗,我看是没有。 „„ 古人所谓笔无虚文,所谓没有闲笔,我看指的就是这个。 (唐弢《为主题服务》,《创作漫谈》,作家出版社1962年版) 5.细节描写 要找到能够表现这个人物性格特征性的细节。什么叫特征性细节,如阿Q,我们马上就想到几个细节,怎么跟赵大爷姓赵,怎么调戏小尼姑,怎么同小D打架,怎么样向假洋鬼子要革命,到死的时候画圆圈还欣赏圆不圆,想到了这些细节,就有了阿Q。假如说这些细节都没有,阿Q就没有了。阿Q的性格其实就是靠这样的细节把他的性格表现出来了。人物的性格就是依靠这些细节突出来的。有些小说从头到尾都是细节,但读了以后却没有印象,因为这些细节,都不是带有特征性的。细节再多,不能表现人物的性格,那也是空的。 (高晓声《漫谈小说创作》,《福建文学》1983年第9期) 学生作文 有这么一对孪生姐妹,长得几乎一模一样?? 一样高矮,一样胖瘦,一样的红苹果似的小脸,一样的亮晶晶的大眼睛。如果你只看一眼,会觉得她俩简直像一个模子刻出来的。但是,你要是和她们多呆一会,你会发现区别她俩的地方。一个的头发似乎特别硬,头上的辫子总是翘着,而另一个的头发却似乎特别软,小辫子柔柔地垂在脑后;一个总爱撅着嘴,似乎老在生气,一个总是咧着嘴,似乎有着说不完的高兴事。难怪她俩的妈妈给她们起名字,一个叫“怨怨”,一个叫“乐乐”呢! ??肖像描写(1990年高考满分作文) 这是一双凝神专注的眼睛。那圆圆的眸子里深藏的是什么,能够折射出如此诱人的光,像浸在水中的一颗乌亮的宝石,那么晶莹;像黑夜中闪烁的一粒珍珠,那么夺目。它是那么黑,似乎包容了世上所有的暗色;它是那么亮,能够折射出人间万物的光芒,以至于竟不易察觉到它的黑。它是一块墨玉,纹理很鲜明,中心是一点,最幽深的一点。周围纤细而浓密的睫毛,偶尔投影在它的正面,宛若微波荡漾的水面上垂下的修长柔美的丝丝柳絮。 ??肖像及细节描写(1991年高考满分作文) “噢,天凉好个秋!”燕子在高枝上低吟着。“哟,小燕子,满有诗意的嘛。你什么时候开始逃亡呀,”麻雀飞过来搭腔。燕子一惊,顿时板起脸来说:“你刚才说什么,”“你没听清吗,逃亡,就是逃避那会让你死亡掉的寒冷。”麻雀怪声怪气地说。“什么逃亡,明年春天我还回来的!”燕子涨红了脸。“是呀,春天到了天气就不再冷了,懦夫当然要回来享受那煦暖的春光了。”“你,你说我是懦夫!”燕子的心被刺痛了,然而她忽地想到什么,这让她冷静了下来,她清了清嗓子,她像自言自语地说:“就算我是,也比有的人让人说成鼠目寸光好,哼!”两只鸟就这样吵着,不多时便各自飞远了。 ??语言描写(1995年高考满分作文) 这是一湖被围绕在连绵起伏的青山怀里的碧水。由于汇集了四周山上流下的小溪,湖水十分明静清澈。湖面呈圆形,仿佛盛满了醇酒的酒杯。湖里的水倒映着青山绿树,倒映着野草野花。白天,湖里拢着蓝天白云;夜晚,湖里映着星星月亮。这是一弯幸福的湖,被四周的山拥着,也拥着四周的山。有时雪花飘飘,带着春的信息落下,湖水立刻无私地用温暖的怀抱把它们融化。四季的景致,太阳起落时的美妙,这一切都让湖水更清澈,更明亮。 ??环境描写(1991年高考满分作文) ,编者说,这几段描写选自高考作文。在考场上能写出这样的描写文字实属不易。第一段是肖像描写, 用比拟手法,妙在突出了特征。第二段也是肖像描写,好处是比较具体,可能从《明湖居听书》和茨威格 《列夫?托尔斯泰》中受到了启发。第三段语言描写用了拟人手法,写出了对话双方的心理。第四段环境 描写,具体细腻,还带着一些诗意。 9 发现幸福(作文) 1 “发现幸福——学习纵向议论”导学案 【学习目标】 1 了解议论文结构的纵向模式。 2 进行纵向层进式思维训练,学会灵活运用纵向模式展开议论。 3 通过写作,认识幸福是什么,学会从生活中发现幸福。 【学习难点】 1 引导学生发现幸福、感受幸福、学会感恩,在写作实践中树立正确的幸福观。 2 学习纵向议论,掌握议论文纵向展开议论的思路结构,提高学生的论证水平。 【学习方法】讨论法 、 讲解法 【课时安排】 一课时 【学习过程】 一、导入新课 以问题“幸福是什么”导入新课。请同学们“各言其志”,引发对课题的兴趣,了解学生对“幸福” 的理解与定义 。 教师总结: 1(幸福是物质与精神的统一 幸福是物质的,抑是精神的,有人以为物质条件优越就能活得幸福,这是把 幸福看成是物质的;有人以为只要精神愉快就幸福,这是把幸福看成是精神的。 其实,幸福是物质与精神的统一。(1)凡人心随境转,舒适的环境是产生幸福 的基础,如事业的成功、家庭的和谐等,这说明幸福需要物质的基础。(2)幸 福由心感受,只有健康的心境,才有幸福可言。心情不好,即使贵为皇帝,拥有 天下的权力、地位和财富,依然会活得痛苦不堪的。 2. 幸福与不幸福 世间由顺境和逆境组成。通常人们以健康为幸运,疾病为不幸;成功为幸运, 失败为不幸;富有为幸运,贫穷为不幸……。生活在顺境中的人,大家都觉得他 很幸运,很幸福;相反,有些人时运不佳,遇到逆境,人们就会认为那是不幸, 是痛苦。然而社会或个人总是存在顺境和逆境,有顺境必然有逆境,这是一种普 遍的现象,人类应该正确的认识它。 顺境与逆境既然是构成社会人生的两个方面,那么它的存在必然是合理的。 (1)我们应该正视顺境与逆境的客观存在。就如白天和黑夜、春夏秋冬的自然 规律一样。(2)在顺境、逆境面前,不应该生起爱嗔之心。 世间的痛苦与快乐是相互依赖的,谁也离不开谁。有些人只要快乐,不要痛 苦;只要顺境,不要逆境。可是假如没有痛苦就没有快乐;没有经历逆境,就无 法认识到顺境的可贵,就像长期享有顺境的人,很难生起幸福感。因此,痛苦使 快乐更快乐,不幸使幸运变得幸福。就如疾病使健康变得快乐,贫穷使富有变得 幸福。 二、解读范文, 提出问题,组织讨论,明确答案: 文中的论点——读书人是幸福人。 第一段:读书人是幸福人。 第二段:阐述阅读能增加人的经历和经验。 第三段:证明读书更能感化与陶冶人的精神。 第四段:进一步指出读书能使人向善避恶,变得情趣高雅,心灵纯净而富有正义感。 第五段:呼应开头,收束全文。 思考—— 本文是如何展开纵向议论的, 三、初步感知(一) 1.纵向议论主要采用层进式结构, 文章各层次之间是: 层层深入、步步推进的, 各层的前后顺序有严格要求,不能随意改动。 2.层进式结构格式: 引论(提出论点和论题) 本论 分论点——是什么, 分论点——为什么, ,, 论 点——怎么办, 结论(照应全文) 四、初步感知(二) 1(基本格式的运用 , 将中心论点进行分解,分成几个分论点,分论点之间的关系是: , 由浅入深、 , 由简单到复杂。 , 过渡关联词语,不仅„„而且„„,、,„„况且,等. 2 .按照,提出问题,分析问题,解决问题,的思路安排论证结构,即围绕中心论点回答三个问题:?是什 么,?为什么,?怎么办。 3. 针对某些不良的现象,分析其危害,挖掘其产生根源,指出解决问题的办法。 摆现象——析危害——挖根源——指办法 4..课堂练习 参考示例:纵向层进式结构的三种基本格式与运用举例 (一)将中心论点进行分解,分成几个分论点,这些分论点之间的关系是由浅入深、由简单到复杂。 层次间可用诸如“不仅……而且……”“……况且”等关联词语过渡,同时又以此反映层次间递进的关系。例 如: 严于解剖自己 1、要不断进步,必须无情地“解剖我自己”。 2、论述如何才能“解剖”好自己。 (1)对自己要有自知之明。(这是“解剖”好自己的前提。不了解“病”在哪里,就无从下刀。) (2) 光有自知之明还不够,还要勇于自我批评。(这是解剖好自己的途径。要敢于下刀,不然,就无 从去“病”。) (3)自我批评的勇气来源于对真理的追求和崇高的信念。(这是解剖好自己的关键。不掌握开刀的规律,刀就开不好,也就难以真正去“病”。) (二)按照“提出问题,分析问题,解决问题”的思路安排论证结构,即围绕中心论点回答三个问题:?是什么,?为什么,?怎么办。 以《要钻牛角尖》为题,编写纵向展开议论的提纲。 结合前面所学习的纵向展开议论的格式和方法,让学生学会分析和运用。 (1)(是什么)什么是“钻牛角尖”:对任何事情,哪怕是极细小的事情,都采取认真钻研的态度。 (2)(为什么)研究学问“钻牛角尖”才会有所成就,进行科研“钻牛角尖”才有发明创造。 (3)(怎么办)“钻牛角尖”要有认真的态度,要有锲而不舍的精神,要掌握科学的方法。 (4)(会怎样)发扬“钻牛角尖”精神会多出成果,多造就人才。 我们再来看一道作文题: 做人和处事,如果凡事追求圆满,事事工于算计,互不相让,人与人之间的关系就会紧张,就会裂变。留一道缝隙,给自己,给他人,给社会留一个可供吻合的人际空间。以“留一道缝隙”为话题,联系社会生活实际,写一篇议论文。 材料告诉我们,在生活中应该留一道缝隙,那么,应该怎么去留缝隙呢,请同学们依此思路用横向结构写出作文的提纲。 可以这样拟定分论点: (1)给自己留一道缝隙,轻松快乐的生活。 )给理想留一道缝隙,坦然面对失去。 (2 (3)给子女留一条缝隙,使自己和孩子获取快乐。 (4)给他人留一道缝隙,建立和谐的人际关系。 我们在这里所使用的提炼分论点的方法叫“条件分析法”。这里的中心论点是结果,而分论点是满足结果的“条件”。此种类型主要从“怎么办”的角度来设立分论点。 任何事物都是由许多要素构成的,都有其许多不同的侧面,揭示这些要素和侧面,也就揭示了整个事物。因此,提炼分论点的方法是有多种的,还有如“因果分析法”、“意义分析法”等,我们可以在实际写作中灵活运用。方法总结如下: 其一是横向比较。在论证中往往少不了横向的比较,或进行不同学科之间的比较,或进行同龄人之间的比较;或进行各国之间对同一个事情采用不同方法得到不同结果的比较„„从而有力地论证作者自己提出的论点。这种讨论问题的角度便于打开论证的思路。 其次是由此及彼。横向比较讲的是同一事物的横向联系,由此及彼是指某一事物与他事物之间的联系,即这一事物的发展或静止对他事物的影响。如教育对人的影响,从而联系到企业,即人对企业的影响,再进一步联系到社会,即企业对社会的影响。这样一联系,你会发现:教育决定了人的素质,人的素质影响到企业的效益,而企业的效益又与社会安定、物质文明不无关系。有了这样的联系,你在讨论问题时,就可以广泛地伸出触角,做一番全面的论证。 其三是正反论证。事物总是一分为二的,因为任何事物都具备正、反两面的因素,当读者完全认识事物的正反面时,就会做出最正确(即作者所主张)的选择。所以写作议论文时,可考虑从正反两面打开思路。 六、归纳总结 文章议论的范畴可分为四个方面: “是什么”,论证性质和类别 ; “为什么”,论证目的和原因 ; “怎么办”,论证方法和途径 ; “会怎样”,论证效能和结果 。 一般来说,一篇文章不会同时论述四个方面的问题,大多是择其一、二来论述。 【作业设置】 课外拓展训练:请以“幸福”为话题写一篇800字的议论文。学生利用已学知识,进行阅读和写作练习 , 让学生将课堂所学运用于实践中。 10 拿来主义 2 《拿来主义》导学案 一、教学目标 1、知识教育目标 (1)理解本文的题旨和拿来主义的基本含义。 (2)掌握本文的论证结构,体会鲁迅杂文的艺术特点。 2、能力训练目标 (1)通过阅读和分析本文,训练学生的逻辑思维能力。 2)通过学习本文的论证方法,训练学生的写作能力 ( (2)通过学习本文形象、含蓄、幽默、犀利的语言,提高学生的语言表达能力。 、情感陶冶目标 3 通过学习本文,加深对祖国改革开放国策的认识和理解,培养学生博大的心胸和远大的政治情怀。 二、教学重点 1、引导学生透过词语的表面,分析其深刻内涵。 2、学习运用生动形象的比喻论证法。 三、教学难点 体会鲁迅先生杂文尖锐犀利、幽默诙谐的语言风格。 四、教学方法 引导探究法 五、教学课时 二课时 第一课时 [导 入] 由于改革开放,国门敞开,大量国外的东西涌入我国。这涌入的有先进的科技,但同时也有腐朽的资本主义生活方式。著名作家冯骥才先生到法国访问,在一次欢迎宴会上,外国记者接二连三地向他提问。其中一位记者问:“尊敬的冯先生,贵国改革开放,学习西方资产阶级的东西,你们就不担心变成资本主义吗,”冯先生回答:“不~人吃了猪肉不会变成猪,吃了牛肉不会变成牛。”他幽默机智的回答,博得满堂喝彩。是的。我们学习资本主义的东西,不会变成资本主义,同样,继承文化遗产时,只要我们坚持正确的原则,就一定能够有助于新事物的发展,这个原则就是“拿来主义”。那么,什么是拿来主义,为什么要实 行拿来主义呢,让我们一起来看看鲁迅先生的答案吧。 [新 课] 一、时代背景 本世纪30年代,国民党反动政府为推行政治上的卖国主义政策,在文化政策及其活动上,主张“全盘西化”,贩卖民族虚无主义的文化观点,极力摧残民族文化。而在当时的左翼文化战线的内部,在“文艺大众化”问题的讨论中,一些受极“左”思潮影响的同志,又把“采用旧形式”视为“机会主义”、“类乎投降”,也不能正确对待民族文化遗产。针对这些情况,鲁迅先生以马列主义学说为指导,凭借自己丰富的知识和建设新文化过程中积累的经验,写下了这篇著名的杂文,回答了如何对待外国文化遗产(也包括民族文化遗产)的立场、态度和方法问题,并富有创新意义地命名为“拿来主义”。这一主张后来成为中国无产阶级对待文化遗产的代名词,普遍采用至今。 [布置阅读]要求学生默读全文,找出生字词,并理出文章思路 二、阅读: 1、正音、解词: [课堂练习]学生上黑板注音,解词,教师正音,评讲 自诩( ): 残羹( )冷炙( ):。 冠冕( ): 脑髓( ): 孱头( ): 蹩( )进: 2、本文思路: [引导思考]在阅读的基础上要求学生思考本文的思路 [提 问]下面有几种有关段落的划分法,你认为哪种最恰当,为什么, (1)、分为两部分: 第一段(1,4):破“送去主义”。 第二段(5,10):立“拿来主义”。 (2)、分为三部分: 第一段(1,5):提出问题,主张“拿来主义”。 第二段(6,9):分析问题,阐明什么是“拿来主义”。 第三段(10):解决问题,总结“拿来主义”。 (3)、分为四部分: 第一段(1,2):开篇扣题(没有人说道“拿来”) 第二段(3,5):否定“送去主义”,主张“拿来主义”。 第三段(6,9):正面分析“拿来主义”(破、立) 第四段(10):总结“拿来主义”的作法、结果、前提和意义。 [组织讨论]教师组织学生讨论以下问题,并巡回指导 (1)、文章前半部分重在批判“送去主义”,为什么要从“闭关主义”谈起, (2)、“送去主义”的表现有哪些,其实质是什么,为什么只把锋芒指向学艺方面的几件小事,而不去揭露出卖领土和资源的重大罪行呢, (3)、第3段用了什么论述方法, (4)、“抛来”与“抛给”两个概念有何不同,为什么又可称为“送来”呢, (5)、全文论述的是拿来主义,却用了五分之二的篇幅批判送去主义,这种破字当头的结构有什么好处, (6)、有关重要词语、句子的咀嚼(即捧、挂等) [小 结]本节课理出了文章的思路,对文章的前半部分进行了分析,提出了五个问题,要求大家重点掌握其中的第五个问题。 [布置作业]希望同学们课间预习文章的后半部分。 第 二 课 时 [复习导入]作者在第一部分中批判了哪几种主义,为什么, [新 课] 一、课文理解: [提 问]1:8——9段论证“怎样‘拿来’”。其中,“大宅子”比喻什么,对待“大宅子”的态度和方法,文章摆出了哪几种, [提 问]2:联系“大宅子”这个比喻,说说前三种态度和方法究竟分别是什么意思。 [提 问]3:第9段借助一系列生动贴切的比喻,对“拿来主义占有,挑选”的态度和方法做了形象的阐述。用平实的语言,该怎么表达, [提 问]4:拿来主义究竟怎么“占有,挑选”,根据上面的分析,用精练的语言加以概括。 [提 问]5:模仿归纳1——7段论证思路的方法,归纳8——9段的论证思路。 [练 习] 根据对上文的理解填写下面括号里的内容 ( ) 错误的态度 ( ) ( ) 拿来主义 ( ) 正确的态度 ( ) ( ) ( ) 二、艺术特点: 1、作者运用比喻论证的方法,使说理清楚明白,生动、形象。 2、先破后立,立中有破,破立结合,鲜明对比的结构方式。 3、 犀利的语言风格,高超的幽默与讽刺的艺术才能。 [小 结]引导学生小结本课内容,并说明重点 三、当堂训练 他占有,挑选。看见鱼翅,并不就__________在路上以显其“平民化”,只要有养料,也和朋友们像萝卜白菜一样的吃掉,只不用它来宴大宾;看见鸦片,也不当众_________在茅厕里,以见其彻底革命,只送到药房里去,以供治病之用,却不弄“出售存膏,售完即止”的玄虚。只有烟枪和烟灯,虽然形式和印度,波斯,阿剌伯的烟具都不同,确可以算是一中国粹,倘使背着周游世界,一定会有人看,但我想,除了___________一点进博物馆之外,其余的是大可以毁掉的了。还有一群姨太太,也大以请她们各自走散为是,要不然,“拿来主义”怕未免有些危机。 总之,我们要拿来。我们要或使用,或存放,或毁灭。那么,主人是新主人,宅子也就会成为新宅子。然而首先有这人沉着,勇猛,有辨别,不自私。没有拿来的,人不能自成为新人,没有拿来的,文艺不能自成为新文艺。 1、与“他占有,挑选”这句话意思相近的一句是( ) A(我们要运用脑髓,放出眼光,自己来~ B.我们没有人根据了“礼尚往来”的仪节,说道:拿来~ C.自己不去,别人也不许来。 D.没有拿来的,文艺不能自成为新文艺。 2、依次填入文中横线上最准确的一组词语是( )。 A.摔 抛 送 B.抛 摔 送 C.抛 送 摔 D.送 抛 摔 3、对“其余的是”和“各自走散为是”中两个“是”的理解正确的一项是( ) A.两个“是”不一样,前者起强调作用,后者是“正确”的意思。 B.两个“是”不一样,前者有分类作用,后者表示跟“非”相对的意思。 C.两个“是”不一样,前者表示存在的意思,后者却表示一个判断。 D.两个“是”不一样,前者含有“凡是”的意思,后者表示“适合”的意思。 4、第一段文字中最主要的论证方法是( ) A.对比法 B.例证法 C.喻证法 D.引证法 四、拓展阅读 创新必须择旧 ??读《拿来主义》札记 为什么创新必须择旧??择取中国的和外国的旧文化, 在这个问题上,鲁迅曾引过卢那察尔斯基的一些话,例如仍要保存农民固有的美术,怕军人的泥靴踏烂了皇宫的地毯等。用鲁迅自己的话便是:“新的阶级及其文化,并非突然从天而降,大抵是发达于对于旧支配者及其文化的反抗中,亦即发达于和旧者的对立中,所以新文化仍然有所承传,于旧文化仍然有所择取。”(《集外集拾遗?〈浮士德与城〉后记》)旧文学衰颓,来了一个新的转变,这转变往往得力于摄取民间文学或外国文学。欧洲的印象派,是吸取了从中国和日本传去的画的养料而形成的。文学遗产中一切进步的,合理的,美的东西, 都仍能对新阶级的文学,无论在思想方面还是艺术方面提供有益的成分。为什么历史悠久、文化遗产丰富会成为一个国家创造新文化的极为有利的条件,就因为可以从旧文化中择用的养料多。鲁迅后期杂文是无产阶级文学的瑰宝,它正是从对地主、资产阶级旧文化的斗争中产生的,不能设想,如果没有择取中国的旧文化,会产生出鲁迅的这种锋利无比的杂文来。 鲁迅认为避忌旧文化,乃是衰病无力,缺乏自信的表现;壮健,有魄力,富于自信的人决不会这样。“无论从那里来的,只要是食物,壮健者大抵就无需思索,承认是吃的东西。惟有衰病的,却总常想到害胃,伤身,特有许多禁条,许多避忌;还有一大套比较利害而终于不得要领的理由,例如吃固无妨,而不吃尤稳,食之或当有益,然究以不吃为宜云云之类。”(《坟?看镜有感》)像这类人物,如果再不振作,就会更加衰弱下去,因为终日战战兢兢,先已丧失活气了。“我们吃东西,吃就吃,若是左思右想,吃牛肉怕不消化,喝茶时又要怀疑,那就不行了,??老年人才如此。有力量,有自信力的人是不至于此的。”(《关于知识阶级》)如果知识阶级如此胆小,一听到俄罗斯,一看到俄国的小说,就怕得发抖,对西洋文明也唯恐受害,一动也不敢动,怎样能进步呢,将来必定要灭亡。他说汉唐虽有边患,魄力究竟雄大,人民具有不至于为异族奴隶的自信心,凡取用外来事物时,自由驱使,绝不介怀。而一到衰弊陵夷的时刻,神经就衰弱过敏,对外国东西,便推拒,惶恐,退缩,逃避。(参见《坟?看镜有感》)我们究竟是甘心当弱者,还是应该努力做强者, 他认为文艺上的创新必须择旧,一味害怕是决不行的。“要进步或不退步,总须时时自出新裁,至少也必取材异域,倘若各种顾忌,各种小心,各种唠叨,这么做既违了祖宗,那么做又像了夷狄,终生惴惴如在薄冰上,发抖尚且来不及,怎么会做出好东西来。”(《坟?看镜有感》)十年后他在论新木刻时,又指出有两条路,一条是“采用外国的良规,加以发挥,使我们的作品更加丰满”;另一条是“择取中国的遗产,融合新机,使将来的作品别开生面”。(《且介亭杂文?〈木刻纪程〉小引》)他认为我们对旧文化,无论是中国遗产中的精华还是外国遗产中的良规,实在都还知道得太少,吸收得太少。只要是优点,“即使那老师是我们的仇敌罢,我们也应该向他学习”(《且介亭杂文?从孩子的照相说起》)。 他说:“我已经确切的相信:将来的光明,必将证明我们不但是文艺上的遗产的保存者,而且也是开拓者和建设者。”(《集外集拾遗?〈引玉集〉后记》)你要在文艺上有新的建树吗,那就要保存遗产。保存当然不同于保古,保存下来也不是模仿和照搬,而只能是为了择取优点。如果只为复古、保古而保存,由于大家都反对复古、保古,遗产就保存不住。只有抱着革新的目的,认识到了择旧的重要作用,遗产才会得到大家的重视与爱护,所以真正的革命者才是最理想的遗产保存者,而遗产中的精华,亦只有在革命者手里才能得到开拓,并成为建设新文化的不可缺少的材料。难道我们还能把保存(择取)和开拓、建设割裂了来理解, 五、拓展训练 针对青年人中的“哈韩”和“哈日”现象,发表自己的看法,写一段200字左右的文章。 《拿来主义》答案 第一课时 1、正音解词 自诩(xu ):自我夸耀。诩,说大话,夸耀。 残羹(geng)冷炙(zhi ):吃剩的饭菜,借指权贵的施舍。 冠冕(mian):很体面,有气派。冕,古代帝王的礼帽。 脑髓(shui):脑筋。 孱头(can):懦弱无能的人。 蹩(bie)进:躲躲闪闪在走进。 2、本文思路 [引导归纳] 这三种意见从思路上看都符合逻辑,都能理清文章的条理。因此,三种分法都有是可以的。但第一种意见强调了“破”,更体现了文章的针对性,且更简洁,所以一般都采用第一种分法。 [组织讨论] (1):国民党的“送去主义”与清王朝的“闭关主义”有着内在的历史的联系,而且二者在本质上是一致的。我们不妨先看清王朝的“闭关主义”。何为之,见注释。当时清政府以“国情特别”等为借口,盲目排外,与世隔绝,“自己不去,别人也不许来”。结果怎样,使古老的中国处于腐败落后的境地,最终在别人的枪炮面前,不堪一击,“碰了一串钉子”。 再看“送去主义”,“到现在”,自然是国民党统治时期,“闭关自守”的“大门”早就没有了,“守”也失去了立足之地,于是由“闭关主义”走向另一个极端“送去主义”。从历史上的“闭关主义”起笔,意在说明:1)二者是血脉相承、衣钵想继的;2)都是腐朽、落后、反动的本质的反映。(板书) (2):老蒋的送去主义表现在很多方面,如27年其上台后,不断出卖主权;“九?一八”事变后,为了投靠、讨好美英帝国主义,更是拱手相送国土,由美英操纵着中国经济命脉,控制中国的军事力量,这些都是“送—”的具体明证。但是作者没有举这方面的事例,而是 列举了学艺方面的三个事例。 这是因为本文论述的中心是如何对待文化遗产问题,所以用“单是”一词将其他尽人皆知的事实一笔推开,从而将批判的重点更集中,这样选材才更准确,与中心一致。列举的是:送古董、送画、送活人。要说明的是,送这些东西,不一定得扣上卖国的帽子,但这决不是正常的文化交流,而是媚外求荣。这里也没有非议梅兰芳的意思,而是斥责国民党利用梅去苏联一事欺世惑众、向西方献媚取宠。这里,我想起一副对联。这是甲午战败后,李鸿章赴日本求和时与日本外相伊藤博文所对下的,上联是伊的:“朝无相,边无将,不得已,玉帛相将,将来怎样,”下联是李的:“天难度,地难量,这才是,帝王度量,量亦无妨。”这可以作为“送去主义”的一个绝妙的注释了。作者在列举这几个事例时,字里行间充满了辛辣的讽刺、无情的嘲弄和深刻的批判。 (3):第3段是难点,它的论证过程比较复杂。本段用了类比的方法。请将类比的内容填上: 尼采:自诩是太阳 中国:—— 光热无穷 —— 只是给予,不想取得 —— 他发疯而死 —— 结论:自诩地大物博;掘起地下的煤就足够全世界几百年之用;只送去,不拿来;子孙沦为乞丐(亡国灭种)。(一、二没有现成的句子,但自己可以概括出来)“送去主义”的危害性不是很可怕吗, (4):几个概念的区别。 “抛来”是随意的,“抛给”是有意的,“送来”是特意的。 也可以理解为 “抛来”是受动的,“抛给”是主动的,“送来”是有目的的。 那么,我们再来看看外国有目的地送来了什么,鸦片、废枪炮、香粉,电影、小东西。 帝国主义者送来这些东西的目的是什么, 毒害中国人民。 欺骗中国人民。 用鸦片来换取中国的银两。(板书:危害:亡国、贻害子孙) 因此,这种送的实质是什么,(板书: 毒害、 欺骗、 侵略、 掠夺。) (5):破字当头的好处使论述有历史的深度,它是在总结近百年的历史教训的基础上立论的。作者从广阔的背景上,点明了从闭关主义到送去主义,是从一个极端到另有个极端,反动落后的本质一脉相传。清政府,搞闭关主义,落后挨打;蒋介石,搞送去主义,丧权辱国。出路在哪里,只要实行拿来主义~这种立论,站得高,看得深,有历史深度。2)从对比中分析问题,有说服力。3)先破后立,相得益彰。前者破得越彻底,后者立得越有力;送去主义的危害揭露得越深刻,实行拿来主义的理由越充分。因此,从揭批入手,再转入正面论述,显得水到渠成,立论更鲜明有力。 (6)、重要词句的理解。(略)幽默、讽刺,是鲁迅杂文的语言风格。“嬉笑怒骂皆文章”。这是敏锐、机智和含蓄、委婉的结合物,显示了作者从容不迫、游刃有余的风度。 第二课时 课文理解 1、(1)“大宅子”比喻外国文化遗产。?对待“大宅子”的态度和方法有:?孱头徘徊不敢走进门 ?昏蛋放一把火烧光 ?废物欣欣然接受一切 ?拿来主义占有,挑选 2、前三种态度和方法的意思分别是:懦弱无能、害怕继承、拒绝借鉴的逃避主义;盲目排斥的虚无主义;崇洋媚外、主张“全盘西化”的投降主义。 3、有益无害的(即“鱼翅”),要“拿来”,而且“使用”;既有毒素又有用处的(即“鸦片”),要吸取、使用它有用处的方面,清除它有害的方面;人民根本不需要的(即“烟灯”“烟枪”和“姨太太”),原则上加以“毁灭”(只留少许送博物馆)。 4、取其精华,弃其糟粕,即批判地继承。这种态度和方法同样适合于本国的文化遗产。 5、或逃避主义,或虚无主义,或投降主义,或批判继承 逃避主义是孱头 虚无主义是昏蛋 投降主义更是废物 所以,对文化遗产要批判地继承 艺术特点: 、作者运用比喻论证的方法,使说理清楚明白,生动、形象。 1 2、先破后立,立中有破,破立结合,鲜明对比的结构方式。 3、 犀利的语言风格,高超的幽默与讽刺的艺术才能。 11 父母与孩子之间的爱 1 《父母与孩子之间的爱》导学案 学习目标: 1、了解弗罗姆及其名作《爱的艺术》。 2、深入把握文中对“爱”的剖析,体会其蕴含的深远意义。 3、理清文章结构脉络及行文特点,体会文章思路清晰、结构严谨的特点。 4、领悟父爱与母爱的博大、无私与精深,树立正确的爱心观,培养自己的博爱、仁义情感。 学习重点:目标3、4 学习难点:目标2 学法指导:阅读法、合作探究法 教学过程: 一、导入新课 当抢救人员发现她的时候,她已经死了,是被垮塌下来的房子压死的,透过那一堆废墟的间隙可以看到她死亡的姿势,双膝跪着,整个上身向前匍匐着,双手扶着地支撑着身体,有些像古人行跪拜礼,只是身体被压得变形了,看上去有些诡异。 经过一番努力,人们小心地把挡着她的废墟清理开,在她的身体下面躺着她的孩子,包在一个红色带黄花的小被子里,大概有三四个月大,因为母亲身体庇护着,他毫发未伤,抱出来的时候,他还安静地睡着,他熟睡的脸让所有在场的人感到很温暖。 随行的医生过来解开被子准备做些检查,发现有一部手机塞在被子里,医生下意识地看了下手机屏幕,发现屏幕上是一条已经写好的短信“亲爱的宝贝,如果你能活着,一定要记住我爱你”,看惯了生离死别的医生却在这一刻落泪了,手机传递着,每个看到短信的人都落泪了。感动有让我们感受了母爱光华的绚烂~ 那么我们今天就站在另外一个角度,站在理性的高度来重新审视这份爱——父母与孩子之间的爱。 这篇文章是从美国著名的心理学家、社会学家弗罗姆《爱的艺术》中节选出来的,在学新课文以前我给大家介绍一下《爱的艺术》这本书的主要观点和内容:它阐释了爱并不是一 种与人的成熟程度无关的感情,而是一个能力的问题,是一门通过训练自己的纪律、集中和耐心学到手的行为艺术。 二、知识积累,预习检查 1、给下面黑体字注音 罪孽 秩序 刹那 咄咄逼人 惶恐 休戚相关 2解释下面的词语 爱情: 节外生枝: 归根结蒂 : 咄咄逼人: 3、找学生介绍弗罗姆及其名作《爱的艺术》。 三、诵读文本,感知文脉 1、在预习基础上,用5分钟时间整体把握全文,自己尝试编写全文结构提纲。 第一部分(第 , 段): 第二部分(第 , 段): 现象: 原因: 第三部分(第 , 段): 2.人在生长过程中各个阶段的情感变化特征表(大屏幕显示) 年龄段 特征 刚出生阶段(婴期) 8岁以下(幼儿期) 8—10岁(童年期) 少年时期(成长期) 成熟时期(成熟期) 3、总结: 通过以上的阅读归纳,我们可以很清楚地知 道: 四、问题探究 1.作者在本文中说,母爱是无条件的。真的如此吗, 2.文章中谈到两种爱的原则:“我爱,因为我被人爱”和“我被人爱,因为我爱人”;“我爱你,因为我需要你”和“我需要你,因为我爱你”。这两种原则区别在哪里, 3、作者认为母爱和父爱在性质上有什么根本区别,各有什么积极面和消极面,联系实际对作者的看法做一番评析。 4、作者认为,成熟的人不依赖父母提供的世界,而是自己心中拥有这两个世界。说一说,父亲、母亲这两个世界给你哪些影响,你怎样努力在心中拥有这两个世界, 5、本文是一篇译作,在遣词造句和修辞方面,与纯粹的汉语写作有所不同。你认为哪些语句反映了译文的特点,可结合前几册学过的译文跟同学一起讨论。 五、课外作业 阅读:朱自清《背影》节选,写一篇不少于300字的有关父爱的感想。 我与父亲不相见已二年余了,我最不能忘记的是他的背影。那年冬天,祖母死了,父亲的差使也交卸了,正是祸不单行的日子,我从北京到徐州,打算跟着父亲奔丧回家。到徐州见着父亲,看见满院狼藉的东西,又想起祖母,不禁簌簌地流下眼泪。父亲说,“事已如此,不必难过,好在天无绝人之路~”回家变卖典质,父亲还了亏空;又借钱办了丧事。这些日 子,家中光景很是惨淡,一半为了丧事,一半为了父亲赋闲。丧事完毕,父亲要到南京谋事,我也要回北京念书,我们便同行。 到南京时,有朋友约去游逛,勾留了一日;第二日上午便须渡江到浦口,下午上车北去。父亲因为事忙,本已说定不送我,叫旅馆里一个熟识的茶房陪我同去。他再三嘱咐茶房,甚是仔细。但他终于不放心,怕茶房不妥帖;颇踌躇了一会。其实我那年已二十岁,北京已来往过两三次,是没有甚么要紧的了。他踌躇了一会,终于决定还是自己送我去。我两三回劝他不必去;他只说,“不要紧,他们去不好~” 我们过了江,进了车站。我买票,他忙着照看行李。行李太多了,得向脚夫行些小费,才可过去。他便又忙着和他们讲价钱。我那时真是聪明过分,总觉他说话不大漂亮,非自己插嘴不可。但他终于讲定了价钱;就送我上车。他给我拣定了靠车门的一张椅子;我将他给我做的紫毛大衣铺好坐位。他嘱我路上小心,夜里警醒些,不要受凉。又嘱托茶房好好照应我。我心里暗笑他的迂;他们只认得钱,托他们直是白托~而且我这样大年纪的人,难道还不能料理自己么,唉,我现在想想,那时真是太聪明了~ 我说道,“爸爸,你走吧。”他望车外看了看,说,“我买几个橘子去。你就在此地,不要走动。”我看那边月台的栅栏外有几个卖东西的等着顾客。走到那边月台,须穿过铁道,须跳下去又爬上去。父亲是一个胖子,走过去自然要费事些。我本来要去的,他不肯,只好让他去。我看见他戴着黑布小帽,穿着黑布大马褂,深青布棉袍,蹒跚地走到铁道边,慢慢探身下去,尚不大难。可是他穿过铁道,要爬上那边月台,就不容易了。他用两手攀着上面,两脚再向上缩;他肥胖的身子向左微倾,显出努力的样子。这时我看见他的背影,我的泪很快地流下来了。我赶紧拭干了泪,怕他看见,也怕别人看见。我再向外看时,他已抱了朱红的橘子望回走了。过铁道时,他先将橘子散放在地上,自己慢慢爬下,再抱起橘子走。到这边时,我赶紧去搀他。他和我走到车上,将橘子一股脑儿放在我的皮大衣上。于是扑扑衣上的泥土,心里很轻松似的,过一会说,“我走了;到那边来信~”我望着他走出去。他走了几步,回过头看见我,说,“进去吧,里边没人。”等他的背影混入来来往往的人里,再找不着了,我便进来坐下,我的眼泪又来了。 近几年来,父亲和我都是东奔西走,家中光景是一日不如一日。他少年出外谋生,独力支持,做了许多大事。那知老境却如此颓唐~他触目伤怀,自然情不能自已。情郁于中,自然要发之于外;家庭琐屑便往往触他之怒。他待我渐渐不同往日。但最近两年的不见,他终于忘却我的不好,只是惦记着我,惦记着我的儿子。我北来后,他写了一信给我,信中说道,“我身体平安,惟膀子疼痛利害,举箸提笔,诸多不便,大约大去之期不远矣。”我读到此处,在晶莹的泪光中,又看见那肥胖的,青布棉袍,黑布马褂的背影。唉~我不知何时再能与他相见~ 《父母与孩子之间的爱》导学案答案 二、知识积累,预习检查 1、字音 niâ zhì chà duî huáng qī 2、词语 爱情:(狭义)男女之间相互爱恋的感情(广义)人与人之间相互爱恋的感情(同性恋等) 节外生枝:本不应该生枝的地方生枝。比喻在原有问题之外又岔出了新问题。多指故意设置障碍,使问题不能顺利解决。 归根结蒂 :蒂,花或瓜果连接枝茎的部分。归结到根本上。 咄咄逼人:咄咄:使人惊奇的声音。形容气势汹汹,盛气凌人,使人难堪。也指形势发展迅速,给人压力。 三、诵读课文,感知文脉 1.明确如下: 第一部分(第1,3段):自恋阶段 被人爱阶段 成长过程 被人爱—爱别人—创造爱 第二部分(第4,7段):情感对象 现象:先理解母爱,再认识父爱 原因:母爱无条件,父爱有条件 第三部分(第8,10段): 总结全文,构成自己健康而成熟的灵魂 2.人在生长过程中各个阶段的情感变化特征表(大屏幕显示) 年龄段 特征 刚出生阶段(婴儿期) 无爱的意识,不具备感受爱的能力 8岁以下(幼儿期) 有被爱的意识,被爱的反应是感谢和高兴 8—10岁(童年期) 有爱的意识,通过自己的努力去唤起爱 少年时期(成长期) 开始呼唤爱(克服自我中心阶段)和学习 爱别人“我爱你,应为我需要你” 成熟时期(成熟期) 爱别人,创造爱“我需要你,因为我爱 你” 3、总结 通过以上的阅读归纳,我们可以很清楚地知道:爱不是与生俱来的~ 爱是一个逐步发展的过程:无爱---被爱---有爱---创造爱 四、问题探究 1.作者在本文中说,母爱是无条件的。真的如此吗, 答:父母与孩子之间的爱作者只是看到了问题的一个方面。不错,相对于其他几种性质的爱而言,它是最少明确意识到爱的动机和目的的一种形式。母爱很少有需要报偿的明显动机。但是,弗罗姆忽视了另一个方面,即母亲之所以爱孩子,是因为孩子给她以慰藉和希望,这种慰藉和希望在很多场合是潜意识的,或是下意识的。母亲并不一定明确意识到这一条件性,但是没有明确意识到并不等于客观上不存在。无论多么伟大的母亲,都会因孩子的离去而感到空虚,也都会有“望子成龙”的思想意识。这种空虚正是由于她期待孩子的慰藉造的, 这种“望子成龙”的意识正是母亲寄希望于孩子的反映。 2.文章中谈到两种爱的原则:“我爱,因为我被人爱”和“我被人爱,因为我爱人”; “我爱你,因为我需要你”和“我需要你,因为我爱你”。这两种原则区别在哪里, 答:按照作者的观点,爱自己的价值,在自我中是不能实现的,只有在他人和社会的存在中才能实现。“他人的要求同自己的要求同等重要??事实上也许更为重要。给比得更能使自己满足,更能使自己快乐,爱比被爱更重要。”后一种爱是成熟的爱,从爱他人中实现爱自己。前一种爱是不成熟的幼稚的爱,因为受到了别人的爱,需要别人,才去爱别人。 3、作者认为母爱和父爱在性质上有什么根本区别,各有什么积极面和消极面,联系实际对作者的看法做一番评析。 答:母亲代表自然世界,母亲是我们的故乡,是大自然、大地和海洋。父亲代表思想世界,代表法律、秩序和纪律等事物的世界。母亲从身体和心理上给孩子以爱和关怀,给孩子生活上的安全感;父亲从孩子六岁左右开始,向孩子指出通往世界之路,树立孩子挑战生活的自信心。母爱从本质上说是无条件的,而父爱是有条件的。父爱的原则是:“我爱你, 因为你符合我的要求,因为你履行你的职责,因为你同我相像。” 母爱的积极面是,它是世界上最伟大的爱,是每个人内心深处最为渴求的。消极面是,母爱不能用努力去换取,而且根本无法赢得。父爱的积极面是,父爱可以通过自己的努力去获得,可以受自己的控制和支配。消极面是,父爱必须靠努力才能得到,在辜负父亲期望的 情况下,可能会失去父爱。因为你符合我的要求,因为你履行你的职责,因为你同我相像。” 学生可以各抒己见,只要言之成理,即给予适当肯定。下列几点可供参考:1.母爱也是有一定条件的,理由见“课文研讨”。2.父爱和母爱一样,也有一定的无私性。3.父爱和母爱的差别除了上文提到的外,还有一些:母爱比较细腻,父爱比较粗犷;母爱比较着眼于眼前,父爱比较着重于未来;母爱以情感来感染和引导孩子,父爱以理智和行动来教育孩子。 、作者认为,成熟的人不依赖父母提供的世界,而是自己心中拥有这两个世界。说一4 说,父亲、母亲这两个世界给你哪些影响,你怎样努力在心中拥有这两个世界, 参考答案:略。 5、本文是一篇译作,在遣词造句和修辞方面,与纯粹的汉语写作有所不同。你认为哪 些语句反映了译文的特点,可结合前几册学过的译文跟同学一起讨论。 译文在遣词造句和修辞方面,与纯粹的汉语写作有不少不同。例如,译文中常有一种复句,组织严密、信息量大,在汉语中极为少见。最典型的是《在马克思墓前的讲话》中的一个长句子:“正像达尔文发现有机界的发展规律一样,马克思发现了人类历史的发展规律,即历来为繁芜丛杂的意识形态所掩盖着的一个简单事实:人们首先必须吃、喝、住、穿,然后才能从事政治、科学、艺术、宗教等等;所以,直接的物质的生活资料的生产,从而一个民族或一个时代的一定的经济发展阶段,便构成基础,人们的国家设施、法的观点、艺术以至宗教观念,就是从这个基础上发展起来的,因而,也必须由这个基础来解释,而不是像过去那样做得相反。”又如,译文中有一种插入句,汉语很少用。《在马克思墓前的讲话》中有这样一句:“任何一门理论科学中的每一个新发现??它的实际应用也许还根本无法预见——都使马克思感到衷心喜悦,而当他看到那种对工业、对一般历史发展立即产生革命性影响的发现的时候,他的喜悦就非同寻常了。”本文没有像《在马克思墓前的讲话》中那样典 型的例子,但有一些近似的例子可供揣摩。 12 短文三篇 1 《短文三篇》导学案 【学习目标】 1(了解文章丰富的文化内涵,提高学生的文化素养。 2(注意学习课文语言,摘抄名言警句。 3(要在阅读过程注重反思,探究论著中的疑点和难点,敢于提出自己的见解,并乐于和他人交流切磋,共同提高。 【学习重难点】 把握论著的主要观点和基本倾向,了解用以支撑观点的关键材料,要在阅读过程注重反思,探究论著中的疑点和难点,敢于提出自己的见解,并乐于和他人交流切磋,共同提高。 【学法指导】 诵读法、质疑法、探究法 【知识链接】 作者简介: 1592),法国文艺复兴后重要的人文主义作家。在16世纪的作 蒙田(1533- 家中,很少有人像蒙田那样受到现代人的崇敬和接受。他是启蒙运动以前法国的一位知识权威和批评家,是一位人类感情的冷峻的观察家,亦是对各民族文化,特别是西方文化进行冷静研究的学者。从他的思想和感情来看,人们似乎可以把他看成是在他那个时代出现的一位现代人。蒙田出身贵族,祖上是波尔多人,他早年学习拉丁文,在波尔多市念完中学后,在相当长的时期内深居简出,闭门读书思考。后来,他在政府部门任职,成为波尔多市议员,并两度被选为波尔多市市长。1562年他皈依天主教;1572年在他父亲死后才开始撰写《随笔集》。 帕斯卡尔(1623-1662),法国数学家、物理学家、思想家。 罗伯特?富尔格姆,美国当代作家、哲学家。在他的经历中,他当过干活的牛仔、民歌手、IBM公司推销员、专业画家、教区牧师、酒吧调酒师、绘画教师和父亲。他和他的妻子住在西雅图的一座船屋里。主要著作《我一躺倒,身下就起火》。 【学习过程】 1、品读课文,要求读准字音,看准字形。 赋予( ) 厚赐( ) 不堪( ) 惋惜( ) 弥补( ) 苇草( ) 培养皿( ) 规矩( ) 推衍( ) 聚苯乙烯( ) 2、熟语积累。 风和日丽: 垂暮之年: 稍纵即逝: 3、读完《热爱生命》,你有什么整体感受, 4、品味下列语句,说出你的理解。 (1)"我们的生命受到大自然的厚赐,它是优越无比的。" (2)"生之本质在于死。" (3)"只有乐于生的人才能真正不感到死之苦恼。" (4)"生活乐趣的大小是随着我们对生活的关心程度而定的。" (5)"剩下的生命愈是短暂,我愈要使之过得丰盈充实。" 5、读完《人是一根能思想的苇草》,你有什么整体感受, 6、问题探讨 (1)文章说,"我们全部的尊严就在于思想。正是由于它而不是由于我们所 无法填充的空间和时间,我们才必须提高自己"。这句话你是如何理解的, (2)文章说:"我们要努力好好地思想'。这就是道德的原则。"试对这句话做一点解释说明。 7、读完《信条》,你有什么整体感受, 8、问题探讨 【提问】作者列出这些最基本的信条有什么意义, 9、结束语: 当代社会节奏越来越快,人们在固定的生活轨道中疲于奔命。从小当学生时忙于考试,快于找工作,成家后忙于生计,生子后又为下一代操劳。在无尽的繁忙中,人的灵性被湮灭,快乐被剥夺,只剩下忙碌与疲惫。那皎洁的月亮,在古人眼中蕴含了情、蕴含了思、蕴含了憧憬,但在当代人眼中,却只是一个布满了环形山的卫星。 世界并不是完满的,连美神维纳斯都是断臂的,但那种残缺的美让人如痴如醉。如果我们对生命能够多一些认识,对生命的快乐有一种切身的体会,就会更远离功名利禄,更远离凡俗,更去掉躁动不安的心理,而是在生命的从容中感受到生命的真正意义。即陶渊明那种"采菊东篱下,悠然见南山"的境界。 要达到这种高妙超然的乐和境界,需要人们一生去追求。 愿我们每个个人都能珍惜生命,热爱生命,在生命的每一天都能焕发"丰盈饱满"的气象~ 【布置作业】 完成"研讨与练习一、二"。 答案: 1、品读课文,要求读准字音,看准字形。 fù cì kān wǎn mí wěi mǐn ju yǎn xī 2、熟语积累。 风和日丽:形容天气晴朗暖和(多用于春天)。 垂暮之年:(书)老年。 稍纵即逝:稍微一放松就溜过去了,形容时间、机会等极易失去。 3、读完《热爱生命》,你有什么整体感受, 奥斯特洛夫斯基有句名言:"人最宝贵的是生命,生命对于每个人只有一次„„"这句名言表达了强烈的生命意识,愿每个人都珍视生命、热爱生命。从蒙田的《热爱生命》里我们领悟到生命的意义,认识生命的本真。 4、品味下列语句,说出你的理解。 (1)"我们的生命受到大自然的厚赐,它是优越无比的。" (生命是经过数十亿年的时光演化而来的。是自然的哺育,也是自然的杰作。无可比拟。) (2)"生之本质在于死。" (有生就有死,没有死也就没有生。死亡是生命的另一种形式)。 (3)"只有乐于生的人才能真正不感到死之苦恼。" (因为好好享受过生活,对死亡就不感到遗憾。) (4)"生活乐趣的大小是随着我们对生活的关心程度而定的。" (关心生活,才能感受到生活的乐趣,更好地享受生活。) (5)"剩下的生命愈是短暂,我愈要使之过得丰盈充实。" (用"丰盈充实"的生活使生命相对延长。) 5、读完《人是一根能思想的苇草》,你有什么整体感受, 这是作者在瞬间进发出的思想火花。 文章的主要内容可以归纳为以下几点:第一,从生理的层面来讲,人只不过是一根苇草,是自然界最脆弱的东西,一口气、一滴水就能致人于死命。第二,人的全部尊严就在于思想。因为能思想,区别于顽石或者牲畜;因为能思想,比能致他于死命的东西高贵得多,比囊括了人并吞没了人的宇宙高贵得多;因为能思想,人囊括了宇宙。第三,人要努力好好地思想,这是道德的原则。人不能求之于空间,占有多少土地都不会有用。总之,人的尊严在于能思想,因此人要"努力好好地思想"。 6、问题探讨 (1)文章说,"我们全部的尊严就在于思想。正是由于它而不是由于我们所无法填充的空间和时间,我们才必须提高自己"。这句话你是如何理解的, 【明确】人有一种希望,就是填充空间和时间,即希望自己占有更大的空间和时间。但与浩瀚的宇宙相比,人占有多少空间都没有用;人无法避免死,因此也占不了多长时间。人的尊严不在于填充空间和时间,而在于思想。思想有深浅高下之分,高明的、有深度的思想才能使人获得力量,体现出人发展自己、完善自己的努力,显示出人的独特性和魅力,也即获得人在自然万物中的"尊严"。因此,要"努力好好地思想","提高自己"。 (2)文章说:"我们要努力好好地思想'。这就是道德的原则。"试对这句话做一点解释说明。 【明确】道德,是人们共同生活及其行为的准则和规范。道德通过对社会的或一定阶级的舆论对社会生活起约束作用。作者把"努力好好地思想"作为道德的原则,说明作者思想的高度重视。 7、读完《信条》,你有什么整体感受, 文章开宗明义:"我真正需要知道的一切,即怎样生活,怎样做事和怎样为人,我都在幼儿园就学过。"接着,文章列出了十七条。作者认为"你需要知道的任何东西都在上边那些条条里"。这些条条是人们过正常合理生活所需要遵守的那些最基本的原则,包括个人的健康习惯,正确处理与他人关系,对生活保持热情和兴趣,承认死亡以及注意观察周围事物等。作者进一步阐释,十七条中的任何一条,都可以应用到成年,实践于家庭、社会,生活、工作之中,甚至可以用到政府行为中。最后,作者强调十七条中的一条:"当你们出门,到世界上去走走,最好还是手拉手,紧挨一起。"这是强调人与人之间要团结,要互相关爱,要有集体精神。 8、问题探讨 【提问】作者列出这些最基本的信条有什么意义, 【明确】人们常说,真理是朴素的。又说,世界上最难的事情,就是用最简明的语言表达最复杂的道理。对于应该怎样生活、怎样做事和怎样做人,在当代社会生活中越来越没有固定的衡量标准,使一般人难以把握、望而生畏。作者却把复杂问题简单化,而且讲得机智、幽默,充满温情,容易为人们所接受。正如文章中所说,这些信条"贴近真实,清晰明了并且坚实可靠",便于大家信心十足地去实践,并获得成功。有一位诺贝尔奖获得者就说过,他之所以能得奖,原因就在于他实践了幼儿园中学到的那些基本的信条。 13 确立自信(作文) 1 唱响自信之歌导学案 知识目标:掌握树立和增强自信的方法。 能力目标:在生活中能运用正确的方法增强自信。 情感、态度与价值观:帮助学生树立自信,增强自信,悦纳自我,培养正确的人生态度。 重难点: 1. 树立和增强自信的方法。 2. 理解个人自信与民族自信的关系,将个人自信上升为民族自信。 学习内容: 活动一 发现自己的进步,看到自己的长处。 1.阅读课本第27页第一小框题的故事,思考问题: 这个盲人为什么变得这样自信, 2.我的点滴进步: 3小调查“我眼中的你” 我对你的印象是什么, 你的优点是什么, 活动二 增强信心与实力 1.分析教材第29页案例。 “跨栏冠军是怎样成长为一名出色的推销员的,” 2. “自信度的测量”调查: 在符合你的情况题目的括号内划“?”,在不符合你的情况题目的括号内划“×”。如果你还具备其他能力请填在第9题中,不具备的能力填在第10题中。 1. 我对自己的自理能力充满信心( ) 2. 我对自己的体育能力(速度、力量、弹跳力等)充满信心 3. 我对自己的组织能力充满信心( ) 4. 我对自己的社交能力充满信心( ) 5. 我对自己的学习能力充满信心( ) 6. 我对自己的表演能力充满信心( ) 7. 我对自己的绘画能力充满信心( ) 8. 我对自己的自我保护能力充满信心( ) 9. 我还对自己的以下能力充满信心: _____________________________________ 10. 我还对自己的以下能力不自信: _____________________________________ 在以后的生活中,怎样进一步增强信心与实力, 活动三:做自信的中国人 发生在上海张家宅小学的故事: 一群大洋彼岸的旅游者,到上海张家宅小学参观,学生们很有礼貌地欢迎,以礼相待。但是,这批游客走后,不少学生在学校的花盆里、黑板槽里发现了好多外国硬币。于是,该校三名小学生郑重地向校长递交了一封信,要求转交给这批游客。信中写道:“我们抱着十分遗憾的心情给你们写信。可曾记得,当你们 来到的时候,我们以礼相待,鼓掌欢迎,可是你们却有意把硬币放在我们教室的黑板槽里和花盆里。我们认为这是不友好的行为。我们坦率地告诉你们:我们不是葛朗台的子孙,我们是共产党培养出来的少先队员„„我们爱自己的祖国胜过爱自己的生命。因为穷可以靠我们的双手来改变。希望你们不要做这种欺侮我们少先队员的傻事。” 1.上海张家宅小学的学生做法是否正确,为什么, 2.你认为在国际交往中我们应怎么做, 课后练习: 1.世纪初,徐悲鸿在法国留学时,曾碰到一个洋人挑衅。那个洋人说:“中国人愚昧无知,生就当亡国奴的材料,即使送到天堂,也成不了才~”,徐悲鸿义愤填膺的回答:“那好,我代表我的祖国,你代表你的国家,等学业结束时,看谁是人才,谁是蠢才”。一年之后,徐悲鸿的油画受到法国艺术家的好评,此后数次竞赛,他都拿了第一。他的个人画展,轰动了整个巴黎艺术界。这样令人惊叹的成就,是那个洋人远远不及的。 1.从短文中,我们看到 ( ) A.徐悲鸿的自信与中华民族的自信结合在一起 B.那个洋人没有自信 B.徐悲鸿是人才,那个洋人是蠢才 D.徐悲鸿为祖国而努力创作。 2.(多项)在国际交往中,我们要做到( ) A.把祖国的利益放在第一位,把祖国的荣辱放在第一位 B.自尊,自爱,不卑不亢 C.同有损祖国尊严的言行做坚决的斗争 D.积极学习外国的先进技术和经验,更好地增强民族自尊心和自信心 3. 一个毕业的大学生去参加某公司的招聘考试,公布成绩时他名落孙山,得知消息后他非常失望,顿生轻生念头,幸亏抢救及时而未果。过了几天消息传来,上次公布的成绩是由于统计出错,才使得他名落孙山,实际上它的成绩名列前茅。但是他仍然未被录取。据此回答: 你认为该大学生没有被录取的原因是 ( ) A.他的成绩不好 B.一个没有自信心的人,将与成功无缘 C.公司又一次出现统计错误 D.他很自负 面对那位沮丧的大学生,你最想对他说 ( ) A.对未来生活的自信,是理智的期望,相信自己“我能行” B.自尊是自己争取的,面子是别人施舍的。自尊才真正属于自己 C.嫉妒是心灵的肿瘤 D.一个人能否有成功,就看他是否具备民族自尊心和自信心。 4.有的同学说,我学习不好,也没有什么特长,我什么都不如别人。你赞成这一说法吗,为什么, 廉颇蔺相如列传》导学案(一) 14.《 [学习目标] 1、了解司马迁生平及《史记》的有关知识及故事发生的背景。 2、掌握“闻”“见”“却”“因”“以”“顾”等文言实词和虚词的意义和用法。 3、分析蔺相如的形象。 教学难点、重点分析 ]学习目标2、3 [ [学习方法] 通过反复诵读掌握重点实虚词和人物形象的客观分析。 [知识链接] 1、作者介绍 1(司马迁简介。 司马迁(约前145,约前90),西汉著名史学家、文学家和思想家。字子长,夏阳(今陕西朝韩城南)人。其父司谈是汉朝太史令(掌管起草文书、编写史料,兼管国家典籍、天文历法的官职)。受其父影响,他幼年时期就酷爱学习。20岁以后,他游遍南北,到处考察风俗,采集传说,为《史记》的写作提供了丰富的资料。后来,继承父职,担任太史令,有机会读遍皇家藏书。公元前99,因上书替投降匈奴的李陵辩解而被捕下狱,受腐刑。出狱后发愤著书,历尽十载完成《史记》。 2(《史记》简介。 《史记》是我国第一部记传体通史,记载了从传说中的黄帝到汉武帝长达三千年间的历史。全书共130篇,包括本纪12篇,世家30篇,列传70篇,年表10篇,书8篇,共52万字。本纪、世家、列传用于记述人物事迹,书用于说明各种制度的发展变化,表用于显示史事的脉络,奠定了后世写史的体例。《史记》高度地反映了历史的真实,有很高的史学价值。不仅如此,《史记》又有很高的文学价值。书中对部分历史人物的叙述,语言生动,形象鲜明。学习《廉颇蔺相如列传》这篇课文,我们将对此有深刻的体会。 3.时代背景简介。 战国末期,秦、楚、齐、赵、韩、魏、燕七国中,秦的势力最大,秦要统一中国,采取远交近攻、各个击破的策略,积极对外扩张。他南面的楚国和西北的赵国,实力比较强。在蔺相如完璧归赵和渑池会的时候,秦国的主要力量正对付楚国,所以它对赵虽虎视眈眈,不时蚕食欺压,却还抽不出主要力量来大举进攻赵国。课文所讲的秦赵两国之间、赵内部之间的将相的矛盾冲突,就是在这种历史条件下发生的。 [学习过程] 一、初读课文,读准加点字音。(A级) 倨( )睨( )镬( )嘻( )觑( )渑( ) 怿( )匿( )驽( )刎( )颈之交 列观( ) 二、积累文言知识(B级) (,) 一词多义 闻:以勇气闻于诸侯 ( ) 忽闻门外虫鸣( ) 见:秦诚恐不可得,徒见欺 ( ) 大王见臣列观,礼节甚倨 ( ) 指:秦指示王 ( ) 头发上指 ( ) 却:相如因持璧却立 ( ) 已却秦存赵,使将将其军归魏( ) 负:秦贪,负其强 ( ) 均之二策,宁许以负秦曲( ) 臣诚恐见欺于王而负赵( ) 因:因宾客至相如门谢罪( ) 不如因而厚遇之 ( ) ( 2 )词类活用(B级) 相如视秦王无意偿赵城,乃前曰 乃使其从者衣褐 左右欲刃相如 舍相如广成传舍 宁许以负秦曲 臣请完璧归赵 毕礼而归之 秦王恐其破璧 大王必欲急臣 未尝有坚明约束者 卒廷见相如。 赵王于是遂遣相如奉璧西入秦 3 )文言句式(B级) ( 1、廉颇者,赵之良将也 。 2、为赵宦者令缪贤舍人。 3、秦,虎狼之国。 4、求人可使报秦者。 5、欲与秦,秦城恐不可得,徒见欺。 6、臣诚恐见欺于王而负赵 7、使不辱于诸侯 8、秦王坐章台见相如 三、赏读1~7文段(C级) 1、浏览1~3段,说说本文在安排主要人物出场上的特点。 2、蔺相如出使秦国后,与秦国围绕着和氏璧展开了一场比智比勇的较量,由此可见他怎样的性格特点, 3、本文第7段已将”完璧归赵”故事叙述完毕,为什么作者在它后面又加上”秦亦不以城予赵,赵亦不予秦璧”这两句话? (D级) 作业布置: 一、熟读课文并背诵最后五段。说说课文中主要写了哪几件大事,从中可以看出廉颇、蔺相如各有什么样的性格特点。 二、联系上下文品味下列语句,注意文中的细节描写。 1、相如因持壁却立,倚柱,怒发上冲冠„„ 2、相如持其璧睨柱,欲以击柱。 3、秦王与群臣相视而嘻。 4、秦王怒,不许。於是相如前进缶,因跪请秦王。秦王不肯击缶。相如曰:“五步之内,相如请得以颈血溅大王矣~”左右欲刃相如,相如张目叱之,左右皆靡。于是秦王不怿,为一击缶。 三、联系上下文,指出下列各句中加点的词语的意义或用法。 1、城不入,臣请完璧归赵。 2、大王必欲急臣,臣头今与璧俱碎于柱矣~” 3、遂许斋五日,舍相如广成传。 4、使其从者衣褐,怀其璧, 5、卒廷见相如,毕礼而归之。 6、且庸人尚羞之,况于将相乎, 7、吾所以为此者,以先国家之急而后私仇也。 四、给下面一段文字加上标点,翻译为现代汉语,并谈谈对这段话的理解。 (注:直截加上标点。) 太史公曰:知死必勇,非死者难也,处死者难。方蔺相如引璧睨柱,及叱秦王左右,势不过诛,然士或怯懦而不敢发。相如一奋其气,威信敌国,退而让颇,名重太山,其处智勇,可谓兼之矣~ 译文: 《 廉颇蔺相如列传》导学案(一)答案 本导学案的答案均见课本。 作业布置: 一、参考答案:本文通过“完璧归赵”“渑池之会”“负荆请罪”三个典型事件,在尖锐的矛盾冲突中刻画廉颇、蔺相如的性格。在“完璧归赵”中,表现出蔺相如的有勇有谋;在“渑池之会”中,表现出蔺相如的英勇果敢、不畏强暴。这两个事件主要表现蔺相如的大智大勇。在“负荆请罪”中,表现蔺相如的顾全大局,宽厚待人,“先国家而后私仇”的精神。本文详写蔺相如,略写廉颇,廉颇的性格主要在“负荆请罪”中体现出来,那就是忠于国家、直率坦白、勇于改错。 二、1、用夸张的手法,形容蔺相如的愤怒,生动形象。 2、一个“睨”字就把那种与璧同存亡的坚决神态勾勒出来;也有不把秦王强权放在眼里的气势。 3、 相视而嘻形容秦王与群臣懊丧而又无可奈何的样子,他们在蔺相如的智勇面前理屈词穷,无计可施,不知所措的尴尬神态。 4、 写蔺相如同秦王的针锋相对的斗争。一方面蔺相如步步进逼,秦王则节节败退。秦王从气势汹汹到无可奈何到狼狈不堪的过程和情状得到鲜明表现。另一方面在武士面前,蔺相如的英勇和威武不屈与秦王武士的迟疑软弱退却形成强烈对比,扫尽秦王与武士的威风。 三、1、完:使动用法,使„„完好无缺 2、急:动词,逼迫。 、舍:动词,安置住宿 3 4、 衣:动词,穿 5、廷:名词作状语,在朝廷上 6、羞:意动用法,以„„为羞 7、先、后:意动用法,以„„为先,以„„为后 2、太史公曰:知死必勇,非死者难也,处死者难。方蔺相如引璧睨柱,及叱秦王左右,势不过诛,然士或怯懦而不敢发。相如一奋其气,威信敌国,退而让颇,名重太山,其处智勇,可谓兼之矣~ 译文:太史公说:知道将死而不害怕,必定是很有勇气的,死并不是难事,勇敢面对死亡这才是难事。当蔺相如手持和氏璧斜视庭柱,以及呵斥秦王左右的时候,就当时的形式来说,最多不过是被杀,然而一般人却往往因为胆小而不敢有这样的表现。相如一旦振奋起他的勇气,其威力就伸张出来压倒了敌国。回来后有对廉颇隐忍退让,他的声誉比泰山还重,就智慧和勇气来说,他可以说是兼而有之。 14.《 廉颇蔺相如列传》导学案(二) [学习目标] 1、分析8~10段,积累重点字词,翻译重点语句。 2、体会本文善于以典型事件以及个性化的语言、行动描写揭示人物性格的表现手法。 3、认识蔺相如机智勇敢、不畏强暴和顾全大局的精神;认识廉颇知错就改和勇于改过的精神。 [教学难点、重点分析 ]学习目标2、3 [学习方法] 诵读法、比较法 [学习过程] 一、齐读4、5、6段(A) 二、 研读8~12段,完成下列问题。 1、一词多义(B级) 以:以相如功大,拜为上卿 ( ) 蔺相如徒以口舌为劳 ( ) 以先国家之急而后私仇也 ( ) 愿以十五城请易壁 ( ) 请以太子为王,以绝秦望 ( ) 引:相如引车避匿 ( ) 左右欲引相如去 ( ) 引赵使者蔺相如 ( ) 顾:顾吾念之,强秦之所以不敢加兵于赵者 ( ) 激昂大义,蹈死不顾 ( ) 三顾臣于草庐之中 ( ) 2、词类活用(B级) 左右欲刃相如 蔺相如固止之 以先国家之而后私仇也 3、古今异义 (B级) 璧有瑕,请指示王。 宣言曰:“我见相如,必辱之~” 于是相如前进缻,因跪请秦王。 未尝有见坚明约束者也 臣所以去亲戚而事君者,徒慕君之高义也 4、翻译下列句子。(C级) (1)计未定,求人可使报秦者。 (2)夫赵强而燕弱,而君幸于赵王,故燕王欲结于君。 (3)公之视廉将军孰与秦王, (4)吾所以为此者,以先国家之急而后私仇也~ 三、文段分析,思考 (D级) 1、根据廉颇送别赵王时说的那一段话,说说廉颇对待国家大事的态度。 2(廉颇始而”宣恶言”,闻蔺相如语即”肉袒负荆”,”至蔺相如门谢罪”,这前后对比说明了什么? 3、本文叙述了蔺相如那几件事,突出了蔺相如那些品质特征, 四、学习反思 五、当堂检测 (B级) 1、下列加点字解释正确的一项是( ) (1)秦城恐不可得,徒见欺( 见到 ) (2)于是相如前进缶,因跪请秦王(因此) (3)秦昭王闻之,使人遗赵王书(留下,留给) (4)臣请就汤镬(接受) 2、下列词的意 思与现代汉语相同的一项是( ) (1)传以示美人及左右 (2)明年,复攻赵,杀两万人 (3)相如既归,赵王以为贤大夫 (4) 赵王于是遣相如奉璧西人秦 3、下列加点字的意义和用法相同的一项是( ) (1)臣舍人相如可使 (2)今君乃亡赵走燕 不如因而厚遇之 设九宾礼于廷,臣乃敢上璧 (3)赵以盛设兵以待秦 (4)怀其璧,从经道亡,归于赵 蔺相如以口舌之劳 今君乃亡赵走燕 4、下列句式和“今君乃亡赵走燕”相同的一项是( ) (1)计未定求人可使报秦者 (2)夫赵强而燕弱,而君幸于赵王 (3)遂于秦王会于渑池 (4)于是秦王不怿,为一击缶 5、请写出下列各句中加点字的词类活用 (1)赵王于是遣相如奉璧西人秦 (2) 设九宾礼于廷,臣乃敢上璧 (3)毕礼而归之 (4) 臣请完璧归赵 (5)左右欲刃相如 [拓展阅读] 在对比中表现人物性格是《廉颇蔺相如列传》的突出特点,仔细分析,对我们的写作学习有很重要的指导 意义。下面,我们就《廉颇蔺相如列传》,谈谈它的八类对比。 一、 廉颇和蔺相如的对比 这是课文开始就有的对比。一个是上卿,一个是舍人,地位差别非常大,从而为下文的将相不和埋下伏笔,这也就是为什么后来廉颇骂蔺相如“素贱人”的原因。这一对比表现了司马迁老先生在结构上的巧妙。如果没有了这里的地位差别对比,下文的将相不和时,就不可能有廉颇的目空一切的话。正是蔺相如的出身低贱,为廉颇攻击对方找到了自以为是的理由。同样,也是在后面的从将相不和到将相和的对比文字中,我们先是看到了廉颇的急躁冒进,不计后果的卤莽,接着我们又看到了蔺相如的沉稳老练,胸怀宽广,最后我们还看到了一个伟大的蔺相如-----不计前嫌,也看到了一个同样伟大的廉颇-----知错就改。性格对比鲜明让人物形象跃然纸上。 二、 蔺相如和缪贤的对比 为了表现蔺相如的足智多谋,司马迁为我们安排了一个材料:蔺相如巧计救缪贤。文章通过俩个人对同一个事情的不同认识的对话-----到底应该不应该“亡赵走燕”,表现了蔺相如的聪明周到和细致,但是作为主人的缪贤却远不如蔺相如高级,真正的上不如下。 三、蔺相如和诸大臣的对比 一个秦国易璧的难题让赵国的大臣们进入了二难推理,“欲予秦,秦城恐不可得,徒见欺;欲勿予,即患秦兵之来”,真正的无可奈何了。但是蔺相如一拿到问题就有了方法,告诉赵王:不可不许。语言斩钉截铁,痛快淋漓。和诸大臣的对比之中,优劣高下,不言自明。 四、 相如和赵王的对比 当蔺相如告诉赵王秦人易璧“不可不许”后,赵王仍然战战兢兢的问“取吾璧,不予我城,奈何,”最后,还是在蔺相如保证完璧归赵后,他才放了心。一个胆小怕事,一个胆大心细,对比写来,栩栩如生。这样的对比,在后面的渑池会前到底去不去的讨论中,司马迁给我们有同样精彩的描写。先是赵王“畏秦,欲勿行”,后来是廉颇和蔺相如的鼓动下的“王遂行”。从这里,我们看到了赵王的毫无主见,也看到了蔺相如的深谋远虑。 五、 蔺相如和秦王的对比 完璧归赵的故事中,蔺相如的机智勇敢,周到细致,随机应变,被作者写的出神入化,叫人拍案叫绝。而与之相对的秦王却是傻瓜一个,蠢驴一头,呆头呆脑地被蔺相如戏耍了个不亦乐乎。特别是蔺相如在秦王斋戒五日后,没有了玉璧,这是非常难处理的。但是蔺相如毕竟是蔺相如,他来了个死猪不怕开水烫,“臣知欺大王之罪当诛”“唯大王与群臣孰计议之”。于是秦王在无计可施后,只好“毕礼而归之”。对于蔺相如的高级和秦王的低级,在后面的渑池会上,司马迁老先生的文字给我们还有同样表现。秦王本来“不肯击缶”,但是在蔺相如的大无畏的精神面前,这个秦王终于“为一击缶”了。相形而见绌,对比是多么鲜明。 六、 赵王和秦王的对比 一个让人莫名其妙的问题是,一个对待蔺相如毫无办法的家伙,在对付赵王的问题上却能游刃有余。渑池会上,气焰嚣张的秦王把赵王调戏了个够,而胆小怕事的赵王却只有言听计从,不敢有任何反抗,秦王让他奏瑟,他也只能“鼓瑟”。其实,这样的对比从文章开始就有了,对秦王的换璧的要求,赵王是除了害怕还是害怕。害怕“见欺”,害怕“秦兵之来”。赵国国王做到这种程度,让读者读到这里也只有叹息的了。 七、 蔺相如和他自己舍人的对比 这主要表现在蔺相如和他的舍人,在如何处理将相不和的态度的对比。在蔺相如和他们的对话中,我们看到了蔺相如的深谋远虑,也看到了舍人的粗浅平庸。“吾所以为此者,以先国家之急而后私仇也”,并不是舍人们认为的害怕。但是我们看到的文字是,在此之前这些舍人自作聪明的“相与谏”和“请辞去”的要挟。无奈之下,才有了蔺相如一番语重心长的告诫和解释。对比之下,蔺相如的性格更加鲜明突出,读者的印象自然更加深刻。 八、 蔺相如和廉颇他们自己的前后对比 这里既有他们地位变化的前后对比,又有他们自己行动变化的前后对比。蔺相如从一个别人的舍人上升到自己养有舍人,“位在廉颇之右”,是他刻苦努力的结果,也是他的性格使然。同样,在从将相不和到将相和的过程中,正是廉颇的性格让他有了前后看似矛盾的举动。从“宣恶言”到“肉袒负荆”是勇敢直爽的体现,正是直爽不周全才导致卤莽,而正是勇敢,才能有自己向蔺相如的负荆请罪。而正是有了蔺相如的 深谋远虑,才有了他的“望见廉颇”“避匿”,才有了他的不想跟矛盾的举动其实并不矛盾,其实都是统一 在人物的性格之中的。 总之,课文《廉颇蔺相如列传》中的八类对比是作者匠心独运的结果,阅读时要仔细体会才是。廉颇“争 列”,才有了他的与廉颇的“卒相与欢”。 《 廉颇蔺相如列传》导学案(二)答案 本导学案的答案均见课本。 15.《苏武传》导学案(一) 学习目标 1(查阅有关资料,了解文章背景。 2. 能用普通话准确、流畅地朗读文本,在理解的基础上,整体感知文本内容。 3. 理解文本中重要文言 实词的意义,大致疏通文意,读懂文章内容。 学习重点: 积累文中的实词、虚词和句式。 学习难点: 理解文本中重要文言实词的意义,疏通文意,读懂文章内容。 一、导语设计: 两千年前,在北海边上,有一位杖节牧羊的老人,他坚守着热爱祖国、忠贞不渝的民族气节,在饥寒交迫中艰难 地生存着,在他的身上体现了“富贵不能淫、贫贱不能移、威武不能屈”的传统美德和高尚人格。其气节, 令人肃然起敬。他没有在敌人的淫威面前屈服,也没有在金钱、富贵面前动心。为了远大的理想、为了亲 爱的祖国,他餐冰卧雪、不辱使命,义无反顾地献出了自己的青春,坚守住了自己的信念。一直以来,这种英 勇义举激励着千秋万代的华夏儿女奋然前行.可以说,苏武,真正称得上是我们民族的脊 梁。 二、学习知识及巩固 作者及《后汉书》 班固(32——92年),字孟坚,扶风安陵(今陕西咸阳市东)人。东汉著名的史学家。《后汉书•班固传》称他“年九岁,能属文,诵诗赋。及长,遂博贯载籍,九流百家之言,无不穷究。所学无常师,不为章句,举大义而已”。其父班彪曾续司马迁《史记》作《史记后传》,未成而故。班固立志继承父业,在《后传》基础上,进一步广搜材料,编写《汉书》。后因有人向汉明帝诬告他篡改国史,被捕入狱。其弟班超上书解释,始得获释,被命为兰台令史,经过二十多年努力,写成了《汉书》。汉和帝永元初年,班固随窦宪出征匈奴,不久窦宪因谋反案被诛,班固也受牵连被捕,死于狱中。《汉书》中的八“表”与“天文志”是由其妹班昭和同郡人马续续成的。班固的《汉书》是我国第一部纪传体断代史,体例模仿《史记》,但略有变更。全书有纪十二篇,表八篇,志十篇,传七十篇,共一百篇,起自汉高祖,止于王莽,记西汉一代二百三十年间史实。《汉书》评价历史人物往往从封建正统观念出发,以儒家的伦理道德作为标准,如对陈涉、项羽加以贬抑,即是显例。历来《汉书》与《史记》并称,史学家刘知几说《汉书》“言皆精炼,事甚该密”(《史通•六家》),则是其特色。 三、 背景知识介绍: 汉武帝开始对匈奴进行长期的讨伐战争,其中取得了三次具有决定意义的胜利,时间为公元前127年、前121年、前119年。匈奴的威势大大削弱之后,表示愿意与汉讲和,但双方矛盾还是根深蒂固。所以,到公元前100年,苏武出使匈奴时,却被扣留,并迫使他投降。《苏武传》集中叙写了苏武出使匈奴被扣留期间的事迹,热烈颂扬了他在敌人面前富贵不能淫,贫贱不能移,威武不能屈,饥寒压不倒,私情无所动的浩然正气,充分肯定了他坚毅忠贞,大义凛然,视死如归的民族气节。 四、学习目标达成过程 1. 听读录音,写出下列句中的生字注音。 ?数通使相窥观( ) ?且鞮侯单于初立( ) ?空以身膏草野( ) ?稍迁至栘中厩监( ) ?置煴火( ) 2(学 生再读课文,标注重点实词虚词。 3. 同桌之间互相讨论交流学习自己不懂的内容。 4. 文言基础知识归纳。 5. 学生再读文本,选出自己认为重要的文言词语,借助注释和工具书制作文言词语卡片,在班内进行交流。 【通假字】 且单于信女,使决人死生 不顾恩义,畔主背亲 与旃毛并咽之 掘野鼠去草实而食之 空自苦亡人之地 信义安所见乎 孺卿从祠河东后土 法令亡常 大臣亡罪夷灭者数十家 武父子亡功德 因泣下沾衿,与武决去 始以疆壮出 【古今异义】 汉亦留之以相当 皆为陛下所成就 位列将 列:一般的,普通的。 兄弟亲近 我丈人行也。 欲因此时降武。 独有女弟二人 且陛下春秋高 武等实在 【词类活用】 1(意动用法 单于壮其节 2(使动用法 (1)欲因此时降武 (2)空以身膏草野 (3)反欲斗两主 (4)单于愈益欲降 (5)何久自苦如此! (6)王必欲降武 3(名词活用 (1)天雨雪 (2)羝乳乃得归 (3)杖汉节牧羊 (4)孺卿从祠河东后土 (5)惠等哭,舆归营 (6)陵与卫律之罪,上通于天 【特殊句式】 检查特殊句子的翻译 1(倒置句 (1)送匈奴使留在汉者 (2)为降虏于蛮夷 (3)何以女为见 (4)子卿尚复谁为乎 (5)何以复加 2(判断句 (1)缑王者,昆邪王姊子也 (2)非汉所望也 (3)汉天子,我丈人行也。 3(被动句 (1)见犯乃死,重负国 (2)大臣亡罪夷灭者数十家 (3)皆为陛下所成就 五、基础知识: 1(填空: (1)班固,东汉著名的 ,《汉书》开创了“包举一代”的 体例,是继《史记》之后 又一部历史与文学巨著。文学史上,司马迁与班固并称 。 (2)“二十四史”是指由《史记》至《明史》的二十四部纪传体史书。它的前四史是 、 、 和 。 (3)班固又是东汉最著名的的辞赋家, 是他最重要的作品之一。 2、下列句子,属于名词用作状语的是 A、惠等哭,舆归营。 B、空以身膏草野,谁复知之, C、收族陵家,为世大戮。 D、使者大喜,如惠语以让单于。 3、下列句子,属于被动句式的是 A、收族陵家,为世大戮。 B、畔主背亲,为降虏于蛮夷,何以女为见, C、见犯乃死,重负国。 D、终不得归汉,空自苦无人之地,信义安所见乎, 4.指出下列各句中的通假字,并写出通哪个字。 ?暴秦之欲无厌 ?使遂蚤得处囊中,乃颖脱而出 ?而境界危恶,层见错出 ?辞决而行 ?以至晋鄙军之日北乡自刭 ?秦有余力而制其弊 ?至舍,四支僵劲不能动 ?同舍生皆被绮绣 ?独不怜公子姊邪 ?可得闻与 5(选出“以”字用法与例句相同的一组:( ) 例:少以父任,兄弟并为郎。 ? 张胜闻之,恐前语发,以状语武。? 以相如功大,拜为上卿。 ? 请立太子为王,以绝秦望。 ? 武以始元六年春至京师。 ? 士亦以此不附焉。 ? 使者大喜,如惠语以让单于。 ? 亦以数直谏,不得久居位。 ? 持节发河南仓粟以振贫民。 A(??? B(??? C(??? D(??? 6(下列句中的“于”字已按其作用作了分类,选出分类正确的一项是:( ) ?赵氏求救于齐。 ?而不及今令有功于国。 ?而谋动干戈于邦内。 ?望长安于日下。 ?夫庸知其年之后生于吾乎。 ?而耻学于师。 ?于其身也,则耻师焉。 ?师不必贤于弟子。 A(??/??/??/?? B.??/??/??/?? C(??/??/??/?? D.??/??/??/?? 7(下列说法不正确的一项是:( ) A(《史记》是我国第一部纪传体通史,它记述了上自传说中的黄帝,下至汉武帝三千年的历史,具有 极高的史学价值和文学价值,被鲁迅誉为“史家之绝唱,无韵之离骚”。 B(东汉人班固写的《汉书》是我国第一部断代史,它记述了汉高祖之后的200多年的历史,行文结构严谨,语言精炼,对后世的史学和文学有较大的影响。 C(《廉颇蔺相如列传》一文运用多种文学手段,揭示人物的性格,将人物写的栩栩如生。文中的成语“完璧归赵”“负荆请罪”“人为刀俎,我为鱼肉”直到现在仍被广泛应用。 D(《鸿门宴》一文紧扣住情节的发展刻画人物性格,同时运用了对照的手法,使人物形象互相映衬,相得益彰。语言运用精当,生动传神,其中“项庄舞剑,意在沛公”等一直沿用至今。 8(把下列句子译成现代汉语。 (1)单于益骄,非汉所望也 (2)左伊秩訾曰:“即谋单于,何以复加,宜皆降之。” (3)今得杀身自效,虽蒙斧钺汤镬,诚甘乐之。 (4)后汉使复至匈奴,常惠请其守与俱,得夜见汉使,具自陈道。 9(下面是对本课内容的分析,不正确的一项是:( ) A(苏武出使匈奴,是在汉与匈奴关系有所改善,两国矛盾有所和缓时期,本不应有危险,然而,由于张胜私下支持虞常等人的谋反选择,导致了苏武被匈奴扣留。 B(虞常沦落匈奴,但没有忘记汉朝,他伏弩射卫律的行动,正是他爱国的具体表现。 C(苏武的自杀举动,表现了汉朝使节宁死不屈的凛然正气,因此赢得了敌国的敬意。 D(卫律使尽威吓、利诱等手段,但始终未能动摇苏武对国家、对民族忠贞不贰的崇高气节。 五、 作业布置 熟读文本内容,复述课文主要内容,并与同学们交流。 《苏武传 》 参考答案一 整理字词句 女:通“汝”,你。 不顾恩义,畔主背亲 畔:通“叛”,背叛。 与旃毛并咽之 旃:通“毡”,毛织品。 掘野鼠去草实而食之 去:通“()”,收藏。 空自苦亡人之地 亡:通“无”,没有。 信义安所见乎 见:通“现”,显现。 孺卿从祠河东后土 祠:通“祀”,祭祀。 法令亡常 亡:通“无”,没有。 大臣亡罪夷灭者数十家 亡:通“无”,没有。 武父子亡功德 亡:通“无”,没有。 因泣下沾衿,与武决去 衿:通“襟”,衣襟。 决:通“诀”,诀别。 始以疆壮出 疆:通“强”,强壮。 【古今异义】 汉亦留之以相当 古义:相抵偿。 今义:副词,表程度。 皆为陛下所成就 古义:提拔。 今义:业绩。 位列将 古义:位;官位。 今义:位置、地位。 列:一般的,普通的。 今义:排列。 兄弟亲近 古义:亲近的侍臣。 今义:动词,靠近。 我丈人行也。 古义:老人,长辈。 今义:岳父。 欲因此时降武。 古义:趁这时。 今义:相当于所以。 独有女弟二人 古义:妹妹。 今义:姐姐(妹妹)和弟弟。 且陛下春秋高 古义:年纪。 今义:春秋战国时期或指季节。 武等实在 古义:确实存在。 今义:诚实、老实。 【词类活用】 1(意动用法 单于壮其节 壮:形容词的意动用法,认为„„壮。 2(使动用法 (1)欲因此时降武 降:使动用法,使„„投降。 (2)空以身膏草野 膏:使动用法,使„„肥沃。 (3)反欲斗两主 斗:使动用法,使„„争斗。 (4)单于愈益欲降之 降:使动用法,使„„投降。 (5)何久自苦如此! 若:使动用法,使„„受苦。 (6)王必欲降武 降:使动用法,使„„投降 3(名词活用 (1)天雨雪 雨:名词做动词,下,$ (2)羝乳乃得归 乳:名词做动词,生子 (3)杖汉节牧羊 杖:名词做动词,拄着 (4)孺卿从祠河东后土 祠:名词做动词,祭祀 (5)惠等哭,舆归营 舆:名词做动词,抬。 (6)陵与卫律之罪,上通于天 上:名词做状语,向上 【特殊句式】 检查特殊句子的翻译 1(倒置句 (1)送匈奴使留在汉者 定语后置句,正常语序“送留在者汉匈奴使”。 (2)为降虏于蛮夷 状语后置句,正常语序“于蛮夷为降虏”。 (3)何以女为见 宾语前置句和介宾倒置句,正常语序“以何见女为” (4)子卿尚复谁为乎 宾语前置句,正常语序“子卿尚复为谁乎” (5)何以复加 介宾倒置句,正常语序“以何复加” 2(判断句 (1)缑王者,昆邪王姊子也 (2)非汉所望也 (3)汉天子,我丈人行也。 3(被动句 (1)见犯乃死,重负国 (2)大臣亡罪夷灭者数十家 (3)皆为陛下所成就 基础知识 1((1)史学家 断代史 班马(2)《史记》《汉书》《后汉书》《三国志》(3)《两都赋》 2.A(舆) 3、C(见犯) 4.?“厌”通“餍”?“蚤”通“早”?“见”通“现”?“决”通“诀”?“乡”通“向”?“弊”通“敝”?“支”通“肢”?“被”通“披”?“邪”通“耶”?“与”通“欤” 5(C 6(D(向/对于/在/比) 7(C (略。 9(B 8 15.《苏武传》导学案(二) 学习目标: 1. 在读懂文本内容的基础上了解和学习本文对比手法的运用。 2. 学习苏武坚贞不屈的爱国精神。 3. 领会文章蕴涵的精神,概括苏武这一人物形象的意义。 学习重点、难点 在对比中分析苏武的人物形象。 学习过程: 一、 学生复述课文内容,导入新课。 二、 走进文本,根据自己的理解,简要分析苏武这一人物形象。 思考探究: 班固是怎样用对照、映衬的手法塑造主人公形象的, 三、阅读下面的文言文,完成1-5题。 初,霍氏(指西汉权臣霍光子孙)奢侈,茂陵徐生曰:“霍氏必亡。夫奢则不逊,不逊必侮上;侮上者,逆道也。在人之右,众必害之。霍氏秉权日久,害之者多矣。天下害之,而又行以逆道,不亡何待~” 乃上疏,言:“霍氏泰盛;陛下即爱厚之,宜以时抑制,无使至亡。”书三上,辄报闻。 其后,霍氏诛灭,而告霍氏者皆封。人为徐生上书曰:“臣闻客有过主人者,见其灶直突(注:突,烟囱),傍有积薪。客谓主人:‘更为曲突,远徙其薪;不者,且有火患。’主人嘿然不应。俄而家果失火,邻里共救之,幸而得息。于是杀牛置酒,谢其邻人。灼烂者在于上行,余各以功次座,而不录言曲突者。人谓主人曰:‘乡使听客之言,不费牛酒,终亡火患。今论功而请宾,曲突徙薪无恩泽,焦头烂额为上客耶?’主人乃寤而请之。今茂陵徐福数上书言霍氏且有变,宜防绝之。乡使福说得行,则国亡裂土出爵之费,臣亡逆乱诛灭之败。往事既已,而福独不蒙其功。唯陛下察之——贵徙薪曲突之策,使居焦发灼烂之右。”上乃赐福帛十匹,后以为郎。 宣帝始立,谒见高庙,大将军霍光从骖乘,上内严惮之,若有芒刺在背。后车骑将军张安世代光骖乘,天子从容肆体,甚安近焉。及光身死。而宗族竟诛。故俗传之曰:“威震主者不畜。霍氏之祸,萌于骖乘。” (《汉书?霍光传》) 1(句中加点词语解释有误的一项是:( ) A(霍氏秉权日久 秉:执掌 B(天下害之 害:陷害 (( C(客有过主人者 过:拜访 D(威震主者不畜 畜:容 (( 2(下列各组句子中,加点词的意义和用法相同的一项是:( ) A(?夫奢则不逊 ?每闻琴瑟之声,则应节而舞 (( B(?而又行以逆道 ?愿君即以遂备员而行矣 (( C(?于更为曲突,远徙其薪 ?遂为猾胥报充里正役 (( D(?数上书言霍氏且有变 ?凡四方之士未有不过而拜且泣者 (( 3(下列各句中“之”字称代内容与其他三句不同的一项是:( ) A(在人之右,众必害之。 ( B(霍氏秉权日久,害之者多矣。 , ( C(陛下即爱厚之,宜以时抑制,无使至亡。 ( D(唯陛下察之。 ( 4(下列句子括号中是补出的省略身份,不正确的一项是:( ) A(宜以时抑制,无使(霍氏)至亡。 B(向使(您)听客之言,不费牛酒。 C(今茂陵徐福数上书言霍氏且有变,(霍氏)宜防绝之。 D(贵徙薪曲突之策,使(徐福)居焦发灼烂之右。 5(下列句子在文中的意思,不正确的一项是:( ) A( 在人之右,众必害之——一个人地位在别人之上,大家一定忌恨他。 B( 更为曲突,远徙其薪——再修一个弯取的烟囱,把柴草移到远处 C( 灼烂者在于上行——被火烧的皮肉焦烂的人坐在上座 D(向使福说得行——假如徐福的主张能够实行 6(选出全为“霍氏诛灭”的原因的一项:( ) ?霍氏奢侈 ?在人之右,众必害之 ?霍氏秉权日久 ?徐生数上书言霍氏且有变 ?大将军霍光从骖乘 ?福独不蒙其功 A(??? B(??? C(??? D(??? 7(下列叙述不符合原文意思的一项是:( ) A(徐生认为,人君倘若爱护臣子,就应该不让臣子生活奢侈,并且不让他们长时期秉权。 B(霍氏诛灭后,徐福上书朝廷,说明自己察微知著,建议皇上抑制霍氏,论功更应受赏。 C(在文中“焦头烂额为上客”一语喻指揭发霍氏变乱的人受到封赏。 D(“贵徙薪曲突之策”的喻意是应该重视那些带预见性的意见,防患于未然。 四、 作业布置: 思考讨论: 有人认为苏武是忠诚的代称,有人认为他是愚忠,不值得效仿。你认为呢, 导学案二答案 班固是怎样用对照、映衬的手法塑造主人公形象的, 在出使匈奴之初,与副使张胜的对照。 在威逼利诱之时,与叛徒卫律的对照。 在以情相劝之时,与降将李陵的对照。 阅读下面的文言文,完成1-5题。 1(B(害:痛恨,忌妒) 2(A(表顺承,就) 3(D(这件事,余代霍氏) 4(C 5(B 6(C(?、?与霍氏被诛无关) 7((B(是另外一人上书朝廷,而徐福是在霍氏被诛之前上书皇上抑制霍氏的。) 参考译文: 当初,霍氏奢侈,茂陵徐生说:“霍氏一定得死。人奢侈就不谦虚,不谦虚就一定玷辱皇上;此人也就是背叛天道。他的地位比别人高,大家一定忌妒他。霍氏掌权很久了,忌妒他的人很多了。全部人都忌妒他,而他又背天道而行,不等待死等什么,”于是上疏皇上说:“霍氏宽裕昌盛,皇上您即使想厚待他,应当适时抑制他,不要让他最后到死亡的地步。”上书三次,才听到。 后来,霍氏被杀,而告发霍氏的人都被封官。有人为徐生上书说:“我听说有个探望主人的客人,看见他家灶上的烟囱是直的,旁边还堆了些柴火。客人对主人说:“改为弯曲的烟囱,把柴火移走,否则将有火患。主人没理他。不久主人家果真失火,邻居们一起救火,有幸使火熄灭。于是主人杀牛备酒,感谢他的邻居。身上烧伤者在上座,剩下的各按他们的功劳就座,而独独不邀请说改烟囱为弯曲的人。有人对主人说:“假使当初听了那客人的话,不用牛、酒,最终可以使火患没有。如今按功劳而邀请宾客,提出把烟囱改成弯曲的、把柴移走的人没有得到奖赏感谢,却把焦头烂额的人作为上宾吗,”主人于是醒悟而 邀请他。今茂陵徐福屡次上书说霍氏会有变化,应当防止杜绝他。假如按福所说的做,那么国家不用分割土地出卖官爵,大臣死,叛乱等事都不会发生。往事既然已经发生,唯独徐福一人没有蒙受皇恩。希望皇上明察——重视徙薪曲突的方法,把它放在焦发灼烂之人的上面。”皇上于是赏福帛十匹,之后任他为郎。 宣帝刚被立为皇上时,到高庙祭祀,大将军霍光以骖乘的身份跟从,皇上心理害怕他,像背上长了芒刺一样。后来车骑将军张安世代替光任了骖乘,天子才能从容地面对,感觉安全在身旁。一直到光死去。而他一族之人竟然全部被杀。因此民间流传说:“威严震主的人不能活。霍氏的祸,是从骖乘开始的。” 16.《张衡传》导学案(一) 【学习目标】 1、了解范晔的生平和《后汉书》。 、疏通课文大意,归纳文言知识点,整体感知课文内容,深入了解张衡。 2 3、学习本文记人叙事详略得当的写法。 【学习重点】 1、积累一定的文言知识。 2、了解张衡在各方面所取得的成就,学习他刻苦求学、科学务实、追求真理的精神。 【学习难点】 学习本文记人叙事详略得当的写法。 【学法指导】 1、利用工具书自主疏通文意。 2、小组合作解决疑难。 【知识链接】 1、作者与作品简介 范晔(398,445),字蔚宗,顺阳(今河南省淅川)人。南朝宋著名历史学家。他根据前人撰述的几十种有关后汉的历史著作,编写成《后汉书》。《后汉书》的记述,起于刘秀起兵推翻王莽,终于汉献帝禅位于曹丕,详载了东汉一百九十五年的历史。该书既具史家识见,又有较高的文学价值。司马迁(西汉)的 《史记》、班固(东汉)的《汉书》、范晔(南朝宋)的《后汉书》、陈寿(西晋)《三国志》合称为“前四史”。 2、张衡简介 张衡不但是伟大的科学家,还是伟大的文学家,他才华横溢,知识渊博,诗文俱佳,造诣精深,是中华民族文化史上一位少见的全才。在自然科学方面,他精通天文、地理、历法、数学、机械、气象;在社会科学方面,历史、文学、经学、哲学、礼法、绘画均至佳境。他既在自然科学方面为我们留下了震惊世界的浑天仪、地动仪、独飞木雕等伟大发明,又在社会科学方面留下了熠熠闪光的理论著作和文学作品。他的《二京赋》,达到了汉代大赋长篇巨制的极致;他的抒情小赋为赋的发展开创了新的途径;他的诗是我国七言诗由发展到成熟的一个标志,在中国文学发展史上起到了划时代的作用。为了纪念张衡的功绩,人们将月球背面的一环形山命名为“张衡环形山”,将小行星1802命名为“张衡小行星”。 【学习过程】 一、难词难句解释。 (1)从容淡静:从容,举止行动,《礼记•缁衣》:“衣服不贰,从容有常。”孔颖达疏:“谓举动有其常度。”《楚辞•九章》:“孰知余之从容。”王逸注:“从容,举动也。”不能作舒缓、不急迫来讲。淡静,淡泊娴静。全句:(他)平时举止淡泊娴静。 (2)衡不慕当世:当世,是当世之人,即当时的掌权者。 (3)自去史职,五载复还:张衡,公元111年(安帝永初五年)拜郎中,114年(安帝元初元年)迁尚书侍 年(安帝建光元年)迁公车司马令,126年(顺帝永建元年)再复为太史令。文中郎,115年迁太史令,121 所指“五载”,即公元121年辞太史令,拜公车司马令,到126年又复为太史令的“五载”。 (4)咸怪其无征:征,是应验、征验、效验的意思,不宜作“证据”讲。 (5)“举孝廉”、“辟公府”、“累召不应”、“公车特征”:举,推荐。辟,征召。召,呼唤,特指上对下的呼唤,引申为招致,征,召,特指君召臣。“举、辟”二字在这里都表被动。 (6)“拜郎中”、“迁为太史令”、“积年不徙”、“再转复为太史令”:拜,授予官职。迁,调动官职,一般指升官,“左迁”才是贬官。徙,表一般的调职,有时也用以表示降职,本文指前者。转,迁调官职。这四字的意思很接近,略有区别。 (7)关于“璇机”的解释:一说是玉饰的测天仪器。一说,“机”即“玑”,“璇机”是两颗星的名字,即“天璇”和“天玑”。从上下文看,似以第二说为宜。用“璇”“玑”二星指代北斗七星,又进而指代天文,“妙尽璇机之正”即精通天文的意思。(把“璇机”指代天文,可参见王力《古代汉语》下册第一分册第787页) 二、拓展延伸: 人物传记是文言文中常见的类别,而且多为官吏的传记,常涉及人宦海生涯中的升迁、调转、罢免等事。与现代汉语比较,古代官职变动用词具有词汇量丰富的特点。了解和掌握这些术语的含义、特点及规律,对于更好的阅读文言文具有很大的帮助。 第一类:表示授予、提升的词语。 征:由君王征聘社会知名人士充任官员; 辟:由中央官署征聘,然后向上荐举,任以官职; 荐、举:由地方向中央推荐品行端正的人,任以官职; 拜:授予官职; 除:任命,授予官职; 授:给予官职; 起:起用某人任以官职。 擢:在原官职上提拔; 拔:提升没有官职的人。 第二类:表示调动官职的词语。 迁:调动官职,一般是提升调用; 转、调、徙:调动官职; 补:补任缺职; 改:改任官职。 第三类:表示兼职、代理的词语。 兼:同时掌管,兼任; 领:兼任; 代理,暂时担任; 权:暂代官职。 署: 第四类:表示降职、罢免的词语。 贬:降职; 谪:被罚流放或降职; 出:离开京城外调(与“入”相对,古人一般以入京任官为荣); 左迁:降职; 罢、免:罢黜、免去官职; 黜、废:废弃不用。 第五类:其他。 还有一些不表示职官的变动,但常常与职官结合在一起,可灵活翻译为执掌、主持的意思。例如: 知:调清河丞,寻知大谷县(1996年) 行:迁左丞,行徐州事(2000年) 主:掌管。例如《史记?吕太后本纪》:“太尉绛侯勃不得入军中主兵。”(勃:人名。) . 其他 (1)年号纪年。 ?永元中 ?阳嘉元年?永和初 ?元丰七年 ?元和十年?德二年 (2)地名。 ?南阳西鄂人也 ?三辅(京城附近的三个地区) ?河间相 ?京师(京城) (3)学校。 ?太学(最高学俯)(“今诸生学于太学”) ?观、游(指游学,考察学习) ?庠、序(学校)(“俾人邑痒”) ?国子监(明清最高学府,入监读书称监生)(“老监生”) ?教授、博士、直讲、助教(教学人员)(“有司业、博士为之师”) (4)姓名。(“名”和“字”有对应关系) ?张衡字平子 ?屈原名平 ?韩愈字退之 ?苏轼字子瞻 ?苏辙字子由 ?诸葛亮字孔明 ?刘备字玄德 ?关羽字云长 ?张飞字翼德 ?赵云字子龙 辛弃疾字幼安 班固字孟坚 三、自读课文,利用工具书,读准字音,疏通文意,完成下面的题。 )这篇文章是从哪几个方面来写张衡的?文章记叙的重点是什么, (一 (二)翻译下列各句。 1、 虽才高于世,而无骄尚之情。常从容淡静,不好交接俗人。 2、 衡善机巧,尤致思于天文阴阳历算。 3、 衡不慕当世,所居之官辄积年不徙。 4、尝一龙机发而地不觉动,京师学者咸怪其无征。后数日驿至,果地震陇西, 5、 时政事渐损,权移于下,衡因上疏陈事。 《张衡传》导学案(一)答案 三、(一)答:三个方面: (1) 写张衡的品格和文才? (2) 写张衡在科学技术上取得的成就 ( ??? ) (3) 写张衡的政治才干 ( ? ) 第二部分是文章的重点,又可分为两层:一是仕途情况,以及制作浑天仪和著《灵宪》、《算图论》的情况;一是专门介绍候风地支仪。显然,后者又是第二部分的重点。 作者这样处理,使文章的详略得当。因为张衡一生有多方面的才能和成就,而他在科技方面的才能和创造发明最为突出,因此成为本文记叙的重点。 (二)翻译下列各句。 1、 译文:虽然才学高出当时一般人,却没有骄傲自大的情绪。(他)总是举止稳重,神态淡泊宁静,不爱和一般的世俗之人往来。 2、 译文:张衡擅长器械制造方面的技巧,尤其专心研究天文、气象、历法等的推算。 3、译文:他不趋附当时的权贵,所担任的官职,常常多年得不到提升。 4、译文:曾有一次,一条龙的机关发动了,可是(洛阳)并没有感到地震,京城里的学者都惊异地动仪这次没有灵验。 5、译文:当时政治越来越腐败,中央权利向下转移,张衡于是给皇帝上疏陈述关于政事的意见。 16《张衡传》导学案(二) 学习目标 1、了解张衡一生对人类文明发展做出的杰出贡献,学习张衡的高尚品德和刻苦钻研的精神。 2、掌握常用文言实词,弄懂一些专指词语,疏理文言否定词。 3、学习记叙和说明相结合的表达方式。 难点重点 目标2、3、 学习方法 自读法、讨论法、展示法 知识链接 张衡的生平和成就。 张衡(78,139),字平子,南阳西鄂(今河南南阳县石桥镇)人,东汉著名文学家,杰出的科学家。17岁离家,先后到长安、洛阳,拒绝官僚贵族召请,在太学日夜攻读,博览群书。28岁任南阳太守鲍德主簿。29岁写成《东京赋》、《西京赋》。《二京赋》是模拟司马相如的《子虚赋》和班固的《两都赋》写成的。34岁为郎中,后又作过尚书郎。太史令、侍中、河间相一类的官。作太史令最久,前后几次,历时数年,掌管天象观测。写有著名天文著作《灵宪》和《算罔论》,创造了世界上第一架用铜壶滴漏带动的浑天仪。他用肉眼观测星星,制成星图。当时他观察到星星有2500多颗(现代天文学用肉眼能看到3000颗左右)。汉阳嘉元年(132年),又创造了世界上第一台测定地震方向的地动仪。他曾上疏要求禁绝当时流行的图谶迷信。任侍中期间,遭受宦官谗毁,遂作《思玄赋》以宣情述志。59岁离京任河间相,在职之年,“郡中大治,称为政理”。以后又征召为尚书,不久即去世,终年62岁。有《张河间集》存世。 张衡勤敏朴实。崔瑗说他:“敏而好学,如川之逝,不舍昼夜”,“体性温良,仁爱笃密,与世无伤”。他自己在一篇述志的作品里说他是“约己博艺,无坚不钻”,“不患位之不立,而患德之不崇,不耻禄之不伙, 而耻智之不博”。 张衡的文学作品主要是辞赋和诗。《东京赋》、《西京赋》描写洛阳,长安的繁华,讽刺贵族官僚的荒淫无耻,描述了一些民情风俗,表现了作者对封建统治危机四伏的深刻忧虑。他的诗歌以《四愁诗》成就最高。诗分四章,写他的“所思”在“泰山”、“桂林”,“汉阳”,“雁门”等“远道”,但都有障碍,而不能致。以一种情诗的形式寄托了政治的怀抱。它是一首七言诗。在它以前,《诗经》和宋玉的《招魂》,荀卿的《成相篇》等有一些七言句。汉代韵文七言句逐渐增多,但通篇都是七言而又首尾完整的作品,当以这首诗为最早。他也研究过地理学,绘出一幅地形图,流传好几百年。唐代张彦远在《历代名图记》卷四里又把他列为后汉时期六大名画家之一,在数学方面,他对圆周率也很有研究, 在世界史上亦属罕见。万祀千龄,令人敬仰”。 关于浑天仪。 汉时,天文学已经形成体系,分有盖天、浑天和宣夜三家。盖天说以周髀算经为代表,认为天圆地方,天在上,像伞盖,地在下,像棋盘,是一种旧的传统说法。宣夜派认为天体为元气构成。浑天说比较进步些,认为天地都是圆的,天在外,像鸡蛋壳,地在内,像鸡蛋黄;这种说法虽然也不完全正确,但比较接近实际。浑天派最大的代表者和卓越的发明家张衡指出,日有光,月没有光,月光是由日光照射而来的。所以向日则光盈,背日则光尽。他还推测月食是地球遮蔽的结果。他还绘制了一部星图,叫《灵宪图》,创制了许多重要的天文仪器。 浑天仪是铜铸的,内外有几层圆圈,都可转动。各层圆圈分别刻有赤道、黄道、南北极,二十四节气,二十八列宿,以及日月星辰的位置,凡张衡所知道的天文现象都刻在上面。为了使浑天仪能自己转动,张衡又设计了一个“滴漏”,作为浑天仪的动力。浑天仪被滴漏带动,它转动时恰好与天空中日月星辰的起落时间完全吻合。可惜这座精巧的浑天仪在西晋战乱中失传了。留下来的只有《浑天仪图注》和《漏水转浑天仪注》两份说明书的部分说明。 (摘自《中国古代的发明创造》) 学习过程 一、作为传记,它的一般写法是怎样的,本文是怎样写的, 二、传记是记述一个人的生平的,是否意味着事无巨细都要写进文章去呢,本文是怎样处理人物的材料的, 列出它的结构提纲。 三、张衡有哪些可贵的品格, 四、本文在语言运用上有什么特点, 五、学生对照注释自读课文,完成以下自读作业: 1(给下列句中加点的字注音: ?衡少善属文( ) ?不好交接俗人( ) ?累召不应( ) ?五载复还( ) 2(解释下列句中加点的词: ?a(游于三辅( ) b(虽才高于世( ) c(尤致思于天文阴阳历算( ) ?a(因入京师,观太学( ) b(„„作《二京赋》,因以讽谏( ) c(伺者因此觉知( ) ?a(公车特征拜郎中( ) b(京师学者咸怪其无征( ) ?a(精思傅会,十年乃成( ) b(以精铜铸成( ) 3(翻译句子,指出各句包含的语法现象。 ?永元中,举孝廉不行,连辟公府不就。 ?大将军邓骘奇其才,累召不应。 ?饰以篆文山龟鸟兽之形。 ?自书典所记,未之有也。 六、回答下列问题。 1(张衡的高尚品德是什么,联系上下文说一说,何以见得“才高于世”,其“从容淡静”又表现在哪里, 2(张衡最杰出的贡献是什么,课文重点记叙的内容是什么, 3(课文的语言质朴简练,你是怎样体会的, 4(和《屈原列传》相比,这两篇传记最大的共同点是什么,最大的区别是什么, 《张衡传》导学案(二)答案 一、分析:按照传记的体例,开头介绍人物的姓名、籍贯,如《廉颇蔺相如列传》、《柳敬亭传》都是。本文也是开头介绍姓名、籍贯,结尾写到他死的年代。所以本文是完全按照一般传记体例的写法来写的。 二、分析:传记记述一个人的生平,但并不等于也不可能把一个人一生的全部经历、事无巨细都写进文章去,而只能记述人物一生中的重要事件,同时显示出这个人的品格特点。 本文先介绍了张衡的品格和他卓越的文学才能;重点写他一生中最突出的辉煌的科学成就,而在科学成就中尤其重要的是他首创的候风地动仪,最后介绍了他政治上的才干。文章这样处理材料,就使重点突出,主次分明。 结构提纲—— 第一段(第1自然段):介绍张衡的品格和卓越的文学才能。 第二段(第2、3、4自然段):介绍张衡辉煌的科学成就。 (1)张衡的潜心科研 (2)张衡的不慕名利 (3)张衡的重大发明 自然段):介绍张衡杰出的政治才干。 第三段(第5 三、分析:他的可贵品格可从两个方面来看。 一是“从容淡静”“不慕当世”。他不慕名利,先是“举孝廉不行”、“连辟公府不就”、“累召不应”,以后做了官,哪怕“所居之官辄积年不徙”他也无所谓,由此反复说明了张衡的高洁。不仅如此,他还“不好交接俗人”,不满当时“王侯以下”的“逾侈”的风气,经十年苦心构思写成《二京赋》用以讽谏;一上任河间相,立即“治威严,整法度”,将图谋不轨的“奸党”“一时收禽”,都反映了他对当时奢靡庸俗风尚的不满和对不轨行动的斗争。这是他从容淡静、不尚虚浮的品德。 二是“无骄尚之情”。他年少时便“善属文”、“通五经”、“贯六艺”,全面发展,甚至“才高于世”,却始终“无骄尚之情”,这确是难能可贵的。正是由于不尚虚浮而又谦逊踏实的品德,他才能把全副精力扑在科研工作上,在获得两次担任太史令、有机会观察天象博览群书的客观条件之后,浑天仪、《灵宪》、《算罔论》,乃至候风地动仪得以相继问世。 四、分析:本文在语言运用上主要有两个特点:第一,记叙语言非常质朴通俗。例如,“不行”、“不就”、“不应”、“不慕”等句用语非常质朴通俗,毫无雕饰,却鲜明地表现出了张衡不追名逐利的高尚品德。第二,说明语言十分准确简洁。例如,第4段中仅用140多字就将候风地动仪的有关情况作了比较全面的介绍,可见用语简洁;用酒樽描摹仪器的外形,用“中”、“傍”、“外”、“下”等指明部位,足见其用语之准确。 五、2(?于,介词。a(表处所,在;b(表比较;c(表范围,在„„方面; ?因,介词。a(趁(机会);b(通过、藉;c(因为; ?a(召,动词;b(征象,证据,名词。 ?a(精心,形容词;b(纯,形容词。 3(?永元年间,被推荐为孝廉,(他)却不去应荐,多次被公府征召,(他)也没有去就任。——“举”、“辟”为被动用法,“不行”、“不就”省略主语“他”,即张衡。“连辟公府”是“连辟于公府”,省略“于”。?大将军邓骘认为他才能奇特,累次召请(他),(他)没有应召。——“奇”,形容词的意动用法。省略“召”的宾语和“不应”的主语(即兼语)“其”。?用篆字山、龟、鸟、兽等图案雕饰。——状语后置。?自从有书典记录以来,不曾有过这样的事。——“所记”,“所”+动词、组成名词性结构。“未之有”即“未有之”,否定句代词宾语前置。 六、明确:1、张衡的高尚品德表现在:(1)“虽才高于世,而无骄尚之情。”写作上“精思傅会”,科学上“致思”“研核”,孜孜不倦,贡献卓越;(2)“从容淡静”,不慕世俗的虚荣,“举孝廉不行,连辟公府不就”,大将军“累召不应”,“不慕当世”;(3)品行端正,忧国忧民,作《二京赋》讽谏王侯,整治法度收禽奸党。 张衡“才”高于世,表现在:一、“善属文”,作《二京赋》;二、“善机巧”,作浑天仪、造候风地动仪;三、“善术学”,著《灵宪》、《算罔论》;四、善政事,为政机智果断,不畏权贵,称为政理。 2、张衡是一位博学多才的伟大学者。他最杰出的贡献是研制成功史无前例的候风地动仪。课文重点记叙的是张衡在自然科学方面的伟大贡献,因而对候风地动仪作了详细的介绍。 3、课文语言质朴简练。如“衡少善属文,游于三辅,因入京师,观太学,遂通五经,贯六艺。”短短二十三字,就写出了张衡的学历、水平和特长。“„„不行,„„不就,„„不应。”充分表现了张衡不慕名利的品德。“衡下车,治威严,整法度,阴知奸党名姓,一时收禽。”节奏感很强的语言,表现了张衡的态度果断,措施得力。 4、和《屈原列传》相比,两篇传记最大的共同点都是按时间顺序,粗线条地勾勒人物的一生,选材有详有略,重点突出。最大的区别是在表达方式上,《屈原列传》为了揭示屈原的“志”、“行”和寄寓作者的情怀,采用了叙议结合的方法,而《张衡传》为了表现张衡的杰出贡献,用说明性文字对张衡的发明候风地动仪作了较详细的介绍。 17.表达交流,,善于思辨 学习辩证分析 一、学习目标 培养辩证唯物主义思想,重视辨证地看问题,学会运用联系的观点、发展的观点、矛盾的观点实事求是地分析问题。 二、学习重点 培养严密的思维能力和提高作文说理能力,能够善于运用辩证的观点分析问题,得出合乎事理的结论。 三、课时:1课时 四、知识链结及应用: 一、辩证分析的基本特征 事物是纷繁芜杂的,我们认识事物、分析事物不能简单化,说理论证要辩证,就是要看到事物的各个侧面以及事物之间的内在联系,全面地分析事物的矛盾,从而对事物作出正确的、合乎事理的评价。 辩证分析是议论文析理应贯彻的一项重要原则。辩证分析就是在分析问题、阐释道理时要有实事求是的态度,客观地、全面地分析事物,学会用发展的、联系的眼光,由此及彼,由表及里地深入分析,抓住问题的实质。辩证地分析应力避说“过头话”和偏执一词,切忌孤立地、静止地看待和分析问题。 二、辩证分析的三个基本观点与运用举例 辩证分析,就是要从事物的两方面进行辩证思维,或一分为二认识事物,或从比较异同思索,或从侧面与全面、个别与整体的关系思考,都有助于全面认识,从现象到本质揭示事物特征,提出准确全面的观点或理由。一篇好的议论文,绝不能宣传片面的、孤立的、绝对化的观点。平时多学习好的议论文,认真研究这些文章怎样对问题进行辩证的分析。 一般情况下,对议论中心进行辩证分析,通常要注意以下哲学观点的运用: (一)用发展的观点分析问题 世上的万事万物都是在运动、发展、变化之中。论证一个问题的时候,如果采用静止不变的观点分析, 就不可能揭示出它的内在的客观规律,就必然违背事理;只有抓住事物之间的普遍联系,在发展中分析问题,才能把握问题的实质所在。例如下面这篇题为《世界每天一个样》的文章: 近日,一位心烦的母亲上门“诉苦”,说自己的孩子已初三了,成绩依旧在四五十分之间徘徊。谈话中,她用得最多的话就是:“他还有什么前途,” 说实在的,第一次听到这句话,我感到同情,为她伤心;第二次听到这句话,我感到难过,为她的孩子竟连母亲的信任都得不到;而当第三次听到这句话时,我感到的已是一种反感情绪了。 明天就一定和今天一样吗, 有一则关于巴尔扎克的故事,说他看了一个小学生的作文本后,根据这小学生书写迟钝、笨拙,做出了这孩子将来出息不大的结论,但有趣的是这正是他自己小时候的作文。 从这一点上,这位母亲和巴尔扎克的错误是一样的——静止的一成不变的狭隘观点。 世界上的一切事物都处在永不停息的运动、变化、发展过程中,科学发展史表明,地球有一个产生、发展的过程,太古代、远古代、古生代、中生代、新生代„„社会发展史表明,社会有一个产生、发展的过程,原始社会,经过了多少万年的发展才进入奴隶社会,此后是封建社会、资本主义社会和社会主义社会乃至共产主义社会。发展和变化是永无止境的。人不也是如此吗,身体的发育,思维的形成,世界观、人生观的更改,知识体系的深化,在这之中谁能保证,自己一定会变得怎么样, 昨日种种,那不过是今天的起点;明日种种,还要看今日的汗水几何。 狭隘的一成不变的观点是一种腐蚀剂,消磨了意志,消磨了自信,也就消磨了前进的动力。连动力都丧失了,你的明天就真的销毁了。从这点看,我很庆幸巴尔扎克小时侯没有静止的一成不变的狭隘观点,否则人间就不会有《人间喜剧》了。 年年岁岁花相似,岁岁年年人不同。这位伤心的母亲,也请你想一想爱因斯坦、爱迪生、华罗庚,他们小时侯读书并不聪明,兴许也和您的孩子一样,可为什么他们成了世界闻名的科学家、发明家、数学家,因为自信、坚持和发展。别再叹息了,你的孩子需要的是鼓励。请相信,一切都会改变。 请运用发展的眼光对此材料给予评论: (二)运用联系的观点分析问题 任何事物都不是孤立存在的,它总是和外界事物有着千丝万缕的联系。分析一个问题时,就要注意它和其他有关问题的联系。如下面这篇文章: “象棋”折射中西文化 中国与外国都有象棋,它们的祖先虽然都为古印度象棋,但经过数千年的演化,早已大相径庭了。有趣的是稍稍观察规则,竟能发现不同的民族、国家在文化、政治上的区别,实在发人深思。 凡略通棋术者一定知晓,国际象棋最厉害的是“后”,它可以直飞、横飞、斜飞,真可谓“天马行空,独来独往”。其他棋子的战斗力远远弗如。由是可见,外国历来不“重男轻女”,女子照样可以“爵禄高登”,如现实生活中的英国女王。而中国的礼教早在几千年前就规定了妇女的地位——“女子无才便是德”,只配做贤妻良母。武则天历尽艰辛登上了皇帝的金銮宝座,把国家治理得井井有条。死后,仍落个千古骂名,原因就在于“大逆不道”~唯其如此,中国象棋上就没有女性的一席之地。 中国象棋在底线中央规定了一个四方框,将、帅只在框里活动,这大概是因为中国皇帝向来只在宫中度日,不能越雷池一步,除非国势危急,断不“御驾亲征”的缘故。但在外国,皇帝领兵打仗却是家常便饭,如英王查理为了平叛,亲自带兵去法国打仗。国际象棋中的“后”威力之大,是与此有关的。 有道是,“过河的兵顶个车”。诚然,中国象棋里的卒只要越过“楚河”“汉界”便可东南西北“任遨游”。殊不知,一旦沉底,就几乎失去了战斗力而任人宰割。这表明中国古代军人为封建朝廷拼了一辈子的命,到头来只会落个悲惨下场。即使韩信这样侥幸从乞丐到将军的人,也只是汉高祖手中的一件“工具”,定国安邦之后,难逃劫数,被处以极刑身亡。在国外则不同,只要士兵在战斗中勇往直前,建立功 勋,那么就有机会“平步青云”。当年拿破仑由普通士兵成为皇帝的史实就是证明。所以,国际象棋中的小兵到达终点线,就立即成了“后”,威力陡增。 象棋这一方寸世界乃是人类社会各不一样的大千逸事之缩影,给人启迪不浅。 请用联系的观点对此材料进行分析: (三)运用一分为二的观点分析问题 用一分为二的观点去分析问题,才能全面地认识问题,避免认识的片面性。分析问题时,既要看到它的这一面,又要看到它的另一面;既要看到它的正面,也要注意它的反面。 勇士耶,莽夫耶, ——评漂流的价值 自尧茂书单人漂流长江不幸遇难之后,越来越多的人涌向那每个炎黄子孙心目中的圣地——青海省海拔6621米的各拉丹冬雪山——亚洲第一大河长江的源头所在。为了骄傲地说一声:“是中国人首先征服长江~”他们没有被前仆者的牺牲所吓倒,也没有畏缩于虎跳峡那几十米的落差,毅然投入这中华民族的大动脉中。 漂流壮举,在全国引起极大反响,褒贬各异,称赞者认为这是为国争光。批评者则认为是冒险活动,寻求刺激。我对它的看法是:“赞赏其精神,不解其行动。” 首先,我们不能把漂流单纯地看作是年轻人寻求刺激。试想一只古老的皮筏,载着血肉之躯,与“惊涛拍岸,卷起千堆雪”的长江搏击,此等壮举,难道是一般人所能为之吗,他们是要为中国人争一口气呀~在过去那段令人辛酸的日子里,外国的炮艇轮船在长江上肆意横行,川江航道就是英国人首先打通的。国耻呀~现在,美国人又宣布要首漂长江,于是,以尧茂书为首的一批勇士站起来了,前仆后继,终于完成第一漂的壮举。所以,他们的精神实在可敬,能够催人奋进。 但即使是最支持漂流的人,我想,在看到那些伤亡报告的时候也不禁会摇头叹息:“代价是不是太高了”这是一场战争吗,是的,是一场人与自然的战争。而作为20世纪的80年代的现代人,所使用的武器是什么呢,说来令人难以相信,是古老的皮筏。这场战争只能称为人以原始的蛮力与大自然的搏斗,因为人把他最具有威力、最强大的武器舍弃了,那就是人的智慧。就身体各方面的因素而言,人在自然界中是弱者。但是人类还是生存下来了,并成为世界的主宰。这一切,都是因为人能够思维,具有智慧。而漂流的勇士们却似乎没有意识到这一点。看着那小小的皮筏被巨浪肆意戏弄,我不禁疑惑,这难道就是曾经遨游过太空与深海的人类吗,此为我不解之一。 那么之二呢,乃是我对漂流者过虎跳峡的方式感到吃惊。一个橡皮艇,密封的,四周捆上一些轮胎,人就在艇里,任激流冲打,里面的人根本无法掌握自己的命运,一旦漏水,使意味着死神降临;否则,功成名就,称为征服长江的英雄。试问,这行动与昏天黑地的赌博者有何区别,(请原谅我用这个词,很明显,两者的目的截然不同,前者是高尚的。)如果说坐在皮筏里靠船浆与长江搏斗是使用蛮力,那么躲在皮艇里漂流又算什么呢,惭愧呀~ 诚然,漂流者在精神上是足以称之为勇士,而在行动上呢,只能算是莽夫。进一步想,目前急待漂流的果真是长江、黄河吗,我们今天所从事的伟业不也像一条待漂的长河吗,朋友们,凭着自己的才华和胆略,用自己的智慧和力量到改革的洪流中去搏击吧~在这激流中,我们也能成为漂流的英雄~ 运用一分为二的观点分析此材料: 五、实战演练: (一)请以“逆境与成才”话题为文,运用辩证分析 春天到了。两颗种子躺在肥沃的土里,开始了下面的对话。 第一颗种子说:“我要努力生长!我要向下扎根,还要‘出人头地’,让茎叶随风摇摆,歌颂春天的到来„„我要感受春晖照耀脸庞的温暖,还有晨露滴落花瓣的喜悦。”于是它努力向上生长。 第二颗种子说:“我没那么勇敢。我若向下扎根,也许会碰到硬石。我若用力往上钻,可能会伤到我脆弱的茎。我若长出幼芽,难保不会被蜗牛吃掉。我若开花结果,只怕小孩子看了会将我连根拔起。我还是等情况安全些再做打算吧。” 于是它继续瑟缩在土里。 几天后,一只母鸡在庭院里东啄西啄,这颗种子就这样进了母鸡的肚子。 两颗种子为什么会有不同的结局,读了这段文字,你一定有所感悟,有所思考„„请根据你的感悟和思考所得,自拟标题,写一篇议论文,不少于600字。 六、拓展与练习(例文展示): 逆境与成才 人们无不向往美好的环境,因为好的环境能使成才的道路畅通无阻;同时,人们也认识到,“自古雄才多磨难,从来纨绔少伟男”,顺境造就幸运儿,而逆境往往造就伟人。这是因为,顺境虽说给人的成长铺平了道路,但逆境更能催人奋进。 历史事实证明,厄运能磨炼人的斗志。身遭腐刑摧残的司马迁,就是以身处逆境而自强不息的历代先贤为楷模,他凭惊人的毅力完成了被鲁迅先生誉为“史家之绝唱,无韵之《离骚》”的皇皇巨著《史记》,成为史家的不祧之祖,我国力学、桥梁专家李国杰教授,在“文革”期间身居“隔离室”,经常受审讯,在这恶劣的环境里,他顶住各方面的压力,写出了10万字的专著《 梁扭转理论—— 梁桥的扭转、稳定和振动》,填补了一项世界桥梁建筑学上的空白。司马迁说:“文王拘而演《周易》,仲尼厄而作《春秋》,屈原放逐乃赋《离骚》,左丘失明著《国语》,孙子膑脚修兵法,韩非囚秦著《说难》„„”可见“艰难困苦,玉成于汝”,确实有道理。 但是,逆境能够出人才,绝不意味着逆境一定出人才。一个人能够成才,关键在于自身主观上是否努力。巴尔扎克说得好,苦难对于天才是一块垫脚石,对能干的人是一笔财富,对弱者是一个万丈深渊。同样是贫寒的生活环境,贝利经受困难的磨练,成为一代球王,而有些人却堕落为小偷;同样是身有残疾,张海迪成了中国当代青年学习的楷模,而有些人却沦落为乞丐;同样是国民党反动派的镣铐和铁窗,江姐成了坚贞不屈的烈士,甫志高却成为可耻的叛徒„„可见,面对逆境,丧失奋斗勇气和决心的人只能是苦苦呻吟、怨天尤人,甚至自甘堕落;只有“扼住命运咽喉”的人,才能“贫贱不能移,威武不能屈”,超越任何障碍,战胜任何困难,成为堪担大任的“大丈夫”。 我们希望整个人生都一帆风顺,但对于绝大多数人来说,这是不可能的。如果遭遇逆境,应该怎么办,不要动摇,不必气馁,积极进取,奋力开拓,这,就是我们应取的态度。 (选自《高二作文老师来示范》) 【例文简析】 这篇议论文阐述了逆境与成才的关系,分析辩证。首段分别从顺境与逆境之于人才的关系进行理论分析:“顺境造就幸运儿,而逆境往往造就伟人”。本论部分分两层展开具体分析。第一层用典型的历史事实证明“厄运能磨练人的斗志”这一分论点。第二层用引证法和正反对比举例的方法论证“逆境能够出人才,绝不意味着逆境一定出人才”这一分论点,并指出:成才与否关键在于自身主观上是否努力。结论部分提出遭遇逆境的应对态度:“不要动摇,不必气馁,积极进取,奋力开拓”。全文结构严谨,是典型的总分总式结构,分述部分,采用转折递进式的层次布局,条理清晰。中间两自然段也都采用总分总式的结构,段首表明分论点,段中举例论证,段尾扼要小结,叙例简洁,析理精当,井然有序。文章不作片面的议论,作者既肯定顺境之于成才的利处,也不否定逆境能够出人才;既指出逆境能催人奋进,又强调人的主观因素:面对逆境,不可丧失奋斗勇气和决心。这样辩证地分析说理,比较中肯、缜密,具有较强的说服力。 18.《辩论》导学案 【学习目标】 1(通过活动锻炼辩论的能力。 2(培养独立思考、辨证地分析问题的能力。 3(培养摆事实、讲道理的科学精神;培养合作的精神。 4(提高思想修养和语文修养,培养正确的人生态度与价值观。 【学习重点】 培养学生“敏捷的才思、流畅的表达”能力 【学习难点】 如何立论、使论辩具有幽默特点。 【学习课时】 1课时 【学习方法】 讲读与口头练习相结合。 【学习过程】 一、辩论中的立论战术 辩论是由立论(辩护)和反驳两个基本环节构成的,其中立论就是为了证明己方的基本立场,它是反驳的基础和必要的阶梯。辩论中如果没有必要的立论,反驳就会显得强词夺理,苍白无力,而且,辩论中如果自己的立论不稳,自然会被对方攻击得只有招架之功,更谈不上对对方的攻击了。可见,立论的好坏,直接关系到辩论的成败。因此,辩论中要特别注意加强立论的力度。下面简单介绍几种立论战术: 逻辑严密,框架严整 立论中,运用严密的逻辑思维,构建严密无懈的理论框架,从而使自己的立论坚实。严谨,无任何漏洞可寻,这是使辩论获胜的关键。 如“万家乐杯”电视辩论大赛上,北京大学队与国际关系学院队的辩论题目是:“我国现阶段应该鼓励私人购买轿车。”这一题目的关键是“轿车”“鼓励”和“我国现阶段”这三个词。如何找准这三者之间的逻辑关系,从而形成一条强有力的立论思路,这是能否构建严密的攻防体系的关键。最后,作为正方的北大队根据其内在的逻辑联系推导出了这样的思路:现阶段发展轿车工业是我国工业发展的主导方向之一。由于轿车工业“三高一快”的特点,轿车工业被证明是经济起飞最有力的助推器,轿车的质量和产量也是衡量一个国家发展水平高低的标志。我国也不例外(要想促进工业发展,必须发展轿车工业。其次,轿车工业要发展,关键在市场。如何扩大轿车市场,最便捷的办法是使轿车“飞人寻常百姓家”。所以,轿车工业同鼓励私人购买就存在着必然的联系。在此基础上,他们再依据其必然的逻辑联系充分论证了“鼓励购买”的现实可能性和必要性,并充分考虑了对方立论中可能会提出的问题(即我国的公路交通的拥挤憎况,轿车的私人消费是否会是一种奢华的超前消费倾向),并对此一一作了周密合理的论述准备。由于北大队在立论中充分运用严密的逻辑思维来确立自己的论证体系,确保了该体系的严整周密,所以他们的立论在实践中既立得起,又防得住,收到了较好的效果。 出其不意,“破”中求“立” 辩论,说到底是一种知识、智谋的较量,辩论的一方在立论时如能充分运用自己的知识和智谋,在透彻地分析辩题的基础上,突破对方立论的防线,巧妙地提出一个全新的概念,给对手一个“措手不及”,这样便能大大削弱对方的攻击力。 如94长虹杯全国大学生辩论赛南京大学队迎战吉林大学队,吉大队作为正方的立场是:大学毕业生择业的首要标准是发挥个人专长。南大队作为反方其立论思路有很多,比如可以说“首要的标准是社会需要”,也可说“是收入丰厚”“是兴趣”等等,但所有这些都是因为太平常而可能落人吉大队事先准备好的猛烈进攻中。你说“社会需要”,他讲择业是主观行为,“发挥个人专长”正是更好地满足“社会需要”,你说“收入丰厚”,他说:对方辩友在养育自己的祖国最需要的时候,以一己私利为先,向人民讨价还价,多么让人痛心和失望~如此,南大队将难以招架。最后,南大队经过缜密的思考,提出了一个极其大胆的观念:大学生择业复杂多样,没有也不应该有一个统一的首要标准~并指出,没有证明大学生择业应当有一个统一的首要标准,就去强调这个首要标准是“发挥个人专长”,这无疑于在流沙上盖楼。此语一出,举座皆惊。由于南大队的观点从根本上动摇了对方精心设计的立论,吉大队毫无准备,顿时乱了阵脚,以致在规范性发言中几乎未对此进行反驳。南大队在以前所未有的创新勇气击破对方的同时,又进一步明确了自己的立论:大学生应以个人的自我完善和推动社会进步为择业方向。如此一来,南大队便很快占据了场上的主动,收到了十分明显的场上的效果。 另辟蹊径,李代桃僵 当辩论中碰到一些在逻辑上或理论上都比较难辩的辩题时,在立论过程中就不得不采用“李代桃僵”的办法,引入新的概念来化解困难。 比如“艾滋病是医学问题,不是社会问题”这一辩题就是很难辩的,因为艾滋病既是医学问题,又是社会问题,从常识上看,是很难把这两个问题截然分开的。复旦大学在处理这个问题时,首先做了以下设想:如果让他们去辩正方的话,他们就会引入“社会影响”这一新概念,从而肯定艾滋病有一定的“社会影响”(但不是“社会问题”,并严格地确定“社会影响”的含义,这样,对方就很难攻进来。后来在辩论时他们却抽到了反方的签,要阐述“艾滋病是社会问题。不是医学问题”,在这种情况下,如果完全否认艾滋病是医学问题,也会于理太悖,因此,他们在辩论中引入了“医学途径”这一概念,强调要用“社会系统工程’’的方法去解决艾滋病,而在这一工程中,“医学途径”则是必要的部分之一。这样一来,他们的周旋余地就大了,对方得花很大气力纠缠在他们提出的概念上,其攻击力就大大地弱化了。“李代桃僵”这一战术的意义就在于引入一个新概念与对方周旋,从而确保己方立论中的某些关键概念隐在后面,不直接受到对方的攻击。 二、讽刺幽默生情趣 在辩论中,反驳对方,有时不采取锋芒毕露、相互评击的语言,而采用风趣含蓄、诙谐生动的语言,其效果会更好,更具有说服力。 在论辩中,讽刺幽默显然是不同于证明与反驳的,它既无论辩的过程,也没有反驳的程序,而是以谐趣的方式,揭开荒唐的外衣,暗示事物的本质,达到明辨是非的目的,从而在论辩中达到很好的效果。 讽刺幽默很讲究技巧,不是出言不逊,恶语伤人,在运用讽刺幽默时应十分注意这一点。 【典型示例】: 例一 传说汉武帝晚年很希望自己长生不老。一天,他对待臣说:“相书上说,一个人鼻子下面‘人中’越长,寿命就越长;‘人中’长一寸,能活一百岁,不知是真是假,” 东方朔听了这话,知道皇上又在做长生不老之梦了。皇上见东方朔似有讥讽之意,面有不悦之色,喝道:“你怎么敢笑话我,” 东方朔脱下帽子,恭恭敬敬地回答:“我怎么敢笑话皇上呢,我是在笑彭祖的脸太难看了。” 汉武帝问:“你为什么笑彭祖呢,” 东方朔说:“据说彭祖活了八百岁,如果真像皇上说的,‘人中’就有八寸长,那么,他的脸不是有丈把长吗,” 汉武帝听了,也哈哈大笑起来。 【评析】 在这个故事中,东方朔以幽默的语言,用笑彭祖的办法讽刺汉武帝的荒唐。东方朔的批驳机智含蓄、风趣诙谐,令正在发怒的皇帝也不禁要哈哈大笑起来,并且很愉快地认输。 例二 1946年5月,远东国际军事法庭审判以东条英机为首的28名日本甲级战犯,曾因排定法庭坐次,10个参与国的法官们展开一场激烈的争论。中国法官理应排坐在庭长左边的第二把交椅,可是由于当时中国国力不强,而被各强权国所否定。在这种情况下,唯一出庭的中国法官,面对列强展开了一场机智的舌战。他首先从正面阐明,排座次应按日本投降时各受降国的签字顺序排列,这是唯一正确的原则立场。接着他微微一笑说:“当然,如果各位同仁不赞成这一办法,我们不妨找一个体重测量器来,然后以体重大小排座,体重者在中,体轻者居旁。”各国法官听了忍俊不禁地笑了。庭长笑着说:“你的建议很好,但它只适用于拳击比赛。”梅法官接着回答说:“若不以受降国签字次序排座,那还是按体重排好。这样纵使我被置末位亦心安理得,并且可以以此对我的国家有所交待,一旦他们认为我坐在边上不合适,可以派一名比我肥胖的来换我呀。”这回答引得法官们大笑。 【评析】 在举世瞩目的国际法庭上,法官的座次是按体重来排定,这岂不是天大的笑话~中国法官正是用这样的笑话辛辣地嘲弄帝国主义分子及其走狗倚仗强权,践踏国际公理的丑恶嘴脸。在这种邪恶压迫正义,谬论强制公理的险恶形势下,也许证明与反驳都很难奏效,幽默才是矫正和诛伐的最有力的武器。 三、实践活动 下面是一组辩论题目,请你从中选择一组,从正方的观点或者反方的观点出发,写一段辩论词。 正方 反方 个人的命运是由个人掌握 个人的命运不是由个人掌握 发掘人才需要考试 发掘人才不需要考试 民族技艺应该保密 民族技艺不应该保密 理想人才以仁为主 理想人才不一定以仁为主 金钱的追求与道德可以并行 金钱的追求与道德不可以并行 个性需要刻意追求 个性不需要刻意追求 通俗文学比文学名著影响大 文学名著比通俗文学影响大 19 逻辑和语文学习 1 《逻辑与语文学习》导学案 学习目标: 1(了解逻辑的一般知识。 2(掌握辨析概念和运用推理的一般方法,提高语言运用能力。 3(培养学生独立思考、深入探究的习惯,锻炼学生的思辨能力和表达能力。 学习重点: 组织学生开展探究式学习,掌握概念、命题、推理和逻辑规的一般知识。 学习难点: 将自发形成的思维习惯转化为自觉的合乎逻辑的思维方式。 学法指导: 点拨法、讨论法、探究法。 知识链接: “逻辑”一词~源于古希腊文“逻各斯”~原意是理性、规律、思想、理智等等。后来~ 随着古希腊社会的发展以及由此而来的社会大论战的兴起~产生了一门专以推理、论证为研 究对象的学科~古希腊学者就用这个词来给这门科学命名。在传统上,逻辑被当做是哲学的 一个分支,和文法、修辞一同被称为古典三学科。 逻辑学是一门历史悠久的科学,从公元前五世纪前后开始,就分别产生于古代希腊、中国和 印度。在古希腊,逻辑学的建立以亚里士多德的《工具论》为标志。中国古代逻辑学是在春秋战 国时期产生发展起来的,战国时后期墨家的《墨经》是第一部比较系统地论述逻辑理论的著作, 所以中国古代逻辑又称“墨家逻辑”。在印度,逻辑学称为因明学,总结古印度逻辑思想的第一部 代表作是《正理经》。相比较而言,古希腊学者对逻辑理论的研究和建树更全面、更系统。因而在 世界逻辑学发展史上影响也最大、最深。 古希腊学者亚里士多德(前384—前322年)被认为是古希腊逻辑学的创始人,他在著作《工 具论》中,第一次全面、系统地论述了传统形式逻辑,为逻辑发展史树起了第一座丰碑。 从19世纪中叶到20世纪初,经过英国数学家布尔、德国数学家弗雷格、英国哲学家、 数学家罗素等人接连不断的努力,吸收莱布尼兹的成果,建立了后来作为电子计算机理论 基础的“正统数理逻辑”的现代公理系统,这是逻辑学发展史上的第二座里程碑。 学习过程: 一、知识梳理 (A级) 1(从逻辑的角度看,“概念”揭示了某一事物或现象的 ,我们凭借概念可以准确地把 一种事物或现象与其他事物或现象区别开来。 2(每一个概念都有自己的内涵和外延:内涵是概念所反映的对象的 ,外延则指概念所 反映的对象的 。 3(命题是 ,是断定或陈述事物情况的思维单 位。 4(推理是 的思维过程,往往要通过 的语言形式来体现。 5(逻辑规律主要有“同一律”“矛盾律”和“排中律”。 同一律是指在同一思维过程中,所使用的概念和命题必须始终保持 ,不能中 途 , 。 矛盾律是指在同一思维过程中,针对同一个对象,两个互相矛盾的命题能 ,肯 定一个,就必须 ,不能两个都肯定。 排中律是指在同一思维过程中,两个互相矛盾的命题不能 。 二、课堂训练 (B级) 1.指出下列各组概念间的关系。 (1)A.看 B.瞧 (2)A.瞥 B.看 (3)A.瞥 B.仰视 (4)A.俯视 B.仰视 2(下面的语句能表达命题的是( )(多选题) A.只有年满18岁才有选举权。 B.能不忆江南, C.多么蓝的天哪~ D.请把门关上。 E.你去过杭州吗, 3.请指出下列句子的逻辑错误,并加以改正。 (1)他是众多死难者中唯一的幸存者。 (2)行车速度的快慢是造成这次车祸的重要原因。 (3)他因为捍卫大家的利益,勇敢地同歹徒搏斗。 三、探究释疑(C级) 1(《韩非子•难势》中有一则众所熟知的寓言:有一楚人在街上卖矛和盾。他先吹嘘自己的盾说:“我的盾坚固无比,任何东西都不能刺穿它。”过了一会儿又吹嘘自己的矛:“我的矛非常锋利,任何东西都能刺破。”旁边有人讽刺地说:“那么,用你的矛刺你的盾又如何呢,”卖矛和盾的人顿时无言以对。 这个故事违反了什么逻辑规律,请简要分析。 ____________________________________________________________________ ______________________________________________________________________________________ __________________________________________________ 2(据说有一次,邻人偷了华盛顿家里的一匹马。华盛顿同一位警官到邻人的农场里去讨索,但那人拒 绝归还,并声称那是他自己的马。华盛顿用双手蒙住马的两眼,对邻人说:“如果这马是你的,那 么,请你告诉我们,马的那只眼睛是瞎的,” “右眼。” 华盛顿放开蒙右眼的手,马的右眼并不瞎。 “我说错了,马的左眼才是瞎的。”邻人急着争辩说。 华盛顿放开蒙左眼的手,马的左眼也不瞎。 “我又说错了„„”邻人还想狡辩。 “是的,你错了,”警官说,“证明这马不是你的,立即把马交还给华盛顿先生吧~” 这个故事违反了什么逻辑规律,请简要分析 _____________________________________________________________________ ______________________________________________________________________________________ __________________________________________________ 四、学习反思: 逻辑对语文学习有什么作用,结合自己的学习的实际情况谈谈看法。(D级) 五、布置作业:(C级) 1、有一个残暴的国王,统治着一个奇怪的国家。这个残暴的国王颁布了一条奇怪的法令,所有从前线 抓回来的俘虏都要回答这样一个问题:你来这里做什么,如果答的是真话,就得用火烧死,如果回 答的是假话,就得绞死。看起来,任何俘虏都难逃噩运。 一个聪明的俘虏巧妙地回答了这个问题, 国王最后只好无可奈何地将俘虏释放了。你知道这名俘虏是如何回答的, _____________________________________________________________________ 2、珠宝店一块钻石被窃,据侦察,作案的是甲、乙、丙、丁中的一人。在审讯中,四人的口供如下: 甲:钻石不是我偷的,我那天在外地。 乙:钻石是丁偷的,那天,他去过珠宝店 丙:乙偷的,我看见他到黑市去卖过钻石。 丁:乙与我有私仇,他有意陷害我。 现在,我们假定这四人中只有一个人讲真话,那么罪犯是谁,你知道吗, _______________________________________________________________________ 拓展阅读: 逻辑学对现代科学发展的作用 蔡曙山 从逻辑、数学和其他学科的关系看,并非所有的学科都要使用数学,而所有的学科都必须使用逻辑。因此,逻辑学应该成为众学科的基础。数理逻辑建立以后,作为学科基础和各学科共同工具的逻辑理论融入现代科学各学科之中。逻辑学理论在现代科学各学科的应用一方面促进了逻辑学理论自身的发展,另一方面也促进了现代科学的其他学科发展。 1968年,尼古拉斯?雷歇尔所作的逻辑分类图和1995年特德?杭德里奇所作的逻辑分类图展示了逻辑学理论在现代科学的应用中得到的新发展。从他们的分类可以看出逻辑学若干新的进展,例如在逻辑学自身理论的发展方面,形成了元逻辑这一学科群体(包括逻辑语形学、逻辑语义学、逻辑语用学等分支学科);在数学的发展方面,形成了数理逻辑学科群体(包括算术理论、代数理论、函数论、证明论、概率逻辑、集合论等分支学科);在科学的发展方面,形成了物理学的逻辑、生物学的逻辑、社会科学的逻辑等学科门类(包括量子论逻辑、物理或因果模态理论、控制论逻辑、义务逻辑、价值逻辑、法律逻辑等分支学科);在哲学的发展方面,形成了哲学逻辑广大的学科群体(包括行为逻辑、义务逻辑、命令逻辑、选择逻辑、搏弈和决策的逻辑、存在逻辑、时态逻辑、变化逻辑、过程逻辑、部分,整体逻辑、本体论逻辑、构成主义逻辑、问题逻辑、认知逻辑、条件逻辑、信息和信息加工的逻辑、证据和证实逻辑、概率逻辑等众多的分支学科)。 逻辑学在现代科学中的应用也促进了现代学科的发展,产生了各学科之下或各学科之间的更多的新兴学科。在哲学方面的发展方面,分析哲学和日常语言学派是以数理逻辑为工具建立起来的。在物理学的发展方面,最典型的例子是量子力学的建立。冯?诺伊曼说,量子力学完全是用公理化方法建立的。他认为,量子力学系统是希尔伯特空间中的一种算符运算公理系统,它只有5条公理。在语言学的发展方面,典型的例子是乔姆斯基的转换生成语法、克里普克的语义模型,以及从奥斯汀到塞尔所发展的言语行为理论和语用逻辑。在计算机科学的发展方面,没有数理逻辑就没有现代计算机,因为现代计算机的原型冯?诺伊曼机的逻辑基础就是经典的二值逻辑。计算机从最初单纯的数值运算,发展到文字处理,直至今天能够 处理声音、图形、图像的多媒体,都得益于“数字化”技术。而“数字化”不过是“形式化”在当代的表现形态。因此可以说,计算机科学的发展及其带来的现代文明都是离不开符号语言的建立和现代逻辑的发展。 《逻辑与语文学习》导学案参考答案 一、知识梳理 1(本质属性 2(根本属性 具体范围 3(运用概念进行判断的语言形式 4(由已知命题得出新的命题 复句 5(同一 偷换概念 改换话题 同时都是真的 否定另一个 同时都是假的 二、课堂训练 ((1)全同关系 (2)包含关系 1 (3)交叉关系 (4)反对关系 2(ABC(D是祈使句,E是疑问句,都没有明确地做出判断) 3.(1)自相矛盾。可改成“他是众多遇难者中唯一的幸存者。” (2)自相矛盾。可改为“行车速度过快(或过慢)是造成这次车祸的重 要原因。” (3)推理不当(将目的关系误作因果关系)。可改为“他为了捍卫大家的利益,勇敢地同歹徒搏斗。” 三、探究释疑 1(这个人违背了矛盾律的逻辑要求。 “任何东西都不能刺穿我的盾”蕴涵“我的矛不能刺穿我的盾”, 而“我的矛能刺穿任何东西”又蕴涵“我的矛能刺穿我的盾”,二者的推论形成矛盾关系,显然不能 同时成立。因此,他的吹嘘是自相矛盾,不能自圆其说。 2(邻居违背了排中律的逻辑要求。华盛顿的问话中包含了“此马有一只眼睛是瞎的”假定,偷马的邻 居胡乱瞎猜,正好中了华盛顿设下的圈套。其实,华盛顿的马双眼都不瞎。 四、学习反思(略) 布置作业 1(“我来这里是为了被绞死的。” 分析:因为如果真的绞死这个俘虏,那么这个俘虏说的是真话,而说了真话是要被烧死的。可是如 果把这人烧死,那他答的就是假话,而说假话的是要被绞死的。这个聪明的俘虏利用真话假 话,绞死烧死之间的逻辑推理关系,救了自己一命。 2(甲是偷钻石的罪犯。 分析:在回答询问中,乙与丁的口供相矛盾,其中必有一个真,一个假。如果这四人中只有一个人 讲真话,那么,这个唯一讲真话的,必定是乙和丁之中的一个。而甲、丙则属于讲假话之列。 即事实与甲和丙的口供相反。因此,可知甲是偷钻石的罪犯。 20 走进文学大师 1 《走近文学大师》导学案 【三维目标】 1、通过此次活动介绍阅读经典文学作品的方法,激发学生的阅读兴趣,增强学生的阅读欲望,推动课外阅读经典文学作品活动的进一步开展。 2、通过此次活动让学生减少对经典文学作品的隔膜,让学生体会经典文学作品的思想力量、感受文学大师的人格魅力。 3、培养学生用多种表达方式(口头作文或书面作文)来表达自己真实情感的能力。 【教学重点】 介绍阅读经典文学作品的方法,激发学生的阅读兴趣,增强学生的阅读欲望。 【教学难点】 培养学生用多种表达方式来表达自己真实情感的能力。 【学法】 合作探究法 激发兴趣法 自主阅读法 准备活动(一周) 学生准备 第一组学生每人选一位自己喜爱的文学大师,并用毛笔把他们的名字写在八开的白纸上。 第二组学生搜集报刊杂志上或自己了解的文学大师的感人轶事。 第三组学生寻找文学大师的经典作品。 第四组学生介绍自己阅读经典作品的方法。 【课时安排】 一课时 【知识链接】 感受大师的思想力量。法王路易十六被革命者囚禁在狱中时,读了伏尔泰和卢梭的著作,哀叹说:“这两个人亡了法国~”以伏尔泰、卢梭等为代表的启蒙主义文学家们,首先吹响了法国大革命的号角,不仅改变了法国的历史,也改变了世界的历史。巴金的《家》《春》《秋》,又促使当时多少青年冲破封建家庭的束缚,走向新生活。仅仅靠思想,仅仅靠手中的笔,这些大师们就推动了历史巨大的车轮,指引人类走向新的世界。 感受大师的优秀人格。许多文学大师,他们对人类的影响,不仅表现在他们精彩的文笔带给世人精神上的享受,更表现在他们以人格魅力,折服世人,体现出人类精神的巨大力量。屈原强烈的爱国主义精神体现在《离骚》诗篇中,也贯穿他生命的始终,并激励后世仁人志士。我们熟悉的大师如不屈服于专制政权,一生为自由而战斗的雨果,“横眉冷对千夫指,俯首甘为孺子牛”的鲁迅等,也是让人尊敬的人格典范。这些精神领域的伟人,代表了人类的良知,引导我们走向精神的自由和完美。大师的人格魅力,必然映照在他们的作品中。 感受大师丰富的内心世界。《老人与海》的主人公说:“人生来不是要给打败的,可以被消灭,但不会被打败。”海明威的人生就是努力实践这个信念,灵魂不屈不挠地时刻在斗争着。人生就是一场战争,最难的就是战胜自己。海明威最后以自杀结局,他是胜利者还是失败者,许多大师们也和海明威一样,内心充满矛盾和斗争,并体现在创作中。托尔斯泰在生命的最后时刻离家出走,是否感到理想和信念的破灭,卡夫卡 遗言毁掉自己的手稿,是否是对自己文学创作的否定,巴金晚年撰写《随想录》等,是否是对自己生活的某种追悔,以上所举,都可以说是大师们丰富的内心世界的体现。正是由于心灵的矛盾冲突,才使得大师的作品博大精深、汪洋恣肆,表现了人类社会复杂的生活和形态,表现了人类的苦闷、彷徨、探索和追求。 感受经典的艺术魅力。“原来是姹紫嫣红开遍,似这般都付与断井颓垣。良晨美景奈何天,赏心乐事谁家院。”一曲《牡丹亭》,数百年来传唱不衰,惹动多少有情人的心灵,成为我国古代戏剧创作的顶峰;“一天早晨,格里高尔?萨姆沙从不安的睡梦中醒来,发现自己躺在床上变成了一只巨大的甲虫。”卡夫卡一下子把我们抛入一个荒诞不经、令人震惊的世界,为我们开启了一道奇特的认识社会的大门。文学在内容上给我们提供了认识生活的窗口,同时还以它独特的、具有创造性的艺术形式,为我们展现无穷的艺术魅力,丰富我们的精神生活。在人类文明的进程中,文学艺术百花争春、群芳斗艳。不仅出现了许多优秀的大师,还产生了大量的文学流派。唐诗中有田园诗派、山水诗派、边塞诗派;现代诗歌创作中有新月派、象征派及后来的七月派、九叶派等。外国如欧美的象征主义诗歌、美国的“黑色幽默”、拉美的魔幻现实主义、法国的“新小说”、日本的新感觉派等,也是佳作泉涌,异彩纷呈。中外文学共同营建了人类精神财富的百花园。文学作品的艺术魅力,可以说是多方面的,题材、主旨、人物、情节、意象、语言风格、叙事模式等等,都能使读者得到艺术的熏陶、审美的享受。 【学习过程】 一、课前准备活动展示 1、请第一组同学把你们准备好的文学大师贴在黑板上。 给大家十分钟准备,把你们了解的这些文学大师给同学们介绍一下,并谈谈你自己的认识和感受。 2、请二组同学介绍你们了解的文学大师的感人轶事。 3、请第三组同学朗读文学大师们的作品(以短文为主)或作品片段。 4、请第四组同学介绍阅读经典作品的方法。 二、梳理积累 1(你读过哪些文学大师的名著,请梳理你的阅读记忆,根据提示填写。(A级) 姓名 国籍 作品名称 作品的思想精髓 狄更斯 巴尔扎克 雨果 莎士比亚 曹雪芹 罗贯中 巴金 2(默写名家名句,走进文学大师。(B级)(C级) (1)屈原《离骚》:“民生各有所乐兮,余独好修以为常。虽体解吾犹未兮, ,” 请你谈谈对“余独好修以为常”的理解,并说说这反映了作者怎样的人格, (2)李清照《醉花阴》:“ , 。 莫道不消魂,帘卷西风,人比黄花瘦。” 《夏日绝句》:“生当做人杰,死亦为鬼雄。至今思项羽, 。” 请你对照李清照的这两首诗歌,谈谈你对这位宋代著名女词人的评价。 (3)毛泽东《沁园春?长沙》:“ , ;书生意气,挥斥方遒。 指点江山,激扬文字, 。” 在这首词中“书生意气”的内涵是什么, “指点江山,激扬文字” 具体指的是什么,这首词反应了作者当时怎样的精神面貌 ,与他后来的成就有怎样的关系, 【学习反思】 “对于文学经典,通读全本和只读节选的感觉是不一样的,现在有许多所谓的简写本,以及许多根据经典改编的电视电影,对于了解经典也有帮助,但这些都代替不了自己诵读经典。”你是否同意这种说法,请结合自己的读书经历说明自己的观点。 【拓展训练】(D级) 觉慧走到琴的旁边,问琴道:“琴姐,你觉得有趣吗,” “我不觉得有什么趣味”,她淡淡地答道。 “你看了,有什么感想,”觉慧不肯放松地追问了一句。 “没有感想,”依旧是简短的答语。 “太平淡了,小时候看起来倒有趣味,现在却不然,”觉民在旁边接口说下去。 “你们当真一点也不感动吗,”觉慧严厉地问道。 觉民不明白他的意思,便掉过头看他一眼,不以为然地说:“这种低级趣味的把戏,怎么能使人感动,” “难道人就没有一点同情心吗,”觉慧愤愤地说。 “你说得太过火了。这跟同情心有什么关系,五舅他们得到了满足,玩龙灯的人得到了赏钱。各人得到了自己想要的东西。这还不好吗,”琴发表她的见解道。 “真不愧一位千金小姐,”觉慧冷笑地赞了一句,“像你这样聪明的人也看不出来。你以为一个人应该把自己的快乐建筑在别人的痛苦上吗,你以为只要出了钱就可以把别人的身体用花炮乱烧吗,这样开来你的眼睛还没有完全睁开嘞~” (摘自巴金《家》) (1)这段对话是针对一件什么事情展开的,请简要概述。 (2)对于这件事情琴姐、觉民、觉慧三人的态度分别是怎样的, (3)作者通过觉慧之口表现出怎样的思想, 《走近文学大师》导学案答案 【梳理积累答案】 1、狄更斯 英国 《大卫 科波菲尔》 大卫 艰难困苦,玉汝于成 巴尔扎克 法国 《高老头》 揭露资产阶级暴发户人与人之间赤裸裸的的金钱关系 雨果 法国 《巴黎圣母院》 教会的丑恶与贫民百姓的真善美互为反衬 莎士比亚 英国 《哈姆雷特》 资产阶级人文主义的道义最终战胜了邪恶 曹雪芹 中国 《红楼梦》 反叛封建礼教,崇尚个性解放,哀悼封建社会的末落 罗贯中 中国 《三国演义》 “天下者,天下人之天下,惟有德者居之” 巴金 中国 《家》 反叛封建家庭,追求民主、幸福的理想家园 2.(1)岂余心之可惩 “余独好修以为常”的意思是,我独爱美,并且习以为常。屈原在此强调了自己坚定的立场和鲜明的信仰,表白他一生自尊、自重、自爱,至死不改的人生态度。 (2)东篱把酒黄昏后 有暗香盈袖 不肯过江东 “东篱把酒黄昏后,有暗香盈袖”写女词人把酒与菊花相对,见菊瓣纤长,菊枝瘦细,而斗风傲霜,而人却悲秋伤别,无从消愁,此情此景让女词人顿生人不如菊之感。“人比黄花瘦”一句,含蕴丰富,情思无尽。“至今思项羽,不肯过江东”,是借古讽今,用项羽的故事发泄对南宋朝廷误国的愤恨。在国难当头,李清照一名纤纤女子不可能象辛弃疾那般“挑灯看剑”,“把吴钩看了”,但她深刻的思想痛苦远比浅薄的欢乐更有人生价值,更有思想意义。南宋统治集团在那"山外青山楼外楼,西湖歌舞几时休" 的醉生梦死之中花天酒地,而区区一柔弱女子,却能痛切国破家亡、始终为此而愁思郁郁,不肯去追欢取乐(《永遇乐?元宵》:"来相召,香车宝马,谢他酒朋诗侣"),这两首诗体现了女词人外柔内刚,忧国思民的宝贵人格。 (3)同学少年 风华正茂 粪土当年万户侯 “书生意气”指有革命理想的青年学生,意气风发,奔放豪迈;“指点江山,激扬文字”表现青年毛泽东和他的战友们撰文评论国家大事,激浊扬清。这首词反映了青年毛泽东以天下为己任的远大抱负和建设美好国家、造福人类的豪情壮志。这种远大的志向激励着老一辈无产阶级革命家艰苦卓绝地奋斗,打下无产阶级的江山,缔造了伟大的中华人民共和国。 【拓展训练参考答案】 (1)五舅等阔少花钱拿耍龙灯的人的身体来放花炮。 (2)琴姐:没有什么趣味。但五舅和耍龙人一个愿打一个愿挨,两下公平。觉民:太平淡了,是低级趣味的把戏,不能使人感动。觉慧:应该同情迫于生计的耍龙人,有钱人不该把自己的快乐建筑在穷人的痛苦上。 (3)富于同情心,正义感,呼唤人的尊严、平等。 22 莎士比亚戏剧 2 《莎士比亚戏剧》导学案(一) 【学习目标】 1、了解莎士比亚和他的作品。 2、了解莎士比亚作品的主要内容 3、了解莎士比亚戏剧特点。 4、赏析莎士比亚的戏剧语言。 【学习重点】 了解莎士比亚作品的主要内容 赏析莎士比亚的戏剧语言。 【学习难点】了解莎士比亚戏剧特点。 【学习方法】圈点勾画法 讨论法 探究法 【知识链接】 莎士比亚简介: 英国文艺复兴时期伟大的剧作家.诗人,欧洲文艺复兴时期人文主义文学的集大成者,他是“英国戏剧之父”,本?琼斯称他为“时代的灵魂”,马克思称他为“人类最伟大的天才之一”。被称为“人类文学奥林匹斯山上的宙斯”。虽然莎士比亚只用英文写作,但他却是世界著名作家。他的大部分作品都已被译成多种文字,其剧作也在许多国家上演。1616年5月3日病逝。如今,在他的故居,已竖起了近200多个国家的国旗,每一面都代表一个国家翻译了他的作品。他的名声也可想而知。就像中国人研究“红学”一样,对莎士比亚的研究也成了一门学问,叫做“莎学”。 莎士比亚作品: 莎士比亚一生创作了两部长诗、一部十四行诗集、37部剧作。他的创作可以分为三个时期。 1、早期(1590——1600): 长诗《维纳斯与阿多尼斯》 《鲁克丽丝受辱记》 《十四行诗集》 九部历史剧——《亨利六世》(上中下)、《理查三世》、《理查二世》、《亨利四世》(上下代表作)、《亨 利五世》、《约翰王》 喜剧代表作是《威尼斯商人》 悲剧代表作是《罗密欧与朱丽叶》 ,这是一部带有喜剧色彩的悲剧。 2、中期(1601——1607):七部悲剧、四部喜剧 “四大悲剧”——《哈姆莱特》 《奥赛罗》 《李尔王》《麦克白》 《雅典的泰门》也很著名。 3、后期(1608——1612):传奇剧 《辛白林》 《冬天的故事》《暴风雨》 【学习过程】 一、速读全文。要求勾画出重点字,注意字音字形字义。(A级) 1、给下列加点字注音。 辍学 ( ) 酝酿 ( ) 庇护 ( ) 铿锵有力 ( ) 悲怆 ( ) 挑唆 ( ) 忐忑 ( )大腹便便 ( ) 刚愎 ( ) 专横 ( ) 抵牾 ( )脍炙人口( ) 2、写出下列注音字的字形。 闻名xiá ( ) 迩 声名quâ ( ) 起 hïng ( ) 沟 cāng ( ) 桑 mâi( )力 迷 wǎng ( ) 跌 dàng ( ) 打hùn ( ) cuàn( )位 3、解释下列词意思。 闻名遐迩: 令人瞩目: 声名鹊起: 推陈出新: 浓墨重彩: 不拘一格: 出类拔萃: 大腹便便: 百家争鸣: 二、整体感知,思考并回答下列问题。 莎士比亚戏剧这篇文章都介绍了哪些内容, 三、具体研读 1、莎士比亚作品的创作背景 2、莎士比亚作品的主要内容是什么, 3、莎士比亚戏剧的特点是什么, 【学习反思】 几百年来,沧桑世变。但莎士比亚的剧作却一直吸引着一代又一代的人。这不仅因为作品深刻的人文内涵、 丰富的人物形象,还有他的语言特色。试分析他的剧作语言的特色。 【作业布置】 阅读莎士比亚剧作,写一篇1000字的读书随笔。 【素材积累】 莎士比亚名言: ? 善良的心地,就是黄金。 ? 惟有埋头,才能出头,急于出人头地,除了自寻苦恼之外,不会真正得到什么。 ? 你还能说„苦啊,最苦没有了?你的苦,还不曾苦到底呢。 ? 在命运的颠沛中,最容易看出一个人的气节。 ? 对众人一视同仁,对少数人推心置腹,对任何人不要亏负。 ? 衣服新的好,朋友旧的好。 ? 酒食上得来的朋友,等到酒尽樽空,转眼成为路人。 ? 我们宁愿重用一个活跃的侏儒,不要一个贪睡的巨人。 ? 时间是审查一切罪犯的最老练的法官。 ?报复不是勇敢,忍受才是勇敢。 ? 千万人的失败,都有是失败在做事不彻底;往往做到离成功尚差一步就终止不做了。 ? 书籍是全人类的营养品。 ? 时间会刺破青春表面的彩饰,会在美人的额上掘深沟浅槽;会吃掉稀世之珍~天生丽质,什么都逃不过他那横扫的镰刀。 ? 思想是生命的奴隶,生命是时间的弄人 ? 名誉是一件无聊的骗人的东西;得到它的人未必有什么功德,失去它的人也未必有什么过失。 ? 名字有什么关系,把玫瑰花叫做别的名称,它还是照样芳香。 ? 青春的特征乃是动不动就要背叛自己,即使身旁没有诱惑的力量。 ? 好花盛开,就该尽先摘,慎莫待美景难再,否则一瞬间,它就要凋零萎谢,落在尘埃。 ? 学问必须合乎自己的兴趣,方才可以得益。 ? 患难可以试验一个人的品格;非常的境遇方才可以显出非常的气节;风平浪静的海面,所有船只都可以 并驱竞胜;命运的铁拳击中要害的时候,只有大勇大智的人才能够处之泰然。 《莎士比亚戏剧》导学案(一)答案 一、 1、chuî yǜn niàng bì kēng qiāng chuàng suō tǎn tâ pián bì hèng dǐ wǔ kuài 2、遐 鹊 鸿 沧 魅 惘 宕 诨 篡 3、闻名遐迩:遐:远;迩:近。形容名声很大,远近都知道。 令人瞩目:瞩:注视。使人注视。 声名鹊起:形容名声迅速提高。 推陈出新:去掉旧事物的糟粕,取其精华,并使它向新的方向发展(多指继承文化遗产) 浓墨重彩:用浓重的墨汁和颜色来描绘。形容着力描写。 不拘一格:拘:限制;格:规格,方式。不局限于一种规格或一个格局。 出类拔萃:拔:超出;类:同类;萃:原为草丛生的样子,引伸为聚集。超出同类之上。多指人的品德才能。 大腹便便:便便:肥胖的样子。形容肥胖的样子。 百家争鸣:指各种学术流派的自由争论互相批评。也指不同意见的争论。百家,这种观点的人或各种学术派别。鸣,发表见解。 二、略 三、1、略 2、略 3、莎士比亚的戏剧大都取材于旧有剧本、小说、编年史或民间传说,但在改写中注入了自已的思想,给旧题材赋予新颖、丰富、深刻的内容。在艺术表现上,他继承古代希腊罗马、中世纪英国和文艺复兴时期欧洲戏剧的三大传统并加以发展,从内容到形式进行了创造性革新。他的戏剧不受三一律束缚,突破悲剧、喜剧界限,努力反映生活的本来面目,深入探索人物内心奥秘,从而能够塑造出众多性格复杂多样、形象真实生动的人物典型,描绘了广阔的 、五光十色的社会生活图景,并以其博大、深刻、富于诗意和哲理著称。 莎士比亚的戏剧是为当时英国的舞台和观众写作的大众化的戏剧。因而,它的悲喜交融、雅俗共赏以及时空自由 、极力调动观众想象来弥补舞台的简陋等特点。 【学习反思】 莎士比亚的戏剧语言变化多端,娴熟灵巧的遣词择句勾勒出给人无尽遐思的丰富意象、扣人心弦的剧情发展和各具特色的鲜明的人物特征。莎士比亚根据人物身份与处境的不同而选用不同语体的语言:文雅或粗俗,哲理或抒情,从而使人物的语言形象化,使剧中三教九流的人物形象栩栩如生地展现在舞台上。在莎士比亚所有的这些形象化的语言中不可或缺地运用了各种各样的修辞手法,丰富的人物独白、双关语、比喻等的手法。 独白,是人物内心所思所想的一种语言的自然流露,它既可以起到交待剧情的作用,同时也有助于人们更好地理解人物的形象。 在《哈姆雷特》的第一幕、第二幕、第三幕、第四幕中,都有哈姆雷特的大段独白,通过这些独白,我们可以进入这位丹麦王子的灵魂深处,理解什么使他忧郁、痛苦、矛盾。 一方面极力否定自己,甚至要化成一堆微不足道的露水。另一方面,在人文主义的影响下,他又极力地赞美人类,认为人类是“宇宙的精华,万物的灵长”。可见,哈姆雷特的性格是复杂的,同时又极具丰富性。 从哈姆雷特的几段细致入微的心理活动中,我们可以更清晰地看出哈姆雷特性格发展的曲折历程。哈姆雷特形象的魅力是无穷的,“一千个读者心中就有一千个哈姆雷特”。我们在欣赏哈姆雷特这一形象时,独白对我们理解这一人物的性格有着不可忽视的作用,作品运用独白,不仅展示出主人公的内心活动,而且也揭示出主人公内心的矛盾冲突。独白独白对主人公哈姆雷特内心矛盾的深刻揭示,是其它艺术手段难以代替的。 二、 双关语的妙用 双关也是莎士比亚的戏剧中运用得比较多的一种语言艺术手段。在莎士比亚剧中,许多台词表面上看起来很简单,易于理解,其实其中暗示的意义却是非常深刻的,这成为莎剧一个显著特征。这显然是莎士比亚用来突出哲理性思想或实质,借以发人深省的艺术手段。 三、 形象的比喻 莎剧人物常常是用作者的语言形象说话的,因此从人物语言中频频出现的比喻里,会找到很多表现人物心理特征的材料。 如第三幕第一场中,奥菲利娅这样形容哈姆雷特:“他的高贵无上的理智,象一串美妙的银铃失去了谐和的音调,无比的青春美貌,在疯狂中凋谢~”奥菲利娅对哈姆雷特发疯一事的痛心,就好比以为音乐家听到不和谐的音乐般痛苦。 《影视文化》导学案 【学习目标】 1、了解影视百年发展历程。 2、从历史的角度观照影视艺术,了解影响影视艺术发展的诸多因素。 3、考察电视和电影的相互影响。 、引导学生比较文学和电影两种艺术的不同特色,了解它们各自的艺术手段。 4 5、提高自己的艺术鉴赏能力。 6、探讨中外影视的异同,认知民族特色。 【学习重点】 1、从历史的角度观照影视艺术,了解影响影视艺术发展的诸多因素。 2、考察电视和电影的相互影响。 3、引导学生比较文学和电影两种艺术的不同特色,了解它们各自的艺术手段。 4、影视作品和文学原著的比较。 【学习难点】 1、提高自己的艺术鉴赏能力。 2、探讨中外影视的异同,认知民族特色。 3、影片中蒙太奇手法的运用。 【学方指导】 1、欣赏鉴赏法 2、讨论法 3、比较鉴赏法 【活动安排】 1、学生8人一小组,利用网络搜集有关中国影视发展概况的资料。 2、安排学生欣赏几部不同时代的具有代表性的影视作品。 【知识链接】 1、七大艺术包括:文学、舞蹈、音乐、绘画、雕塑、戏剧、电影。电影把所有的艺术都加以综合,形成运动中的造型艺术。作为第七艺术的电影是把“静”的艺术和“动”的艺术,“时间艺术”和“空间艺术”,“造型艺术”和“节奏艺术”全部包括在内的一种综合艺术。 2、蒙太奇手法 (1)文学、音乐或美术的组合体的音译。 (2)原为建筑学术语,意为构成、装配。 (3)电影将一系列在不同地点,从不同距离和角度,从不同方法拍摄的镜头排列起来,叙述情节,刻画人物。但当不同的镜头组接在一起时,往往有产生各个镜头单独存在时所不具备的含义。 一言以蔽之:蒙太奇就是把分切的镜头组接起来的手段 通过蒙太奇手段,电影的叙述在时间空间的运用上取得极大的自由。可从空间上从巴黎跳到纽约,或者在时间上跨过几十年,操纵时空的能力,使电影艺术家能根据他对生活的分析,截取他认为最能阐明生活实质的,最能说明人物性格、人物关系的,乃至最能抒述艺术家自己感受的部分,结合在一起,经过分析与组合,保留下最重要的,最有启迪力的部分,摒弃省略大量无关轻重的琐屑,去芜存菁地提炼生活,获得最生动的叙述、最丰富的感染力。总之,“蒙太奇是电影艺术的基础”,“没有蒙太奇,就没有了电影的功能”。 3、关于大片 (1)1994年中国引进的第一部大片是《亡命天涯》 (2)以好莱坞高成本电影为主,如《真实的谎言》《泰坦尼克号》《指环王》 (3)中国市场的大片,如《英雄》《无极》《十面埋伏》《满城尽带黄金甲》《夜宴》 (4)贺岁片 A、香港占用春节的档期B、美国在圣诞节前后上映C、中国元旦前后 中国的贺岁片以冯小刚执导的较为有名,如《甲方乙方》《有话好好说》《不见不散》《大腕》《没完没了》《手机》《天下无贼》 【学习过程】 一、知识梳理(A级) 简答下列与影视文化有关的问题。 1.“1895年,被称为第七艺术的电影诞生了”。电影之所以被称作“第七艺术”,是针对其余哪六种艺术形式而言,它较之其余艺术形式有什么突出的特点, 答: 2.科学家在“视觉存留”原理和“心理认同”原理的基础上,发明了“连续摄影术”“胶片”以及“活动放映术”,这才有了电影的诞生,你对此有什么看法, 答 : 3.“1903年,电视悄然问世。人们习惯上把电影和电视合称为‘影视艺术’”。电影和电视这两种艺术有太多的相似、想通和相互借鉴之处,你能说出它们分类的区别吗, 答: 二、实践探究 阅读探究 一部影片的创作是从编写文学剧本开始的,电影文学剧本是用文字表述和描绘未来影片内容的一种文学体裁。根据文学名著改编的电影剧本是对原著的再创造,体现了改编者的匠心。下面两段文字,甲文节选自鲁迅的《祝福》,乙文节选自夏衍根据该文改编的同名电影剧本, 请比较阅读下面两段文字,思考并回答后面的问题。 (甲) 她整日紧闭了嘴唇,头上带着大家以为耻辱的记号的那伤疤,默默地跑街,扫地,洗菜,淘米。快够一年,她才从四婶手里支取了历年积存的工钱,换算了十二元鹰洋?,请假到镇的西头去。但不到一顿饭时候,她便回来,神气很舒畅,眼光也分外有神,高兴似的对四婶说,自己已经在土地庙捐了门槛了。 冬至的祭祖时节,她做得更出力,看四婶装好祭品,和阿牛将桌子抬到堂屋中央,她便坦然地去拿酒杯和筷子。 “你放着罢,祥林嫂!”四婶慌忙大声说。 她像是受了炮烙?似的缩手,脸色同时变作灰黑,也不再去取烛台,只是失神的站着。 直到四叔上香的时候,教她走开,她才走开。这一回她的变化非常大,第二天,不但眼睛窈陷?下去,连精神也更不济了。而且很胆怯,不独怕暗夜,怕黑影,即使看见人,虽是自己 的主人,也总惴惴的,有如 头发也花白起来了,记性尤其坏,甚而在白天出穴游行的小鼠;否则呆坐着,直是一个木偶人。不半年, 至于常常忘却了去淘米。 “祥林嫂怎么这样了?倒不如那时不留她。”四婶有时当面就这样说,似乎是警告她。 然而她总如此,全不见有怜俐?起来的希望。他们于是想打发她走了,教她回到卫老婆 子那里去。但当我还在鲁镇的时候,不过单是这样说;看现在的情状,可见后来终于实行 了。然而她是从四叔家出去就成了乞丐的呢,还是先到卫老婆子家然后再成乞丐的呢?那我 可不知道。 (乙) (低沉的旁白):“这样,她听从柳嫂的劝告整日紧闭了嘴唇,默默地跑街,扫地,洗 菜,烧火,好容易,她积下了十吊工钱„„” 土地庙,祥林嫂把毛巾包得好好的十块鹰洋拿出来,交给庙祝,千恩万谢。庙祝将银洋一块一块地敲过,点过,对她点点头,讲了几句话,祥林嫂的表情起了变化,安心了,解除了负担似的,跑回去。 I 鲁家废园,下着雪,祥林嫂搬了一大捆柴进来。 客厅(大年夜,又是祝福上供的时候了。 柳嫂端了大盘装着一个煮熟了的猪头,猪头上面横七竖八地插上八九只筷子,上来。阿香跟在后面,端了另一只大盘,盘里装着一只鸡,放好。柳嫂正要回身再去端供品,只见祥林嫂兴冲冲地端了一只盘子,里面是一条大鱼,上来。走到供桌前面。 四太太大惊,大声怪叫:“啊哟,祥林嫂,谁叫你端的?放着放着„„”拦住她,不让她走到供桌前面。 祥林嫂完全出乎意料,不知说什么才好,几秒钟之后,才磕磕巴巴地:“我,捐过门槛„„捐过„„” 四太太一扫她平时的慈眉善脸,迎头痛击:“什么门槛不门槛!放下,不准开口。” 四老爷怒容满面:“我早说,不能用,不能用这种人„„” 四太太对柳嫂:“快去换过,换一条草鱼,快。”对阿香,“站着干什么?叫她(指着祥林嫂)出去。” 祥林嫂茫然地出去。失神落魄的表情,嘴角微微地抽搐。 土地庙。长明灯的微光下,祥林嫂发疯似的用刀砍一条门槛。 被惊动了的庙祝,掌着油灯跑出来,大惊,失声叫喊:“哪个?” 像受了惊的老鼠,祥林嫂回身向外就走。庙祝追到门口。 (节选自《夏衍电影制作集》) 1.画面和声音是电影语言最基本的艺术元素。甲文第1自然段所描述的内容在乙文中是如何表现的,为什么要这样表现, 答: 2.乙文的章节是根据什么划分的,为什么要这样划分, 答: 3. 甲画线文字的内容在乙文中有怎样的变化,这样会产生怎样的艺术效果, 答: 4.乙文最后一个章节在甲文中没有,是剧作者在原著基础上创造性的发挥,你认为祥林嫂怒砍门槛的情节是否符合人物性格,为什么, 提示:该题为开放性答案,只要合情合理即可。可从鲁迅“哀其不幸,怒其不争”的思想来谈,也可从夏衍教育、鼓励后人的意图来谈;可联系祥林嫂的温驯善良来谈,也可以联系祥林嫂被逼再婚出格的反抗行为来谈,等等。 答: 三、合作探究 1、影视作品与文学原著进行比较,分析一下它们各自的艺术特色,并举例说明。 答: 2、电视剧中的“清宫戏”已经流行很长一段时间,请谈谈你对“清宫戏热播”这一现象的看法。 答: 【小结】 影视艺术的产生不过百年的时间,但它早已渗入到我们生活的各个角落,深刻地影响着我们的生活。与社会上的风光无限相比,它们在基础教育中却遭到了冷落,本节课的学习就是从综合的角度引导学生去了解影视艺术及其有关的东西,引导学生阅读相关的影视艺术著作,思考观察过的影视剧,梳理自己的影视知识。通过对影视文化的了解,给学生一个正面的引导,从而很好地去欣赏文学,欣赏影视,达到对文学、影视的双层解读。 【学习反思】 【作业布置】 哪部电视剧给你留下的印象最深,说说它的故事情节,并对主题、风格、演员表演、摄影、背景音乐、美工、制作等方面情况做一番评论。 【拓展阅读】 “剧本医生”能为影视剧“治病救本”, 《剧本诊断报告》出炉并将投入使用 随着关渤率队做出的《剧本诊断报告》的出炉,“剧本医生”这个概念日前在国内首次被明确。据介绍,进一步完善的《剧本诊断报告》将投入使用。 作为舶来品的“剧本医生”概念,会否在国内“水土不服”,成为许多业内人士关注的焦点。 “医生”本是舶来品 关渤介绍,涉及人物、创意、情节点、台词等百余个项目的《剧本诊断报告》,其功用简单而言就是对电影剧本、电视剧剧本进行“诊断”。关渤说:“这份报告就像我们日常的体检报告,只不过它针对的是剧本,而不是人。自然,做出这份报告的就是剧本医生。” 事实上,“剧本医生”这一概念早在上世纪就出现在好莱坞。今天好莱坞的大部分电影的剧本定稿背后都有大量“剧本医生”参与工作。经典电影《教父》中的经典台词更出自“剧本医生”罗伯特?唐恩之手。而包括《辛德勒的名单》的编剧史蒂文?泽里安、《少数派报告》编剧斯科特?弗兰克等在内的几乎所有好莱坞顶尖编剧都有过当“剧本医生”的经历。 “规范”难免“一刀切” 关渤表示,目前国内一些剧本已经通过剧本医生的“诊治”,收效良好。而多位国内编剧也肯定了剧本医生这一概念。 电视剧《铁齿铜牙纪晓岚3》《万卷楼》的编剧汪海林表示,《剧本诊断报告》对编剧行业有积极意义,报告明确了剧本版权这一概念,有利于维护编剧的合法权益。电影《美人草》的编剧黄兢晶认为,这起码是一个信号,表明编剧地位正在提高。“剧本的质量有据可循,不再是制片方单方面主观决定”。 然而,一个硬币有两面,“剧本医生”在为剧本完善作出积极方面的同时,也存在一些质疑。业内人士指出,有时剧本医生的程序化修改也会让剧本流于“中庸无过”,从而减弱原创作者的个性意图。对此,关渤表示,用一个客观的“剧本医生”来对主观的剧本创作进行百分百的限定是不科学的,但主观艺术创作有一些客观规律和原则可循,这就是“剧本医生”的作用。 “剧本医生”也需理性 对于“剧本医生”的新概念,业内也更加理性对待。《亲兄热弟》《半路夫妻》的编剧彭三源认为,“剧本医生”这一概念在国内首次正式被提出,存在积极意义,但其实剧本策划、剧本编辑等职位的工作人员,一直就是在做这件事情。《新剧本》副主编、资深编剧林蔚然亦表示:“中国的„剧本医生?目前多半跟策划或编剧的概念重叠。„剧本医生?的概念推行,关键还是要制片方认可。” 《剧本诊断报告》的出炉能否真的被推行并起到“治病救本”的规模效应,尚需实践的检验。但其提出的国内“剧本医生”的概念,也让很多人重新关注起影视剧本创作链中的一些固存问题。 关渤表示:“很多制作单位对待剧本还停留在作坊水平。我们推出这个概念,就是要强调剧本属于影视工业的一部分,期待行业以更加理性的态度对待剧本。同时,„剧本医生?的推出对于由第三方更合理客观地判定一个剧本的好坏,有更积极的专业意义。” 《影视文化》导学案参考答案 一、知识梳理 1.(1)文学、舞蹈、音乐、绘画、雕塑、戏剧 (2)电影把所有的艺术都加以综合,形成运动中的造型艺术。作为第七艺术的电影是把“静”的艺术和“动”的艺术,“时间艺术”和“空间艺术”,“造型艺术”和“节奏艺术”全部包括在内的一种综合艺术。 2.开放性答案,言之成理即可。可从电影的特点,科学与艺术的联姻,科学对社会文化的推动,物质文明与精神文明的关系等方面作答。 3.最主要的区别是传播手段不一样,电影用胶片拍摄,用放映机传播,电视用数字摄像机、DV机拍摄,用电视机传播;其次是审美取向不完全相同,一部电影放映时间一般在2,3个小时,表达的内容须对生活高度提炼,而一部电视剧可长达上百集,反映的内容可以更接近生活原生态。、 二、实践探究 1、 用旁白(画外音)和人物的行为动作来表现,为了达到电影的视听效果,满足观众的需求。 2、根据场景的转换来划分的,为了拍摄的需要。 3、增添了许多细节,使剧情更加丰富、情节更加逼真,表演更加形象,大大增加了观赏效果。 4、该题为开放性答案,只要合情合理即可。可从鲁迅“哀其不幸,怒其不争”的思想来谈,也可从夏衍教育、鼓励后人的意图来谈;可联系祥林嫂的温驯善良来谈,也可以联系祥林嫂被逼再婚出格的反抗行为来谈,等等。 三、合作探究 1、(1)名著的作者侧重于语言的修炼、意境的展开;而影视剧作者更加注重视觉上的效果,如场景、特技之美的安排 (2)文学作品主要是利用动作的描写、外貌描写、心理描写、语言描写来刻画人物性格,表现故事情节,体现作品主题;而电影(电视)除去这些刻画、表现手段之外,还要使用视频所特有的蒙太奇、特写、化装等手段来刻画人物,推进情节的发展,进而体现作品的主题。 (3)其实就是诗与画的区别,同样是艺术,但是带给人的感受不同,原著是“诗”,而影视剧是“画”。 (4)举例 A、《三国演义》书上的刘备就没有电视剧上演的那么爱哭,并且很多情节能写但演不出来 B、官渡之战曹操军队肯玉米,玉米是以后传入中国的 2、例:A、河 本名凌解放,创作清代“帝王系列”历史小说《雍正皇帝》《康熙大帝》《乾隆皇帝》《光绪皇帝》〈少年天子〉等一系列的正史小说 B、戏说的电视剧,琼瑶小说《还珠格格1、2、3》,《戏说乾隆》《宰相刘罗锅》《铁齿铜牙纪晓岚》 观点:八旗子弟的服饰和外在气质上,马褂、马甲、旗袍、马蹄袖,精干利落的打扮,APEC会议期间,唐装是清人马褂演变而来的,<<花样年华>>中的张曼玉穿起旗袍来美得一塌糊涂,而”旗袍”实则是由清代男装嫁接到女人身上的。 《莎士比亚戏剧》导学案(二) 【学习目标】 1、把握莎士比亚戏剧中一些主要人物形象。 2、了解女扮男装的情节作用。 【学习重点】把握莎士比亚戏剧中一些主要人物形象。 【学习难点】女性人物形象的深刻内涵。 【学习方法】精读和略读结合 读议结合 读写结合 【知识链接】 1、历史剧主要剧作内容梗概: 《亨利四世》创造了一个不朽的艺术典型--喜剧人物福斯塔夫。福斯塔夫是一个年过五十的破落骑士,一个好吹牛的懦夫,一个贪婪的冒险家。他是流氓头子,善于见风使舵、混水摸鱼。他否认任何道德,既无良心谴责也无怜悯之心,平生第一快事就是以粗鲁低级的方式向女子献殷勤,生活理想只是如何更大了限度地满足口腹之欲。他生性幽默,喜剧情节和笑料通过他的所言所行妙趣横生。他自恃是太子亲信,整 天醉眼朦胧,坐在野猪头酒馆里,率领着一群年轻的地痞无赖,说谎、欺诈、抢劫。一但被太子拉上战场却又胆小如鼠,毫不顾及封建骑士的勇敢精神与荣誉观念,只会倒在地上装死,驮来敌人的尸体冒功。作品最后通过福斯塔夫被掌权的太子——“贤明”君王亨利五世驱逐,反映了封建势力及其附庸必将灭亡的命运。而福斯塔夫则作为一个活在老百姓身边为老百姓喜闻乐见的人物,一个封建关系解体时期“无衣无食的雇用兵”和“冒险家”的典型,成为莎士比亚笔下最成功的喜剧形象之一。 2、喜剧主要剧作内容梗概: 《第十二夜》描写孪生兄妹西巴斯辛和薇奥拉在海上航行遇险,先后流落到伊利里亚。这里的统治者奥西诺公爵正在向一位名叫奥丽维亚的贵族小姐求婚,却遭到拒绝。薇奥拉女扮男装,化名西萨里奥做了公爵的少年侍人。从此,爱上奥西诺公爵的薇奥拉作为爱情的使者,奔跑于奥西诺和奥丽维亚之间,凭着她与自己孪生哥哥相貌异常相像,顺利解决了一系列复杂而又严重的爱情冲突。钟情少年侍从薇奥拉的奥丽维亚成了薇奥拉哥哥的妻子;而饱受对奥丽维亚单相思之苦的奥西诺公爵在发现他的侍从原来是一个美丽的女孩之后,也认识到她感情的可贵,就和薇奥拉结了婚。剧中错综复杂的情节充满了巧合、悬念、误会以及移花接木、巧装改扮的喜剧因素,而处于全剧中心的是勇敢、热情、纯真的少女薇奥拉,在争取人格独立、幸福的权力及人与人之间平等关系的斗争中,她显示出忘我无私、坚贞不渝的品格,充分体现了文艺复兴时期人文主义者的理想。全剧充满了对自由的渴望、对生活的热爱,反映了文艺复兴时代人们朝气蓬勃、勇于进取的精神风貌。 《威尼斯商人》则以较强的对社会讽刺、批判的态度见称。威尼斯富商安东尼奥为了成全好友巴萨尼奥的婚事,向犹太人高利贷者夏洛克借债。由于安东尼奥货款给人从不要利息,并帮夏洛克的女儿私奔,怀恨在心的夏洛克乘机报复,佯装也不要利息,但若逾期不还要从安东尼奥身上割下一磅肉。不巧传来安东尼奥的商船失事的消息,资金周转不灵,贷款无力偿还。夏洛克去法庭控告,根据法律条文要安东尼奥履行诺言。为救安东尼奥的性命,巴萨尼奥的未婚妻鲍西娅假扮律师出庭,她答允夏洛克的要求,但要求所割的一磅肉必须正好是一磅肉,不能多也不能少,更不准流血。夏洛克因无法执行而败诉,害人不成反而失去了财产。 3、悲剧主要剧作内容梗概: 《麦克白》内容简介: 苏格兰国王邓肯的表弟麦克白将军,为国王平叛和抵御外族入侵立功归来,路遇三个女巫。女巫对他说了一些预言和隐语,说他将进爵为王,但他并无子嗣能继承王位,反而是同僚班柯将军的后代要做王。麦克白是有野心的英雄,他在夫人的怂恿下谋杀邓肯,做了国王。为掩人耳目和防止他人夺位,他一步步害死了邓肯的侍卫,害死了班柯,害死了贵族麦克德夫的妻子和小孩。恐惧和猜疑使麦克白心里越来越有鬼, 也越来越冷酷。麦克白夫人神经失常而自杀,对他也是一大刺激。在众叛亲离的情况下,麦克白面对邓肯之子和他请来的英格兰援军的围攻,落得袅首的下场。 《麦克白》(1605)是莎士比亚戏剧中心理描写的佳作。全剧弥漫着一种阴鸷可怕的气氛。莎士比亚通过对曾经屡建奇勋的英雄麦克白变成一个残忍暴君的过程的描述,批判了野心对良知的侵蚀作用。由于女巫的蛊惑和夫人的影响,不乏善良本性的麦克白想干一番大事业的雄心蜕变成野心,而野心实现又导致了一连串新的犯罪,结果是倒行逆施,必然死亡。在迷信、罪恶、恐怖的氛围里,作者不时让他笔下的罪人深思、反省、剖析内心,麦克白夫妇弑君前后的心理变化显得层次分明,这就更加增大了悲剧的深度。 《罗密欧与朱丽叶》讲述的是在一个名叫维洛那的意大利小城里,有两个门户相当的大家族——蒙太古家和凯普莱特家。世代的恩怨导致了两家人之间纷争不断,他们的族人一碰面就会出现械斗场面。 在这样充满深仇大恨的两个家族之间,却产生了一段感人至深的恋情。 蒙太古家的儿子罗密欧与凯姆莱特家的女儿朱丽叶在一次宴会中一见钟情。他们很为这段产生于仇恨压制下的感情担心。罗密欧请求劳伦斯神父为他们举行秘密婚礼,好心的神父很乐意成全他们这段美好的恋情,并希望他们的结合可以使两家尽释前嫌。可就在这时,一件非常不幸的事情发生了:罗密欧的好朋友茂丘西奥因为帮罗密欧出头被朱丽叶的表哥提伯尔特一剑刺死。迫于正义的呼唤,也为了替朋友报仇,罗密欧不得不拔剑杀死了提伯尔特。朱丽叶焦急万分,请求劳伦斯长老帮助,长老急中生智,一面让朱丽叶在婚前的夜晚服下他的假药,一面派人送信给罗密欧,让他适时赶来带走朱丽叶。朱丽叶依计行事,可是送信的人却没有如期找到罗密欧。罗密欧听到朱丽叶的死讯,悲痛欲绝,匆匆回到维洛那城,来到朱丽叶的墓地,以死殉情。当朱丽叶从假死中醒过来后,满怀着同罗密欧相聚的希望,但是当她发现罗密欧已经死去后,不顾外面喧嚣的人声,也不顾长老的劝告,立即决定同罗密欧死在一起。她听到守夜人的声音后,果敢地拿起匕首,刺进了自己的心脏。罗密欧与朱丽叶的爱情虽然毁灭了,却因此赢得了两家人的悔悟,换来了两家人的世代和平。 【学习过程】 一、阅读《亨利四世》,简要分析福斯塔夫的人物形象。 二、阅读《第十二夜》,简要分析薇奥拉的人物形象。 三、阅读《威尼斯商人》,简要分析夏洛克的人物形象。 四、阅读《威尼斯商人》,简要分析安东尼奥的人物形象。 五、阅读《麦克白》,简要分析麦克白的人物形象。 六、阅读《罗密欧与朱丽叶》, 该剧主要刻画了怎样的人物形象? 【学习反思】 莎士比亚在剧作中为什么喜欢用女扮男装的情节,,他对女性的描写具有什么样的深刻内涵, 【作业布置】 举出莎士比亚剧作中给你留下深刻印象的几位女性。你怎样评价她们,作者对女性的刻画有哪些精彩之处, 《莎士比亚戏剧》导学案(二)答案 一、福斯塔夫是封建关系崩溃时期无衣无食的雇佣兵和冒险家的典型。他是个道德败坏的酒色之徒,专靠偷盗和招摇撞骗为生,却又能随口胡诌出各种道理来为自己的丑恶行为辩解,以锋利的言辞对世事加以讽刺和讥诮。他出生封建阶级,却蔑视封建的荣誉观念;身上沾满了没落阶级的坏习气,却又时而流露出平民社会底层人物的乐观精神和粗犷风趣。福斯塔夫是历史过渡时期中过渡性小人物的真实写照,莎士比亚通过这类典型人物的行动和冲突,展示出福斯塔夫式的背景,反映了封建关系崩溃时期五光十色的平民社会。 二、剧中主要人物薇奥拉温柔善良,洁身自好,但是性格内向、软弱,能忍受痛苦而毫无怨言。当她被派去代自己所爱的人向伯爵小姐求婚,却能忍受痛苦,极尽动听的言辞去打动小姐的心,充分表现了她崇高的自我牺牲精神。甚至当公爵因误会而产生杀她的念头时,也毫无怨言。她具有人文主义理想,敢于冲破封建禁欲主义和等级观念的束缚,执着地追求爱情幸福。有胆有识,才智过人且生性温柔,是一位既执着追求爱情幸福又极富自我牺牲精神的青年女性形象,她身上所体现的人文主义理想的美和力量使得她成为莎翁笔下“最可爱的”女性形象之一。 三、夏洛克是一个复杂的人物形象。一方面,他贪婪吝啬、冷酷无情、奸诈狠毒。但同时,夏洛克也是一个悲剧性的人物。他最后彻底失败,一无所有,固然是咎由自取,但其中也夹杂着宗教压迫和种族矛盾的因素。作为一个饱受歧视和迫害的犹太人,夏洛克有值得同情的一面,夏洛克有时也会流露出一些极为大胆的思想,对上层社会和等级制度进行抨击。这就使这一形象带有了一定的复杂性。 四、安东尼奥是理想化的商业资产者形象。他从事海外贸易,船队遍布全球。他慷慨乐助,宽宏大度,重视友谊,济人于危。莎士比亚站在新兴资产阶级立场,肯定当时对生产发展、市场繁荣有促进作用的商业资本,憎恶对生产发展起阻挠作用的高利贷资本。在道德上,安东尼奥体现了讲求仁爱和友谊的人文主义理想。 五、剧中的麦克白是一个谋权篡位、凶残的人,为了登上王位,他谋杀了国王,即它的表哥邓肯;登上王位后,为了巩固王位,他残暴屠杀人民,使全国血六成河,置社会于混乱,陷人民于水火。可是他也有人性化的一面。他一面作恶,一面恐惧、悔恨。并且有痛苦的挣扎。其恐惧、挣扎和悔恨皆发自内心深处,可以看出他还有善的一面。 总之,他的堕落是人性弱点促成他的悲剧;他的毁灭死亡给人们以深思. 莎士比亚塑造的这一悲剧性人物是对刻画人物、探索人类灵魂黑暗面而获得的一大成功 六、 提示:《罗密欧与朱丽叶》描写一对青年男女因家族间的世仇而不能联姻,结果自杀的故事,揭露了封建制度的残酷无情,同时歌颂了青年男女纯洁坚贞的爱情,表达了作者早期的人文主义思想。 剧中的女主人公朱丽叶是一位名门闺秀。她热爱生活,天真纯洁,在同罗密欧相爱之前是一个循规蹈矩、惟命是从的贤淑少女,她爱上罗密欧后,爱情灼热的火焰在她身上立刻变成一股不可遏止的力量。她变得大胆起来了,再也不是俯首帖耳,唯父母之命是听的闺秀了。月夜中,她自言自语,道出了对罗密欧的爱慕与思念,当心上人突然出现在眼前的时候,她抛弃了少女的矜持,勇敢地冲破封建束缚,彻底抛弃了封建家族观念,大胆地表白了自己的爱情,显示了她是一个坚定果断的体现了时代精神的资产阶级新女性。 罗密欧也是一个具有人文主义生活理想的新时代的青年,为了追求个人幸福生活。他敢于挣脱封建伦理观念的束缚。他出生于名门贵族,热爱生活,有理想,为了纯洁的爱情。他敢于反抗封建制度,视死如归,虽赴汤蹈火在所不惜,显示了人文主义者为追求幸福生活,实现生活理想的巨大的精神力量。 学习反思: 在多部喜剧中,莎士比亚将其主角女扮男装。“女扮男装”现象决非偶然。推究而论,在于作者思想立意方面的需要。从思想立意的视角来看,“女扮男装”不啻古代奴隶、封建社会的女性在特殊环境之下的一种特殊的择偶方式,其中充溢着女性对自由的大胆追求和男女平等生活的热切憧憬。莎士比亚有感于此,所以他对此题材的关注与开掘,在一定程度上表现出对女性追求自由行为的首肯、褒奖乃至张扬。 还因为这种题材本身富于强烈的戏剧性,首先,从戏剧人物塑造的角度来看,女扮男装者具有双重性格:人物本身是个女性,但又要装成男性。所以,在她的身上,以男性特征为主,但总会不免时时不自觉地流露出女性所特有的某些特征来。因此,女扮男装者性格较为复杂,感情相当丰富,集刚柔、阴阳于一身,不单调、不乏味。 其次,“女扮男装”的题材之所以富有戏剧性,还在于乔装本身便富于戏剧性。女性人物既然是硬性装扮出来的,其所装扮的那个“男人”,自然可能有时装得很像,有时又装得不太像,甚至有时还会露出破绽而被人识破。总之,女扮男装不会那么一帆风顺、天衣无缝。由此就决定了该类题材的作品情节曲折复杂,矛盾冲突迭起纷纭。 总之,透过表面现象分析剧他频繁使用伪装的原因,我们发现伪装对剧作家来说至关重要,因为伪装是戏剧情节,塑造人物个性,吸引观众的重要手段。 这些女性身上表现出的所有特征,都是莎士比亚人文主义思想的集中表现,也是他所倡导的道德原则和生活理想的形象化表现。莎士比亚把人文主义对人性的基本认识运用到他的女性形象身上,他把笔下的女性写成具有纯真节操的人,从而肯定了她们的人格不容侵犯,肯定她们有独自主享受自主并支配生活的权利和自由。他给女性从外表到内心赋予了那么多美好的禀赋,这是他用浪漫的方式从正面讴歌了人文主义理想中的人物。她们忠贞的品德、她们聪敏的思想、优雅的外表,都可以成为人文主义者理想中的人物的楷模。莎士比亚的那个时代的先进思想寄予到妇女形象身上,这充分反映了他世界观中的进步方面,他敢于冲破整个封建社会遗留下来的对妇女的歧视和偏见,同情她们,赞美她们。这些女性身上,我们可以看出莎士比亚提倡的是个性、自由、平等。他反对禁欲主义,肯定人世俗的幸福;反对封建包办婚姻,肯定爱情的自由选择。 23 谈美 1 《谈美》导学案 学习目标: 1(了解作者朱光潜的生平经历和主要作品。 2(了解《谈美》的主要内容和相关美学理论。。 学习重点: 理解《谈美》的主要内容和相关美学理论。 学习难点: 如何理解和运用《谈美》中的相关美学理论。 学法指导: 点拨法、讨论法、探究法。 知识链接: 知识链接: 1(作者简介: 朱光潜(1897-1986),安徽桐城人。我国著名美学家,文艺理论家、翻译家。笔名孟实、盟石。1923年毕业于香港大学教育系。1925年起先后赴英、法等国学习,1930年荣获法国斯特拉斯堡大学文学博士学位。1933年回国,历任四川大学、武汉大学、北京大学教授。1962年,他转入北京大学哲学系,讲授美学,任博士生导师。曾经担任中华全国美学学会名誉会长、顾问等职务。 朱光潜是我国现代美学的开拓者和奠基者之一,也是第一个在中国广泛介绍西方美学的人。主要著作有《文艺心理学》《悲剧心理学》《西方美学史》《给青年的二十封信》《谈修养》《谈美》《诗论》《谈文学》《美学拾穗集》等。他以自己深湛的研究沟通了西方美学和中国传统美学,沟通了旧的唯心主义美学和马克思主义美学,沟通了“五四”以来中国现代美学和当代美学。他是中国美学史上一座横跨古今、沟通中外的“桥梁”,是我国现当代最负盛名并赢得崇高国际声誉的美学大师。 2(写作背景 《谈美》写于1932年,由开明书店出版。在这本书中,朱光潜善于把西方各种美学、心理学理论与中国人熟悉的文学现象结合起来,和鲜活的审美现象结合起来,书中的精辟见解层出不穷,让人读 起来津津有味。他怀着一种崇高的使命感和社会责任感来写这一本不厚的小册子。他用通俗易懂的方式和明白如话的语言把高深的美学问题讲得深入浅出,引人入胜,从而起到净化读者心灵、提升一代青年精神境界的作用。 学习过程: 一、初步感知(A级) 作者如何解释美是什么, 二、品读探究(C级) 1(结合生活中的例子,如何理解“欣赏的距离”, 2(结合生活中的例子,如何理解“移情作用”, 3(结合生活中的例子,如何理解美感与快感的区别, 三、学习反思(B级) 美感产生的三个必要条件是什么, 四、布置作业(D级) 如何理解“人生艺术化”,谈谈你的看法, 五、拓展阅读 读《谈美》 胡经之 看完朱光潜的《谈美》,感觉好像听到了苏格拉底的谈话,翻看了塞涅卡的书信,阅读了蒙田的随笔,一些众人皆知的道理和微不足道的小事,被他用来阐明自己的理论。深入浅出,微言大义。一滴水虽小,却能从中看见七彩的阳光;一片枯叶的飘落虽不起眼,却能预示着秋的来临;眼睛在整个身体中占的比例 很小,但你仔细观察它,却能洞悉人心;脉搏的跳动经常让人意识不到,但医生认真感受它,则可以了解你的身体状况。理论希望为我们揭示事物的本质,它是抽象的,但它却来源于无数的具体现象。在生活中,我们看到的现象太多了,心中的误解也太多了,而这本书,正是一位能够帮助我们拨开迷雾,透过现象看本质的朋友。 我们生活的世界像一本有歧义的书,书里有着各种各样的解释。“真”的领域有逻辑学来研究,“善”的领域有伦理学来研究,“感性”的领域则有美学来研究。美从哪里来,美是什么,美是怎样的,一连串的问题等待着美学去回答。开卷有益,作善至祥。读完《谈美》,确实对美学有了一点体味。 (1)美,是客观存在还是主观感受 。 记得好像是2000年,国际大专辩论会上有这样一个辩题:美是客观存在还是主观感受。当时场上双方唇枪舌剑,说得天花乱坠,把我弄得一头雾水。说它是客观存在,有些大家看起来很难看人的,在艺术家那里也许就是美人,这显然是人主观意愿的作用;说它是主观感受,倒也不完全是,毕竟美是有一定标准的,不是纯主观的。 世界,远远不止于地球这颗行星,远远不止于我们整个物理性的宇宙。世界物理性地和逻辑性地从人身体的现在条件出发向外伸展。它从我们的位置、时间和方式出发向外伸展——在所有的空、时间和逻辑方向上。世界太大了,无限的大,我们则太渺小。还是禅者说的好:穷诸玄辩,若一毫置于太虚;竭世枢机,似一滴投于巨壑。 (2)人,诗意地栖居在大地上。 看完这一节,又想起叔本华的一段话:“事实上当我们正在进行某项伟大事业或创造某件不朽作品时,并没有意识到这件事的本身。我们每日辛勤劳作,考虑的只是达到眼前的目标,完成当下的计划。然而,只有当我们把人生看作一个联系着的整体时,我们的品质和能力才能展现其真实面貌。我们也才能明白,在形形色色的情境中,仿佛是某种幸福的灵感引导着我们在千百条趋向毁灭的道路中,选择那唯一真实的途径。”日子本质上都是一样的,它可以是普通的,也可以是特殊的,关键在于你怎样去看它。人生也是如此,你觉得它是一本日历,每天撕掉一页就少一页,那你的人生就是麻木的、情趣干枯的。你觉得它是一 本日记,每天记录一些故事就会使你的生命丰富一些,那你的人生就是积极的、情趣丰富的。在这个世界上,青史留名、活得轰轰烈烈的人毕竟是少数,我们不过都是些平凡人,但我们要相信自己在这个世界上的独一无二。我们是平凡人,但我们要做有情趣的平凡人,诗意地栖居在大地上。 (3)慢慢走,欣赏啊~ 读完最后一节,心里觉得很震撼。“人生的艺术化”,可以说是朱光潜美学研究的理想目标,这是一个十分诗意的令人神往的境界。“人生本来就是一种较广义的艺术。每个人的生命史就是他自己的作品。这种作品可以是艺术的,也可以是不艺术的,正犹如同是一种顽石,有人能把它雕刻成一座伟大的雕像,而另一个人却不能使它‘成器’,分别全在性分与修养。”任何理论和学说,都来源于对于世界和生活的思考,但却又经常给人感觉远离生活之感。想起孔子曾和曾点、季路、冉有、公西赤坐在一起,听他们的政治理想。四人说完后,孔子最终赞赏曾点的意思。曾点希望自己能够陶醉在礼乐教化所带来的社会和谐、人民安乐的景象之中,使礼乐教化真正渗透到人们的生活里。朱光潜似乎也有同样的意思,美是艺术的,它就在生活中。我们艺术的生活,让美渗透在生活中,使生活即是艺术,即是美。这是何等美丽的境界啊。中国的民乐其实很美,宁静、淳朴且毫无功利性,但是我们的心太浮躁了,听不下去这些“土气”的东西。但是,在伤心的时候偶然听到《二泉映月》,在高兴的时候凑巧听到《金蛇狂午》,也会怦然心动,深陷其中不能自拔。夜来香其实白天也很香,但是很少有人闻得到,因为白天的人心太浮躁。如果一个人白天的心也很沉静,就会发现夜来香、桂花、七里香,连酷热的中午也是香的。城市里的绿地越来越多了,每天从旁边经过的时候,总是匆匆忙忙。其实只要稍作停留,会发现草地里的野花和蒲公英很美,在其间蹦蹦跳跳的喜鹊和麻雀也很可爱。世界上并不缺少美,少的只是发现美、感受美的心灵。还是那句话:“慢慢走,欣赏啊~” 《谈美》导学案参考答案 一、初步感知 作者如何解释美是什么, 答案: 他举例说:同一棵古松,木材商看到的是它值多少钱,植物学家看到的是它的 生长结构,而画家则看到它的气质、风韵,也就是“美感”。这就是我们对待客观对“实 用的”、“科学的”、“美感的”三种不同的态度。通过木材商、植物学家和画家对一棵古 松的不同由此结出:美是无功利性的,是无用之用。而人之所以是高级动物,在于追求“无用”上。 二、品读探究 1(结合生活中的例子,如何理解“欣赏的距离”, 答案: 审美者与审美对象之间要保持一定的心理距离才能产生美感体验。所谓心理距离是指审美者不要去注意和思考与审美对象的美学价值无关的事情。 譬如我们乘坐轮船在海上旅行,遇到一场突如其来的大雾,对于绝大多数人来说,是一个极不愉快的经验。对于轮船误期的焦虑,对不可预见的危险的担忧,再加上潮湿的空气,水手们紧张地在船上跑动,汽笛尖利的声音此起彼伏,这一切都使人们的旅行兴致全无。但是,假如换一个角度看,也许你会看到下面这幅景色: 轻烟似的薄纱,笼罩着这平谧如镜的海水,许多远山和飞鸟被它盖上一层面网,都现出梦境的依稀隐约,它把天和海联成一气,你仿佛伸一只手就可握住在天上浮游的仙子。你的四围全是广阔、沉寂、秘奥和雄伟,你见不到人世的鸡犬和烟火,你究竟在人间还是在天上,这也许就是“欣赏的距离”所产生的美感。 2(结合生活中的例子,如何理解“移情作用”, 答案: 移情作用是把自己的情感移到外物身上,觉得外物也有同样的情感.,好像自己欢喜时,所看到景物都像在微笑.;悲伤时,景物也像在叹气。自古以来,文人墨客在诗歌创作中往往赋予自然景物以人的行动性格,生命及思想感情,使自然景物反映出人和社会生活的美。这就是美学所称的诗歌创作的“移情作用”。 诗歌中的“移情现象”处处可见。例如唐代杜甫在《春望 》中写道“感时花溅泪,恨别鸟惊心。”南宋辛弃疾在《贺新郎》中写道:“我见青山多妩媚,料青山见我应如是。”等等。 3(结合生活中的例子,如何理解美感与快感的区别, 答案: 美感不同于快感,美感与实用活动无关,快感则起于实际要求的满足。美感态度不带意志,快感却是强烈的占有欲的满足。”美感和快感是很容易分别的。美感与实用活动无关,而快感则起于实际要求的满足。口渴时要喝水,喝了水就得到快感;腹饥时要吃饭,吃了饭也就得到快感。 三、学习反思 美感产生的三个必要条件是什么, 答案:(1)我们的态度是美感产生的首要条件。 由于每个人的出发点不同,其观察事物的立足点就不同,同样的事物在不同心态的人的心理或视觉上都会有其独特的反应。而那实用层次的态度是我们的本能对物质欲望的反映,因而它带有很强的功利性;但对于美或美感而言,它是物质活动之外的有意义,是精神的,也是无功利的,就如文中说的“在审美的态度中,我们的注意力专于事物本身的形象,心理活动偏于直觉,活动空间也更广阔无疆”。美是从我们的态度中得来的,那么美代表了一种精神;而作为精神与心灵的主体的我们,我们会从艺术作品中获得自己心灵的满足,也体味出了在实用与科学中无存的性质,此时我们便是获得了美感。因为审美的态度正是审美及美的存在的意义,所以艺术作品才能获得不朽,于人才更亲切。 (2)距离是美感产生的另一个因素。 作者引用典故“当局者迷,旁观者清”说明:我们在审美的过程中是不能以一种 实用的态度,因为美的差别是艺术与实际人生的距离,这个距离不能太过,也不能不 及,只有“均衡”、“适当”才能引起美感。 (3)移情是美感产生的又一个条件。 作者在第三章说到,“推己及物”即“移情”会让我们对于自身以外的客观事物, 产生了审美的情素,移情和美感经验是有密切关系的。但移情不一定就是美感经验的 产生,不过美感经验中却常含有移情作用。经过一系列的“推己及物”和“以物及我”, 即是移情的美感经验---物与我情趣之间的往复回流后,心里有美的意象,美的情素, 则是美感产生的功效。 四、布置作业 如何理解“人生艺术化”,谈谈你的看法, 答案: 所谓“人生艺术化”就是主张审美、艺术、人生相统一,倡导主体以美的艺术 精神来濡染提升个体的人格情致与生命境界,从而建构诗意的人格和美的人生,实现并享 受生命、人生的意义与韵味。简单地说,“人生艺术化”就是追求艺术的人格和审美地 生活。在科技因素、实用理性占据重要地位的现实生活中,人生艺术化的理想及其精神 具有独特的意义。人生艺术化在本质上是试图从生存的事实超向生命的意义。它突出了 人及其现实生存中情感、理想、诗意超越这些富有意义而又常常自觉不自觉被遗忘的纬 度;同时,它也以对这些纬度的倡扬体现了对于现实实践中科学与艺术、理性与情感、物 质与精神、尚实与超越和谐共生的期待,对于人的知情意全面和谐生成的期待。 24 附录:中华文化精神 1 《中华文化精神》导学案 【学习目标】 1、中华文化精神对中华儿女的影响 2、中华文化的主要特点 3、如何正确看待中华文化精神 4、中华文化精神对社会发展的意义 【学习难点】 1、 中华文化精神的特点及对人们的深远影响 2、 领悟中华文化精神对社会发展的意义 3、 在学习中对学生培养学生对中华文化的热爱 【学法指导】 诵读法、质疑法、探究法、讨论法(引导学生结合自己的知识储备和生活体验进行讨论) 【知识链接】 1、作者简介 袁行霈(1936— ),男,1936年4月出生于山东济南,原籍江苏武进。第八、九届全国政协常委,第十届全国人大常委,民盟中央副主席,国务院学位委员会委员、全国高校古籍整理工作委员会委员、北京大学国学研究院院长,《国学研究》主编。 袁行霈出身于传统的读书人家庭,家庭培养了他对古典文学的兴趣。1953年他考入北京大学中文系, 1957年大学毕业留校任教,从此开始了教学与科研生涯。 1957年至1966年,袁行霈在北大讲授中国文学史,结合备课系统读书和撰写论文。同时,他跟随导师林庚先生一起主编了《魏晋南北朝文学史参考资料》。其间,他多 次下乡、下厂、下煤矿劳动锻炼。文革中他下放“五七干校”劳动,回北大后,参加了集体编写《中国小说史》的工作。同时,他独自撰写了《山海经初探》、《汉书艺文志小说家考辨》、《魏晋玄学中的言意之辨与中国古代文艺理论》等论文在1979年发表,开始引起学术界的注意。 1977年高校恢复招生后,袁行霈在北大为本科生和研究生讲授中国文学史、中国诗歌艺术研究、陶渊明研究、唐诗研究、李贺研究、唐宋词研究等课程。他在教学工作中不断探索进取,取得较好的效果。1979年晋升讲师,1980年任副教授,1984年成为中文系特聘教授,1986年取得博士生导师资格。1992年,袁行霈兼任北京大学中国传统文化研究中心主任、《国学研究》主编;1994年任大型系列电视专题片“中华文明之光”(150集)总顾问;1999年出任北京大学人文学部主任;2000年又出任北京大学国学研究院院长。 1982年至1983年,他应日本东京大学的邀请,前往讲授陶渊明研究等五门课程,受到广泛称赞,东京大学中文系主任在致北大的公函中说他“学识渊博,人格高尚”,并要求延长他的任期,他遵照北大的安排按时回国。1992年至1993年,他任新加坡国立大学中文系客座教授。1997年以美国哈佛燕京学社访问学者身份在哈佛大学访问研究,并在哈佛、耶鲁、哥伦比亚、华盛顿、夏威夷等大学演讲。1998年再次任新加坡国立大学中文系客座教授。2004年任香港城市大学客座教授。2005年被北京大学确认为资深教授,享受理科院士待遇,同年被新加坡南洋理工大学聘为教授(与北京大学联聘)。 由于他在教学工作中做出的贡献,1989年获全国高等学校首届优秀教学成果奖国家级特等奖;1991年获国家教委和人事部授予的全国教育系统劳动模范称号暨人民教师奖章;1993年获北京市人民政府授予的北京市人民教师称号(公众推选的十佳教师);1995年获北京市先进工作者称号。 袁行霈主要的著作有:《中国诗歌艺术研究》,此书于1991年获北京市第二届哲学社会科学优秀成果(著作)一等奖,于1992年获国家教委授予的高等学校出版社优秀学术专著特等奖。《中国诗学通论》(合著)于1995年获第二届国家图书奖提名奖,教育部授予的全国高等学校第二届优秀社科成果奖二等奖。他主编的《中国文学史》四卷本于2000年获北京市第六届哲学社会科学优秀成果(著作)特等奖,2003年又获国家图书奖、高等学校优秀教材奖一等奖。此外还著有《陶渊明研究》、《中国文学概论》、《当代学者自选文库•袁行霈卷》、《中国文学史纲要》(二)、《中国文言小说书目》(合编)、《历代名篇赏析集成》(主编)、《袁行霈学术文化随笔》、《陶渊明集笺注》、《唐诗风神及其他》、《中华文明史》四卷本(主编之一)等。有的著作被译成日文和韩文,在日本和韩国出版。 2、探索背景 (1) 、中华文化精神有独特价值,需要中国人去弘扬 (2) 、西方文化精神的冲击给中国人带来的精神问题 【学习过程】 一、品读全文,要求读准字音(A级) 熏陶( ) 贲卦( ) 彖辞( ) 祭祀( ) (((((( 驾驭( ) 祆教( ) 商贾( ) 糟粕( ) (((((二、整体感知,试概括每个部分的含义(B级) 三、语段品读,回答问题 1、在第一部分中,概括中华文化是怎样产生的, 2、文章的主体部分,具体阐述了中华文化精神的哪些特点, 3、结合第三部分,谈谈我们该如何传承中华文化精神, 4、结合最后一段,谈谈你对传承中华文化精神重大意义的理解, 【作业布置】 1(广大青年对一些外国节日目,津津乐道,并且热烈庆贺。相反,一些中国传统节日目却差不多淡忘了,只有春节还保留着重大节日的风采。对此,我们应该( ) A、全盘吸收外来文化 B、通过立法禁止外国的节日 C、大力弘扬中国的优秀的文化传统 D、固守中国的习俗 2(千百年来,各族人民在中华大地上生息繁衍,相互团结,相互学习,共同创造了灿烂的中华文化。博大精深的中华文化包括 ( ) ?中国的四大名著 ?中国古代的四大发明 ?异彩纷呈的外国文学艺术 ?充满智慧的中国哲学 A(??? B(??? C(??? D(??? 3(从孟子的“富贵不能淫,威武不能屈”到夏明翰的“砍头不要紧,只要主义真”,从“愚公移山”精神到“载人航天精神”、“劳模精神”,伟大的民族精神成为中华民族进步的不竭动力。 (1)列举一位我国古代或近代史上体现伟大民族精神的代表人物和相关史实,谈谈表现出来的民族精神。(4分) (2)民族精神的内容很丰富,就你最有感触的一个方面,结合自己的学习生活,说说你该怎么做,(5分) 4(一位中国留学生在海外曾有这样一段经历:一位法国老太太热情地免费教他女儿学法语。原来老太太热爱自己的祖国,希望更多的人学习她的母语。这位留学生想到自己总是要求女儿学好外语,心里万分难过。从此他也开始免费教外国入学习汉语,他要让更多的外国人知道中国语言是多么博大精深。(提示:汉语是中华民族创造的优秀文化成果) (1)大家就材料中的留学生行为的意义进行了交流(你有何想法。 (2)请结合上述材料,谈谈为了弘扬和培育民族精神(我们应该如何对待民族文化和外来文化? (3)通过讨论,同学们认为应以实际行动向材料中的留学生学习,你准备采取哪些行动? 《中华文化精神》导学案 参考答案 【学习过程】 一、 熏陶(xūn táo) 贲卦(bì ) 彖辞(tuàn) 祭祀 (jì sì ) (((((( 驾驭(yù ) 祆教(xiān ) 商贾(gǔ ) 糟粕(zāo pò) ((((( 二、 第一部分:中华文化精神对中华儿女的深远影响。 第二部分:中华文化的特点。 第三部分:如何继承和弘扬中华文化。 第四部分:对中华文化必将推动社会巨大发展的坚定信念。 三、1、自古以来,中华民族就在中华大地上劳动和生活,各族人民相互团结,相互学习,用自己的勤劳和智慧共同开发了祖国的大好河山,创造了灿烂的中华文化。 2、(1)、中华文化包含着强烈的人文精神。(2)、中华文化是尚群的文化。 (3)、中华文化是尚和的文化 (4)、中华文化注重整体思维。 3、(1)我们要认清先人给我们留下的文化遗产,弘扬优秀的中华文化精神,吸取那些至今仍然有助于社会前进的东西,以增进民族凝聚力,提高民族自信心。 (2)我们要认清传统文化的局限性,清理遗产,包括剔除糟粕这个艰巨的任务。 4、我们只要坚持汲取中华文化中的精华,同时吸取世界各民族文化中的优秀成分,不断开拓创新,就必定会使中华文化永葆青春、不断创造辉煌,为人类社会的进步做出更大的贡献 【作业布置】 1、C、 2、B 3、(1)例如:林则徐的虎门销烟,捍卫了国家和民族的尊严,长了中国人的志气,体现了强烈的爱国主义精神。 (2)我们要发扬自强不息的民族精神,在学习中要做到乐观开朗、自强自信。遇到挫折不气馁,不退缩,如考试失败后不能心灰意冷,要善于找到差距,弥补不足,争取成功。 4、(1)留学生从法国老太太的行为中意识到宣传母语、弘扬中华民族的文化是热爱祖国的表现,并付诸实际行动(这有利于维护国家和民族荣誉,增强民族自豪感、自信心和自尊心。民族文化是各自人民勤劳和智慧的结晶(是民族的根。更是一个民族的灵魂。文化的力量深深熔铸在巾华民族的生命力、创造力和凝聚力之中。留学生的行为有助于弘扬民族文化,增强民族凝聚力。当今国际竞争激烈,综合国力的竞争也包括文化力的竞争,留学生的行为有助于提高我国的综合国力。 (2)中华民族创造了辉煌的科技文化,我们应深深植根于民族土壤(积极传承优秀的传统文化,保持中国特色(增强民族文化的认同感。汉语是中华民族创造的优秀文化成果之一,博大精深,我们应像那位留学生那样,学好它,用好它,并力求将其发扬光大。我们应借鉴外来文化优秀成果,同时对待传统文化还要结合新的时代和实践加以创新,这样才能更好地保持中国传统文化的旺盛活力。 (3)我决定要说好普通话,写好中国字,传承中华民族语盲文化;我准备更多地了解我国 的风土人情,增强对我国民族文化的认同感;我准备跟美术班的老师认真学习国域(珍惜学 习中华民族优秀文化的机会.
本文档为【高中语文必修4导学案】,请使用软件OFFICE或WPS软件打开。作品中的文字与图均可以修改和编辑, 图片更改请在作品中右键图片并更换,文字修改请直接点击文字进行修改,也可以新增和删除文档中的内容。
该文档来自用户分享,如有侵权行为请发邮件ishare@vip.sina.com联系网站客服,我们会及时删除。
[版权声明] 本站所有资料为用户分享产生,若发现您的权利被侵害,请联系客服邮件isharekefu@iask.cn,我们尽快处理。
本作品所展示的图片、画像、字体、音乐的版权可能需版权方额外授权,请谨慎使用。
网站提供的党政主题相关内容(国旗、国徽、党徽..)目的在于配合国家政策宣传,仅限个人学习分享使用,禁止用于任何广告和商用目的。
下载需要: 免费 已有0 人下载
最新资料
资料动态
专题动态
is_841159
暂无简介~
格式:doc
大小:411KB
软件:Word
页数:245
分类:初中语文
上传时间:2017-10-07
浏览量:210